Sunteți pe pagina 1din 117

PECI 2013 Nvel 2 Mdulo 1

Robrio Bacelar, Fbio Brochero, Svio Ribas e Paulo Rodrigues 1 de Fevereiro de 2013

RA

FT

D RA FT

Contedo
1 Informaes sobre o Programa 1.1 Apresentao . . . . . . . . . . . . . . . . . . . . . . . . . . . . . . . . . . . . . . . . . . . . . . . . . . . . . . 1.2 Utilizando o Frum . . . . . . . . . . . . . . . . . . . . . . . . . . . . . . . . . . . . . . . . . . . . . . . . . . 7 7 7 7 8

1.3 Avaliaes e Simulados . . . . . . . . . . . . . . . . . . . . . . . . . . . . . . . . . . . . . . . . . . . . . . . . 1.4 Como participar da OBM? . . . . . . . . . . . . . . . . . . . . . . . . . . . . . . . . . . . . . . . . . . . . . . .

lgebra
2 Produtos Notveis e Fatorao

FT
3

9
10

2.1 Produtos Notveis . . . . . . . . . . . . . . . . . . . . . . . . . . . . . . . . . . . . . . . . . . . . . . . . . . . 10 2.2 Tringulo de Pascal . . . . . . . . . . . . . . . . . . . . . . . . . . . . . . . . . . . . . . . . . . . . . . . . . . 10 2.2.1 Exemplos . . . . . . . . . . . . . . . . . . . . . . . . . . . . . . . . . . . . . . . . . . . . . . . . . . . . 11 2.3 Fatorao . . . . . . . . . . . . . . . . . . . . . . . . . . . . . . . . . . . . . . . . . . . . . . . . . . . . . . . . 11 2.4 Potnciao em R . . . . . . . . . . . . . . . . . . . . . . . . . . . . . . . . . . . . . . . . . . . . . . . . . . . 12 2.4.1 Potncia com Expoente Natural . . . . . . . . . . . . . . . . . . . . . . . . . . . . . . . . . . . . . . . . 12 2.4.2 Propriedades . . . . . . . . . . . . . . . . . . . . . . . . . . . . . . . . . . . . . . . . . . . . . . . . . . 12 2.5 Radiciao em R . . . . . . . . . . . . . . . . . . . . . . . . . . . . . . . . . . . . . . . . . . . . . . . . . . . . 13 2.5.1 Propriedades . . . . . . . . . . . . . . . . . . . . . . . . . . . . . . . . . . . . . . . . . . . . . . . . . . 13 2.6 Potncia com Expoente Racional . . . . . . . . . . . . . . . . . . . . . . . . . . . . . . . . . . . . . . . . . . . 13 2.7 Radical Duplo . . . . . . . . . . . . . . . . . . . . . . . . . . . . . . . . . . . . . . . . . . . . . . . . . . . . . . 13 2.8 Problemas Propostos 3

Equaes do Segundo Grau

RA

. . . . . . . . . . . . . . . . . . . . . . . . . . . . . . . . . . . . . . . . . . . . . . . . . 13 18

3.1 Equao do Segundo Grau

. . . . . . . . . . . . . . . . . . . . . . . . . . . . . . . . . . . . . . . . . . . . . . 18

3.1.1 Forma Cannica do Trinmio de Segundo Grau . . . . . . . . . . . . . . . . . . . . . . . . . . . . . . 18 3.1.2 Frmula de Bscara . . . . . . . . . . . . . . . . . . . . . . . . . . . . . . . . . . . . . . . . . . . . . . 18

3.1.3 Soma das razes da Equao do Segundo Grau . . . . . . . . . . . . . . . . . . . . . . . . . . . . . . . 19 3.1.4 Produto das Razes da Equao de Segundo Grau . . . . . . . . . . . . . . . . . . . . . . . . . . . . . 19 3.2 Problemas Resolvidos . . . . . . . . . . . . . . . . . . . . . . . . . . . . . . . . . . . . . . . . . . . . . . . . . 19 3.3 Forma Fatorada do Trinmio de Segundo Grau . . . . . . . . . . . . . . . . . . . . . . . . . . . . . . . . . . 20 3.3.1 Exerccios Propostos . . . . . . . . . . . . . . . . . . . . . . . . . . . . . . . . . . . . . . . . . . . . . . 20

Combinatria
4 Paridade 4.1 Problemas Propostos

23
24 . . . . . . . . . . . . . . . . . . . . . . . . . . . . . . . . . . . . . . . . . . . . . . . . . 31

4 5 Princpio da Casa dos Pombos 5.2 Pombos Geomtricos 5.3 Problemas Propostos

CONTEDO 33

5.1 Exemplos Introdutrios . . . . . . . . . . . . . . . . . . . . . . . . . . . . . . . . . . . . . . . . . . . . . . . . 33 . . . . . . . . . . . . . . . . . . . . . . . . . . . . . . . . . . . . . . . . . . . . . . . . . 34 . . . . . . . . . . . . . . . . . . . . . . . . . . . . . . . . . . . . . . . . . . . . . . . . . 35

5.4 Ramsey . . . . . . . . . . . . . . . . . . . . . . . . . . . . . . . . . . . . . . . . . . . . . . . . . . . . . . . . . 35 5.4.1 Problemas Propostos . . . . . . . . . . . . . . . . . . . . . . . . . . . . . . . . . . . . . . . . . . . . . . 36 5.5 Pombos e Divisibilidade . . . . . . . . . . . . . . . . . . . . . . . . . . . . . . . . . . . . . . . . . . . . . . . . 37 5.5.1 Problemas Propostos . . . . . . . . . . . . . . . . . . . . . . . . . . . . . . . . . . . . . . . . . . . . . . 40 5.6 Combinatria Aditiva . . . . . . . . . . . . . . . . . . . . . . . . . . . . . . . . . . . . . . . . . . . . . . . . . 40 5.7 Problemas Avanados . . . . . . . . . . . . . . . . . . . . . . . . . . . . . . . . . . . . . . . . . . . . . . . . . 41 5.8 Problemas Complementares . . . . . . . . . . . . . . . . . . . . . . . . . . . . . . . . . . . . . . . . . . . . . 42 5.9 Problemas Propostos 6 Grafos . . . . . . . . . . . . . . . . . . . . . . . . . . . . . . . . . . . . . . . . . . . . . . . . . 43 45

6.1 Viajando em Pecilndia . . . . . . . . . . . . . . . . . . . . . . . . . . . . . . . . . . . . . . . . . . . . . . . . 45 6.2 Entre Amigos . . . . . . . . . . . . . . . . . . . . . . . . . . . . . . . . . . . . . . . . . . . . . . . . . . . . . . 46 6.4 Problemas Propostos . . . . . . . . . . . . . . . . . . . . . . . . . . . . . . . . . . . . . . . . . . . . . . . . . 51 6.3 Tringulos em Grafos . . . . . . . . . . . . . . . . . . . . . . . . . . . . . . . . . . . . . . . . . . . . . . . . . 49

Teoria dos Nmeros


7 Ferramentas Bsicas

FT

53
54

7.1 Aquecendo os motores . . . . . . . . . . . . . . . . . . . . . . . . . . . . . . . . . . . . . . . . . . . . . . . . 54 7.1.2 Exerccios para esquentar . . . . . . . . . . . . . . . . . . . . . . . . . . . . . . . . . . . . . . . . . . . 54 7.2 Ferramentas preliminares . . . . . . . . . . . . . . . . . . . . . . . . . . . . . . . . . . . . . . . . . . . . . . . 55 7.2.1 Princpio da Induo Finita - 1 forma . . . . . . . . . . . . . . . . . . . . . . . . . . . . . . . . . . . . 55 7.2.2 Princpio da Induo Finita - 2 forma . . . . . . . . . . . . . . . . . . . . . . . . . . . . . . . . . . . . 55 7.2.3 Princpio da Boa Ordenao . . . . . . . . . . . . . . . . . . . . . . . . . . . . . . . . . . . . . . . . . . 56 7.2.4 Princpio da Casa dos Pombos . . . . . . . . . . . . . . . . . . . . . . . . . . . . . . . . . . . . . . . . 56 7.2.5 Exerccios de fixao . . . . . . . . . . . . . . . . . . . . . . . . . . . . . . . . . . . . . . . . . . . . . . 56 7.3 Divisibilidade . . . . . . . . . . . . . . . . . . . . . . . . . . . . . . . . . . . . . . . . . . . . . . . . . . . . . . 57 7.3.1 Mltiplos e divisores . . . . . . . . . . . . . . . . . . . . . . . . . . . . . . . . . . . . . . . . . . . . . . 57 7.3.2 O algoritmo de Euclides . . . . . . . . . . . . . . . . . . . . . . . . . . . . . . . . . . . . . . . . . . . . 57 7.3.3 O Teorema Fundamental da Aritmtica . . . . . . . . . . . . . . . . . . . . . . . . . . . . . . . . . . . 58 7.4 Problemas Propostos . . . . . . . . . . . . . . . . . . . . . . . . . . . . . . . . . . . . . . . . . . . . . . . . . 60

Geometria
8 9 Problemas de Aquecimento Bases Mdias, Medianas e Tesouros

RA

7.1.1 Critrios de divisibilidade . . . . . . . . . . . . . . . . . . . . . . . . . . . . . . . . . . . . . . . . . . . 54

62
63 65

9.1 A Base Mdia . . . . . . . . . . . . . . . . . . . . . . . . . . . . . . . . . . . . . . . . . . . . . . . . . . . . . . 65 9.2 O Primeiro Tesouro . . . . . . . . . . . . . . . . . . . . . . . . . . . . . . . . . . . . . . . . . . . . . . . . . . 66 9.3 Outro Tesouro Perdido . . . . . . . . . . . . . . . . . . . . . . . . . . . . . . . . . . . . . . . . . . . . . . . . 67 9.4 Problemas Propostos . . . . . . . . . . . . . . . . . . . . . . . . . . . . . . . . . . . . . . . . . . . . . . . . . 68 9.5 Medianas . . . . . . . . . . . . . . . . . . . . . . . . . . . . . . . . . . . . . . . . . . . . . . . . . . . . . . . . 69 9.6 Mediana em um tringulo retngulo . . . . . . . . . . . . . . . . . . . . . . . . . . . . . . . . . . . . . . . . . 70 9.7 Problemas Propostos . . . . . . . . . . . . . . . . . . . . . . . . . . . . . . . . . . . . . . . . . . . . . . . . . 72

CONTEDO 10 Problemas Extras

5 73

Sesses de Problemas
11 12 13 14 15 Sesso 1 Sesso 2 Sesso 3 Sesso 4 Sesso 5

74
75 76 77 78 79

Avaliaes e Simulados
16 17 Prova de Seleo 2013 Avaliao 1

80
81

FT

83

17.1 lgebra . . . . . . . . . . . . . . . . . . . . . . . . . . . . . . . . . . . . . . . . . . . . . . . . . . . . . . . . . 83 17.2 Combinatria . . . . . . . . . . . . . . . . . . . . . . . . . . . . . . . . . . . . . . . . . . . . . . . . . . . . . . 83 17.3 Geometria . . . . . . . . . . . . . . . . . . . . . . . . . . . . . . . . . . . . . . . . . . . . . . . . . . . . . . . . 83 17.4 Nmeros . . . . . . . . . . . . . . . . . . . . . . . . . . . . . . . . . . . . . . . . . . . . . . . . . . . . . . . . . 84 18 19 20 21 Simulado 1 Simulado 2 85 86 87 95

Gabarito da Prova de Seleo Gabarito da Avaliao

21.1 lgebra . . . . . . . . . . . . . . . . . . . . . . . . . . . . . . . . . . . . . . . . . . . . . . . . . . . . . . . . . 95 21.2 Combinatria . . . . . . . . . . . . . . . . . . . . . . . . . . . . . . . . . . . . . . . . . . . . . . . . . . . . . . 96 21.3 Geometria . . . . . . . . . . . . . . . . . . . . . . . . . . . . . . . . . . . . . . . . . . . . . . . . . . . . . . . . 96 21.4 Nmeros . . . . . . . . . . . . . . . . . . . . . . . . . . . . . . . . . . . . . . . . . . . . . . . . . . . . . . . . . 98 22 23 Gabarito do Simulado 1 Gabarito do Simulado 2

RA

100 103

A Utilizando TEX no Frum 105 A.1 O que TEX? . . . . . . . . . . . . . . . . . . . . . . . . . . . . . . . . . . . . . . . . . . . . . . . . . . . . . . 105 A.1.1 Knuth inventou o TEX... . . . . . . . . . . . . . . . . . . . . . . . . . . . . . . . . . . . . . . . . . . . . 105 A A.1.2 ... e Lamport criou o LTEX . . . . . . . . . . . . . . . . . . . . . . . . . . . . . . . . . . . . . . . . . . . 105 A.2 Escrevendo e desenhando no frum . . . . . . . . . . . . . . . . . . . . . . . . . . . . . . . . . . . . . . . . . 106 A.2.1 Exemplos Bsicos . . . . . . . . . . . . . . . . . . . . . . . . . . . . . . . . . . . . . . . . . . . . . . . . 107 A.2.2 Letras Gregas . . . . . . . . . . . . . . . . . . . . . . . . . . . . . . . . . . . . . . . . . . . . . . . . . . 107 A.2.3 Aritmtica . . . . . . . . . . . . . . . . . . . . . . . . . . . . . . . . . . . . . . . . . . . . . . . . . . . . 107 A.2.4 Geometria . . . . . . . . . . . . . . . . . . . . . . . . . . . . . . . . . . . . . . . . . . . . . . . . . . . . 107 A.2.5 Setas . . . . . . . . . . . . . . . . . . . . . . . . . . . . . . . . . . . . . . . . . . . . . . . . . . . . . . . 108 A.2.6 Smbolos Diversos . . . . . . . . . . . . . . . . . . . . . . . . . . . . . . . . . . . . . . . . . . . . . . . 108 A.2.7 Conjuntos . . . . . . . . . . . . . . . . . . . . . . . . . . . . . . . . . . . . . . . . . . . . . . . . . . . . 108 A.2.8 Matrizes e Determinantes . . . . . . . . . . . . . . . . . . . . . . . . . . . . . . . . . . . . . . . . . . . 108

CONTEDO A.2.9 Somatrios e Produtrios . . . . . . . . . . . . . . . . . . . . . . . . . . . . . . . . . . . . . . . . . . . 109 A.2.10 Diversos . . . . . . . . . . . . . . . . . . . . . . . . . . . . . . . . . . . . . . . . . . . . . . . . . . . . . 109


A A.2.11 Construindo figuras com LTEX . . . . . . . . . . . . . . . . . . . . . . . . . . . . . . . . . . . . . . . . 110

B Competies e Prmios

113

B.1 Torneio Internacional das Cidades . . . . . . . . . . . . . . . . . . . . . . . . . . . . . . . . . . . . . . . . . . 113 B.2 Asian Pacific Mathematical Olympiad . . . . . . . . . . . . . . . . . . . . . . . . . . . . . . . . . . . . . . . . 113 B.3 Olimpada de Matemtica do Cone Sul . . . . . . . . . . . . . . . . . . . . . . . . . . . . . . . . . . . . . . . 114 B.4 Olimpada Iberoamericana de Matemtica . . . . . . . . . . . . . . . . . . . . . . . . . . . . . . . . . . . . . 114 B.5 Romanian Masters in Mathematics . . . . . . . . . . . . . . . . . . . . . . . . . . . . . . . . . . . . . . . . . . 115 B.6 Olimpada de Matemtica dos Pases de Lngua Portuguesa . . . . . . . . . . . . . . . . . . . . . . . . . . . 115 B.7 Olimpada Rioplatense de Matemtica . . . . . . . . . . . . . . . . . . . . . . . . . . . . . . . . . . . . . . . . 115 B.8 Olimpada Internacional de Matemtica . . . . . . . . . . . . . . . . . . . . . . . . . . . . . . . . . . . . . . . 116

RA

FT

Informaes sobre o Programa

1.1 Apresentao
Caro aluno, parabns por ter sido escolhido para participar das atividades virtuais do PECI 2013. O objetivo do PECI preparar alunos para competies internacionais de matemtica. Desde 2009, quando foi criado, nossos alunos j obtiveram mais de 35 premiaes internacionais, incluindo 8 na Olimpada Internacional de Matemtica (IMO). Para alunos do nvel 2, o primeiro passo para participar de uma competio internacional ser premiado na de ser premiado nesta competio que acontecer ao longo do ano. Olimpada Brasileira de Matemtica OBM. Deste modo, como participante do PECI em 2013, seu objetivo deve ser o Para ajud-lo nessa preparao voc contar com um frum exclusivo na internet, para discutir os problemas e contedos propostos pela equipe de professores. Voc ser acompanhado por professores e realizar periodicamente simulados. Para um bom aproveitamento do programa estimamos um mnimo de 12h semanais de dedicao. Apresentamos em anexo os contedos do Mdulo 1, o qual ser utilizado durante 8 semanas. So contedos de lgebra, Combinatria, Geometria e Teoria dos Nmeros. Constam tambm os enunciados e gabaritos dos simulados Atenciosamente, aplicados no encontro presencial de Janeiro em Braslia e as solues da prova de seleo do PECI 2013.

RA
7

FT

Paulo Rodrigues Coordenador do PECI

1.2 Utilizando o Frum

mas propostos no material. Sinta-se vontade para propor problemas, esclarecer dvidas dos contedos abordados e tratar de tudo o que auxiliar a sua aprendizagem no PECI. Para ajudar neste estudo contaremos com os monitores. O trabalho dos monitores ser o de fomentar a discusso dos problemas, encaminhando ideias e corrigindo rumos. Para acessar o Frum do PECI voc dever digitar no navegador web o endereo abaixo:

O frum a nosso ambiente de ensino e aprendizagem. Sua principal utilidade a discusso e a resoluo dos proble-

http://peci.obmep.org.br/
Aps o carregamento do site, visualize os campos Nome de Usurio e Senha. Entre com seu Nome de Usurio e Senha e clique no boto Entrar. Estes dados so essenciais para acesso ao frum e so sigilosos. Clique na opo

PECI para acessar o frum. 1.3 Avaliaes e Simulados


Ao longo de todo o ano realizaremos no frum atividades obrigatrias para todos os alunos. So simulados, avaliaes e maratonas. Estas atividades so postadas no frum normalmente numa sexta-feira e o aluno deve postar as suas solues at a segunda-feira seguinte.

Informaes sobre o Programa Professor O calendrio abaixo mostra todas as atividades previstas em 2013 para a turma do nvel 2. As atividades marcadas

com sero realizadas nos encontros presenciais pelos alunos participantes dos mesmos na data de incio e nesta mesma data sero postadas no frum para os alunos que participam distncia. # 1 2 3 4 5 6 7 8 9 10 11 12 13 14 15 Incio 15/02 08/03 22/03 10/04 26/04 10/05 24/05 14/06 04/07 08/07 02/08 21/08 06/09 20/09 04/10 Fim 18/02 11/03 25/03 13/04 29/04 13/05 27/05 17/06 07/07 11/07 05/08 24/08 09/09 23/09 07/10 Atividade Simulado Maratona Avaliao 2 Maratona Avaliao 1 Simulado Maratona Maratona Simulado Simulado Avaliao Simulado Simulado Simulado Avaliao Objetivo / Assunto OBM 1a Fase Nmeros Mdulo 1 lgebra Mdulo 2 OBM 1a Fase Combinatria Geometria OBM 2a Fase OBM 3a Fase Mdulo 3 OBM 2a Fase Mdulo 4 OBM 3a Fase OBM 3a Fase

1.4 Como participar da OBM?

Para participar da OBM, sua escola deve se inscrever diretamente pelo site www.obm.org.br. As inscries estaro abertas de 25 de Maro a 30 de Abril de 2013.

Calendrio de provas

Primeira Fase: sbado, 15 de junho de 2013.

Segunda Fase: sbado, 21 de setembro de 2013.

Terceira Fase: sbado, 26 de outubro de 2013, (nveis 1, 2 e 3) e domingo, 27 de outubro de 2013, para os nveis 2

e 3 (segundo dia de prova).

RA

FT

D
semelhantes.

RA
PARTE I

lgebra
Professor Robrio Bacelar

Assuntos do Mdulo: Produtos Notveis e Fatorao; Equaes do Segundo Grau.

Pr-requisitos: Monmios; grau de um monmio; Monmios

FT
.

lgebra

Produtos Notveis e Fatorao

2.1 Produtos Notveis

1 Caso: Quadrado da Soma e Quadrado da Diferena de Dois Termos (x y)2 = x2 2xy + y2 2 Caso: Produto da Soma pela Diferena de Dois Termos

(x + y)(x y) = x2 y2

3 Caso: Quadrado da soma de trs termos

4 Caso: Cubo da Soma e Cubo da Diferena de Dois Termos

5 Caso: Cubo da Soma de Trs Termos

6 Caso: Produto de Stevin

7 Caso: Identidade de Lagrange (ax + by)2 + (ay bx)2 = (a2 + b2 )(x2 + y2 )

2.2 Tringulo de Pascal


A disposio de nmeros abaixo, formada somente pelos coeficientes dos termos do desenvolvimento de (x + y)n denominada Tringulo de Pascal (1623-1662). 10

RA

(x + y + z)2 = x2 + y2 + z2 + 2(xy + yz + zx)

(x y)3 = x3 3x2 y + 3xy2 y3

(x + y + z)3 = x3 + y3 + z3 + 3(x + y)(y + z)(z + x)

(x + a)(x + b) = x2 + x(a + b) + ab

(x + a)(x + b)(x + c) = x3 + (a + b + c)x2 + (ab + bc + ca)x + abc

FT

OBMEP PECI Preparao Especial para Competies Internacionais . 1


.

11

Linha 0: (x + y)0 Linha 1: (x + y)1 Linha 2: (x + y)2 Linha 3: (x + y)3 Linha 4: (x + y)4 Linha 5: (x + y)5 Linha 6: (x + y)6 Linha 7: (x + y)
7

1
.

1
.

1
.

2
.

1
.

1
.

3
.

3
.

1
.

1
.

4
.

6
.

4
.

1
.

1
.

5
.

10
.

10
.

5
.

1
.

1
.

6
.

15
.

20
.

15
.

6
.

1
.

1
.

7
.

21
.

35
.

35
.

21
.

7
.

1
.

Linha 8: (x + y)8

1
.

8
.

28
.

56
.

70
.

56
.

28
.

8
.

1
.

Nela, observamos que: (1) O primeiro elemento 1; (2) O nmero de termos da linha n n + 1; (3) Todas as linhas comeam e terminam por 1;

(4) Dois elementos de uma mesma linha, equidistantes dos extremos, so iguais; (5) Cada elemento de uma linha, a partir do segundo, a soma do elemento que lhe fica acima com o que est a esquerda deste ltimo;

(6) A soma dos coeficientes da linha de ordem n igual a 2n ;

(7) No desenvolvimento de (x + y)n , os expoentes de x decrescem de n at 0, enquanto os de y crescem de 0 at n.

Exemplos
1.

2.2.1

2.

(2a2 + 1)4 = 1(2a2 )4 + 4(2a2 )3 1 + 6(2a2 )2 12 + 4(2a2 )1 13 + 1 14 = 16a8 + 32a6 + 24a4 + 8a2 + 1
3. No desenvolvimento de (x y)n , os sinais (+) e (), alternam-se a partir do 1 termo.

2.3 Fatorao

1 Caso: Fator Comum em Evidncia ax + ay = a(x + y)

RA

(x + y)4 = 1x4 + 4x3 y + 6x2 y2 + 4xy3 + 1y4

(x y)5 = 1 x5 5x4 y + 10x3 y2 10x2 y3 + 5xy4 1 y5

FT

12

lgebra Produtos Notveis e Fatorao Professor Robrio Bacelar

2 Caso: Trinmio Quadrado Perfeito a2 2ab + b2 = (a b)2 3 Caso: Soma de dois cubos a3 + b3 = (a + b)(a2 ab + b2 ) 4 Caso: Diferena de dois cubos a3 b3 = (a b)(a2 + ab + b2 ) 5 Caso: xn yn , sendo n um nmero natural xn yn = (x y)(xn1 + xn2 y + xn3 y2 + + xyn2 + yn1 )
Exemplo:

x5 y5 = (x y)(x4 + x3 y1 + x2 y2 + x1 y3 + y4 )

xn + yn = (x + y)(xn1 xn2 y + xn3 y2 xyn2 + yn1 )


Exemplo:

x5 + y5 = (x + y)(x4 x3 y1 + x2 y2 x1 y3 + y4 )

Potnciao em R 2.4.1

Potncia com Expoente Natural

Se n um inteiro positivo qualquer e x um nmero real, definimos

2.4.2

Propriedades

P1: xm xn = xm+n P2:

xm = xmn , x = 0 xn

P3: (x y)n = xn yn

( )m x xm P4: = m , y = 0 y y
P5: (xm )n = xmn

( )m ( ) x y m = P6: y x

RA
xn = 1, se n = 0
n vezes

2.4

x x . . . x, se n > 1

FT

6 Caso: xn + yn , quando n mpar

OBMEP PECI Preparao Especial para Competies Internacionais

13

2.5 Radiciao em R 2.5.1 Propriedades

a = b a = bn , onde

radical n ndice

R1: R2: R3: R4:

R5:

2.6 Potncia com Expoente Racional

2.7 Radical Duplo

2.8

Problemas Propostos Problema 1

Problema 2

D
(OCM) Se x2 + x + 1 = 0, calcule o valor numrico de

(ESPANHA-ADAPTADA) Sejam a, b, c nmeros reais no nulos com a + b + c = 0 tais que

Mostre que tambm se verifica a igualdade

RA
xn =
m

m n xm ; Q, n N, n 2. n

ab=

a+c 2

( )2 ( )2 ( )2 ( )2 1 1 1 1 x+ + x2 + 2 + x3 + 3 + + x27 + 27 . x x x x

1 1 1 1 + + = . a b c a+b+c

1 a2013

1 b2013

1 c2013

FT
ac , onde c = a2 b 2 = 1 a2013 + b2013 + c2013 .

a radicando b raiz Se n um nmero natural, n 2, x, y so reais tais que n x e n y existem, ento: ( n x)n = x n x n y = n xy n x x = n , y = 0. n y y m n x = mn x, m N, m 2. x, se n mpar n xn = |x|, se n par

14

lgebra Produtos Notveis e Fatorao Professor Robrio Bacelar

Problema 3

Prove que se a + b + c = 0, ento:

(a)a3 + b3 + c3 = 3abc (b) (c) a3 + b3 + c3 a2 + b2 + c2 a5 + b5 + c5 = 5 3 2 a7 + b7 + c7 a5 + b5 + c5 a2 + b2 + c2 = 7 5 2


(Torneio das Cidades) Sejam a,b,c e d nmeros reais tais que

Problema 4

a3 + b3 + c3 + d3 = a + b + c + d = 0.
Prove que a soma de dois desses nmeros zero.

Problema 5

(COLMBIA) Sejam a,b,c reais tais que

a12 + b12 + c12 = 8


e

(a b)2 + (b c)2 + (c a)2 6 = . abc a+b+c

Calcule a6 + b6 + c6 .

Problema 6

(China) Seja um nmero real tal que 3 1 = 0. Determine o valor numrico de

Problema 7

Sejam x,y,z nmeros reais dois a dois distintos. Prove que a expresso

Problema 8

Problema 9

no depende de x, y e z.

Sejam a,b,c nmeros reais distintos. Prove que

(OMG) Seja a = 2008. Determine o valor da soma


k=2007 k=2007

Problema 10

Os dois menores lados de um tringulo retngulo, a e b, satisfazem desigualdade

Encontre o permetro desse tringulo.

RA
3 32 4 + 1 . 1 + ak

x(y + z) y(x + z) z(x + y) + + (x y)(x z) (y z)(y x) (z x)(z y)

a2 b2 c2 + + = 1. (a b)(a c) (b c)(b a) (c a)(c b)

2 6a 2 + 19 + a b2 4b 3 + 16 3.

FT
4 22 + 3 + 2.

OBMEP PECI Preparao Especial para Competies Internacionais

15

Problema 11

(Canad) Calcule:

1 5 1 5 (a) + 6 18 6 18 2 2 2 2 2 2 (b) 1 + 1 + 1 + 1 + 1 + 1+ . 5 6 7 8 57 58
Problema 12
(Romnia) Sejam x e y nmeros naturais tas que

( 1 1 1 ) ( 1 1 1 )2 ( 1 1 1 )3 ( 1 )59 1 1 1 2x .3y = 24 2 + 3 + 4 ++ 60 . 24 3 + 4 + 5 ++ 60 . 24 4 + 5 + 6 ++ 60 24 60 .
Determinar o valor de x + y.

Problema 13

(Rssia) Sejam a e b nmeros reais no nulos que satisfazem equao a2 b2 (a2 b2 + 4) = 2(a6 + b6 ).

Mostre que a e b no podem ser ambos racionais.

Problema 14 Problema 15

(REOIM) Demonstrar que (REOIM) Dados os nmeros

1 5 + > . 6 2+ 2 6+ 6

=
e

13 +

= 5 + 2 3 + 18 2 3 + 2 65 26 3,

mostrar que = .

Problema 16

(PERU) A expresso

a+b c pode ser escritana forma , onde a, b, c e d so os menores inteiros positivos possveis. Determine o valor de d a + b + c + d.
Problema 17 Problema 18
(OCM) Determine qual o maior dos dois nmeros

RA
2207 1
8

2207

(CANAD) Se os nmers reais positivos a1 , a2 , ...an so so comprimentos dos lados de um polgono

inscrito em uma circunferncia, tais que:

a2 + a2 + a2 + ... + a2 = a1 a2 + a2 a3 + a3 a4 + ... + an a1 , 1 2 3 n
pode-se afirmar que o polgono regular? Justifique sua resposta.

Problema 19

(OBM) As representaes decimais dos nmeros 21999 e 51999 so escritas lado a lado. Qual o nmero

total de algarismos escritos?

Problema 20

(PERU) Determine todos o sinteiros positivos x tais que a expresso

92 +

92 1 1 + 2 2 11 100 1100 x+1

inteiro.

FT
10 + 2 13 1 2207 1 2207 123456 + 10999 123457 + 10999 e . 123457 + 10999 123458 + 10999

16

lgebra Produtos Notveis e Fatorao Professor Robrio Bacelar (AIME) Sejam x e y nmeros reais, com x = y, e

Problema 21

x3 = 13x + 3y y3 = 13y + 3x
Determine (x2 y2 )2 .

Problema 22

(RIOPLATENSE) Ache o valor da soma

1 1 1+ 2 + 2 + 1 2

1 1 1+ 2 + 2 + 2 3

1 1 1 + 2 + 2 + ... + 3 4

1+

1 1 + . 20052 20062

Problema 23

(CANAD) Ache os nmeros x, y R+ tais que

x + 2y = 9 x 4y = 9 1 1 1 + + = 1, prove que x y z 3 a+ 3 b+

Problema 24

(Crocia) Se ax3 = by3 = cz3 e

ax2 + by2 + cz2 =

Problema 26

(Alemanha) Sejam x,y,a e b reais tais que

Prove que

Problema 27

D
x8 y8 1 + 3 = . 3 a b (a + b)3

(Canad) Determine todos os reais positivos x,y,z tais que

Problema 28

(Stanford) Ache os nmeros reais x e y tais que

RA
x2 + y2 = 1 4 4 x + y = 1 a b a+b 2 4x =y 2 4x + 1 2 4y =z 4y2 + 1 4z2 =x 4z2 + 1

Problema 25

Sejam x,y,z reas no nulos com x + y + z = 0. Prove que

x2 + y2 y2 + z2 x2 + z2 x3 y3 z3 + + = + + . x+y y+z x+z yz xz xy

4 x + 2x3 y = 1 + 3 4 y4 + 2y3 x = 1 3 4

FT
3 c.

OBMEP PECI Preparao Especial para Competies Internacionais

17

Problema 29

Qual a soma dos algarismos do natural n, no nulo, de modo que

25 + 2
tambm um inteiro?

625 n+ 4 25 2

625 n 4

Problema 30

(EUA) Determine o valor da expresso

1 + 2 1 + 3 1 + 4 1 .

Problema 31 Problema 32

(Sophie Germain) Fatore x4 + 4y4 . Desenvolva (a + b + c + d)3 .

RA

FT

lgebra

Equaes do Segundo Grau

3.1 Equao do Segundo Grau


Uma equao em x dita do Segundo Grau, quando pode ser escrita na forma ax2 + bx + c = 0, onde a, b e c so nmeros reais e a = 0.

3.1.1

3.1.2 Frmula de Bscara

Resolver uma equao do Segundo Grau significa determinar, atravs de processos algbricos, os valores de x que verifiquem a igualdade correspondente equao. A partir dos coeficientes a, b e c da equao algbrica ax2 +bx+c =

0 (a = 0) possvel demonstrar a existncia de uma relao entre as razes e esses coeficientes. Veja: ax2 + bx + c = 0 (2ax + b)2 = b2 4ac
extraindo-se a raiz

multiplicando-se por 4a
2 2

RA
b x= 2a
18

D
2 2

4a x + 4abx + 4ac = 0 subtraindo-se por 4ac 4a2 x2 + 4abx = 4ac somando-se b2

4a x + 4abx + b2 = b2 4ac
fatorando-se o trinmio

A expresso b2 4ac chama-se discriminante e indicada pela letra grega (delta). Logo:

Conforme o valor do discriminante, tm-se as seguintes possibilidades quanto natureza das razes:

> 0 : a equao admite duas razes reais e distintas; = 0 : a equao admite duas razes reais e iguais; < 0 : a equao admite razes complexas ou imaginrias (razes no-reais).

FT
x= b 2a

Forma Cannica do Trinmio de Segundo Grau ( ) ( ) b c b b2 b2 c 2 2 2 ax + bx + c = a x + x + =a x + x+ 2 2 + = a a a 4a 4a a [( ] [( ) ] )2 b b2 b2 4ac b b2 4ac a x2 + x + 2 2 + 2 = a x + a 4a 4a 4a 2a 4a2

2ax + b = b2 4ac subtraindo-se b 2ax = b b2 4ac dividindo-se por 2a


b2 4ac

OBMEP PECI Preparao Especial para Competies Internacionais

19

3.1.3 Soma das razes da Equao do Segundo Grau

b + b b x1 + x2 = + = 2a 2a a

3.1.4 Produto das Razes da Equao de Segundo Grau

( x1 x2 =

)( ) c b + b = 2a 2a a

3.2 Problemas Resolvidos

Exemplo 1
2

(OBM Jnior 1996) a, b, c e d so nmeros reais distintos tais que a e b so razes da equao

x 3cx 8d = 0 e c e d so razes da equao x2 3ax 8b = 0. Calcule a soma a + b + c + d.


Soluo:
obtemos fcil perceber que a+b = 3c e c+d = 3a. Somando e subtraindo membro a membro as duas igualdades,

Como a raiz de x2 3cx 8d = 0, segue que

RA

{ b + d = 2(a + c) . b d = 4(c a)

a2 3ac 8d = 0

FT

(3.1)

Do mesmo modo, como c raiz de x2 3ax 8b = 0, temos que

c2 3ac 8b = 0

(3.2)

Subtraindo as igualdades (1) e (2) e utilizando as relaes anteriormente obtidas, vem:

Como a c = 0, conclumos que a + c = 32. Portanto, a + c = 32 e b + d = 2(a + c) = 64, donde a + b + c + d = 96.

a2 c2 = 8(d b) (a c)(a + c) = 8 4(a c).

Exemplo 2

(Romnia) Sejam a, b, c nmeros reais no-nulos tais que a e 4a + 3b + 2c tm o mesmo sinal. Mostre

que a equao ax2 + bx + c = 0 no pode ter duas razes no intervalo (1, 2).

3b 2c 4a + 3b + c = 4+ + = 2x1 x2 3 (x1 + x2 )+4 = (x1 1) (x2 2)+(x1 2) (x2 1). a a a Se x1 e x2 pertencem ao intervalo (1, 2), ento cada termo da soma acima ser estritamente negativo, o que uma contradio.
Soluo:
Temos que 0

3.3

20

lgebra Equaes do Segundo Grau Professor Robrio Bacelar

Forma Fatorada do Trinmio de Segundo Grau


Se a equao ax2 + bx + c = 0, com a = 0, possui razes x1 e x2 , ento verdadeira a igualdade:

ax2 + bx + c = a(x x1 )(x x2 ).


Prova:

b a(x x1 )(x x2 ) = a[x (x1 + x2 )x + x1 x2 ] = a x a


2 2

] c x+ = ax2 + bx + c. a

3.3.1 Exerccios Propostos Problema 1


(a) x2 5x = 0 (b) x2 64 = 0 (c) x2 + 5x + 6 = 0 (d) x4 + 10x2 56 = 0 (e) x6 7x3 8 = 0 (f) x2 2mx + m2 n2 = 0 Resolva as seguintes equaes:

(g)

x+

1 x

(h) (x2 x + 3) 10(x2 x) = 105

Problema 2 Problema 3 Problema 4 Problema 5 Problema 6 Problema 7

Determine m para que a equao 2x2 8x + m = 0 admita razes iguais. Determine m para que a equao 3x2 + 6x + m = 0 admita razes reais distintas. Determine m para que a equao x2 8x + m = 0 admita razes imaginrias. Determine p para que a equao x2 + p2 x + 2px 4 = 0 admita razes simtricas. Determine m na equao x2 6x + 2m = 0 de modo que uma de suas razes seja ao dobro da outra.

do segundo graus que admite a e b como razes.

Problema 8
das razes 40.

Problema 9 Problema 10

sejam recprocas? E para que sejam opostas (simtricas)?

Problema 11 5 . a 12 Problema 12

do segundo graus cujas razes so a + 1 e b + 1.

Se a mdia aritmtica de dois nmeros a e b 7 e a mdia geomtrica entre eles 9, escreva a equao

Achar a equao do segundo grau na qual uma das razes o triplo da outra e a soma dos quadrados

Determine m na equao 3x2 2x + 5m = 0 de modo que a difere a de suas razes seja 1. Qual a relao que deve existir entre os coeficientes da equao ax2 + bx + c = 0 para que suas razes

Determine k na equao x2 + kx + 36 = 0 de modo que a soma dos inversos de suas razes seja igual

Considere a equao x2 mx + 1 = 0 cujas razes so a e b, reais e distintas. Componha a equao

RA

)2

( ) 1 5 x+ +6=0 x

FT

OBMEP PECI Preparao Especial para Competies Internacionais

21

Problema 13 Problema 14 Problema 15 Problema 16

Sejam a e b razes da equao 2x2 3x + k = 0. Determine k de modo que a3 + b3 = (OBM) Determine todos os reais x tais que x2 + x + 1 =

243 . 8

156 . x2 + x

(OBM) Se raiz da equao x2 + x 1 = 0, determine o valor de 5 5. Seja uma das razes da equao x2 3x + 1 = 0. Determine os possveis valores de 21 4 .

Problema 17 Provar que a condio para que uma raiz da equao ax2 + bx + c = 0 seja n vezes a outra (n + 1)2 b2 = ac. n Problema 18 Problema 19
a) b) c) d) e) f) Forme a equao biquadrada que admite 2 e 3 como razes. Resolva as equaes abaixo no conjunto dos nmeros reais:

Problema 20

Ache o valor de x na equao x =

Problema 21 Problema 22 Problema 23 Problema 24

Resolva, para x R a equao

x2 = 3. (x + 1)2 ( ) 1 1 Resolva, para x R a equao x2 + 2 4 x + + 5 = 0. x x


Resolva, para x R a equao x2 + (ESPECEX) Determine todas as razes reais de 2
x2 +
1 x2

RA
1+ 1 1+ 1
..

1+

(2 + 3)x + (2 3)x = 4.

Problema 25

Mostre que, para todos a,b,c R, sendo a = 0, a equao

razes reais e distintas.

Problema 26 Problema 27

(OCM) Encontre todas as razes da equao

(OCM) Determine o valor de p, para que as razes x1 e x2 da equao 2x2 px 1 = 0 satisfaam

x2 + x2 = 1. 1 2
Problema 28

FT
1 1
. .

2x 3 = 5 x2 + 5x + 1 + 1 = 2x x + 25 x2 = 1 16 + x + 4 = 5 x 2 + x 7 = x + 5 + x 10 3 2x2 + 3x 1 = 2x 1

1024 2x+ x
1

1 1 1 + = 2 sempre possui duas xb xc a x2 + 4x + 2 = 2. x2 2x + 2

x2 2x + 2 + x2 + 4x + 2

tenha razes reais x1 e x2 . Se x1 = x2 (bx2 ), prove que b < 0.


Problema 29

(OCM) Seja b um nmero real no nulo de modo que a equao do segundo grau x2 + b2 x +

=0

x2 48 (OCM) Determine o nmero de razes da equao + 2 = 10 3 x


Prove que, para todo x real, temos 2x + 3x 4x + 6x 9x 1.

) x 4 . 3 x

Problema 30 Problema 31

(ITA) Sejam x1 e x2 razes distintas de x2 + px + 8 = 0, p R fixo. Prove que |x1 + x2 | > 4 2.

22

lgebra Equaes do Segundo Grau Professor Robrio Bacelar (ITA) Ache a soma das razes positivas da equao 4x 5.2x + 4 = 0. (IME) Sejam x1 e x2 as razes da equao x2 + (m + 15)x + m = 0. Sabendo que x1 e x2 so naturais,
2 2

Problema 32 Problema 33

ache os possveis valores de m.

Problema 34 Problema 35 Problema 36

(IME) Ache as solues de

5 5 x = x. x + x + 2 + x2 + 2x = 3.

Ache as solues reais da equao x +

(MOLDVIA) Seja x um real. Determine todas as solues reais de

(x + 127)6 8 7 (x 127)6 = 7 7 (x2 1272 )3 .

Problema 37 Problema 38

(MOLDVIA) Ache as solues reais de 4( 1 + x 1)( 1 x + 1) = x. (Teste Brasil/Cone Sul) Sejam a,b,c nmeros reais tais que as equaes x2 +ax+1 = 0 e x2 +bx+c = 0

tm exatamente uma raiz em comum e as equaes x2 + x + a = 0 e x2 + cx + b = 0 tambm tm uma raiz real em

Problema 39

Sejam e as razes da equao x2 + px + q = 0 e e as razes da equao x2 + Px + Q = 0.

Expresse o produto ( )( )( )( ) em funo dos coeficientes das equaes dadas.

Problema 40

Considere as equaes x2 + ax + 1 = 0 e x2 + x + a = 0. Determine todos os valores de a para que

as duas equaes admitam pelo menos uma raiz comum.

Problema 41

Resolva a equao (x + 3)(x + 2)(x + 1)x = 48.

RA

FT

comum. Determine a soma a + b + c.

RA
PARTE II

Combinatria
Professor Fbio Brochero

Assuntos do Mdulo: Paridade, Princpio da Casa dos Pombos, Grafos.

Pr-requisitos: Contagem Simples. Princpio Multiplicativo. ( ) Um conjunto com n elementos possui n = n(n 1)/2 subcon2
juntos com 2 elementos (captulos 1 e 4 da apostila 2 do PIC).

FT
.

Combinatria

Paridade

Nesta seo o nico conhecimento prvio de que precisamos a diferena entre nmero par e mpar. Em particular, usaremos frequentemente o fato de que uma soma de dois nmeros mpar se e somente se um deles mpar e o outro par. Para comear, pensemos no seguinte problema.

Exemplo 1

Os nmeros 1, 2, 3, . . . , 2013 so escritos em um quadro-negro. permitido apagar do quadro quaisquer

dois nmeros e substitu-los por sua diferena positiva. Aps fazer essa operao vrias vezes, sobrar somente um

Soluo:

Vejamos que no possvel mostrando que o ltimo nmero que aparece no quadro mpar. De fato,

cada vez que fazemos o processo temos trs possibilidades:

Trocamos dois pares por um par, Trocamos dois mpares por um par, Trocamos um par e um mpar por um mpar

Nos trs casos a quantidade de nmeros mpares continuou igual ou diminuiu em 2, portanto conservamos a paridade da quantidade de nmeros mpares. Assim como na lista 1, 3, 5, . . . , 2013 existem 1007 nmeros mpares, ao final obtemos um nmero mpar.

RA
24

FT

nmero no quadro. Este nmero pode ser igual a 10?

Soluo: Observe que o tabuleiro tem um nmero mpar de quadradinhos, mas cada pea cobre um nmero par de
quadradinhos. Portanto no possvel cobrir todo o tabuleiro.

Exemplo 2

possvel cobrir um tabuleiro 2009 2009 com peas de tamanho 1 2?

Agora modifiquemos um pouco o problema anterior.

Exemplo 3

possvel cobrir com peas de tamanho 1 2 um tabuleiro 2010 2010 do qual retiramos a casa do

canto superior direita e a casa do canto inferior esquerdo?

Soluo:

Observemos que o argumento anterior j no funciona, porque agora temos que cobrir 20102 2 quadri-

nhos, que par, com peas de tamanho 1 2. Porm, podemos pensar da seguinte forma: vamos pintar o tabuleiro como um tabuleiro de xadrez, isto , alternadamente de branco e preto. Assim, se dois quadrinhos compartilham um lado, ento eles tm cores diferente. Deste modo, cada pea que colocamos sobre o tabuleiro vai cobrir um quadrinho preto e outro branco. Ento, a quantidade de quadradinhos brancos e pretos cobertos a mesma. Agora, os cantos superior direito e inferior esquerdo tm a mesma cor porque esto sobre a mesma diagonal. Logo tiramos dois quadrinhos da mesma cor e no tabuleiro restante ficaram mais casas de uma cor que de outra e, portanto, o restante do tabuleiro no pode ser coberto pelas peas.

OBMEP PECI Preparao Especial para Competies Internacionais

25

Exemplo 4

Considere a expresso

1 2 3 4 2009.
possvel escolher os sinais + ou de tal forma que o resultado final seja 1000?

Soluo: Novamente a resposta no, por paridade. Como a quantidade de nmero mpares nesta soma mpar,
ento para qualquer escolha dos sinais o resultado final ser sempre mpar.

Exemplo 5

Uma linha poligonal fechada composta por 7 segmentos. possvel que uma reta intersecte o interior

de cada um dos 7 segmentos?

Soluo:

Suponha que a reta intersecte os segmentos P1 P2 , P2 P3 , . . . , P6 P7 e P7 P1 . A reta divide o plano em dois

semiplanos, os quais chamaremos de e .

P1
.

P4

P2

P3 P5

P7

Suponha que o ponto P1 seja um ponto do semiplano . Ento P2 est em , P3 em e assim sucessivamente. Deste modo P7 est em e como o segmento P1 P7 cortado por , P1 est em , absurdo.

FT
P6 6 5 4 3 1 2

Exemplo 6

possvel que um cavalo faa um percurso passando por todas as casas de um tabuleiro de xadrez 8 8,

comeando no canto superior esquerdo e terminando no canto inferior direito?

Soluo:

casa onde sai. Se queremos que o cavalo percorra todas as casas do tabuleiro uma nica vez, teremos que mov-lo 63 vezes. Portanto, teremos um nmero mpar de trocas de cor, e a cor da primeira casa e a da ltima sero diferentes. Desse modo, impossvel fazer tal passeio, porque a casa do canto superior esquerdo e a casa do canto inferior direito possuem a mesma cor.

D
.

Observe que ao movimentar uma vez o cavalo no tabuleiro, a cor da casa onde chega diferente da cor da

RA
7 8

NZ0Z0Z0Z Z0Z0Z0Z0 0Z0Z0Z0Z Z0Z0Z0Z0 0Z0Z0Z0Z Z0Z0Z0Z0 0Z0Z0Z0Z Z0Z0Z0Z0

26

Combinatria Paridade Professor Fbio Brochero

Exemplo 7

(Seleo para o PECI 2013) Colocamos 15 cavalos em um tabuleiro 15 15 de modo que no existam

dois cavalos em uma mesma linha ou em uma mesma coluna. Cada cavalo faz ento um movimento. Prove que agora existem dois cavalos na mesma linha ou na mesma coluna.
Observao: O cavalo movimenta-se em L. Assim, um cavalo na casa marcada com pode mover-se para as casas destacadas.

Soluo:

. Adaptada da Soluo de Thiago Lucas Faustino da Silva de Itumbiara Gois.

dois cavalos troquem de posio no movimento, se fizermos isto, sobra um cavalo sem movimentar-se. Uma maneira de provar a seguinte, vamos numerar cada cavalo pela posio, se est na linha 1 coluna 2, como

1 + 2 = 3, o nmero dele 3. A soma de todos os nmeros ser

2(1 + 2 + + 15) = 450.

Se no houver cavalos diferentes na mesma linha ou coluna, a cada movimento, o nmero de um cavalo se altera em: So nmeros mpares, para que continue com apenas um cavalo em cada linha e em cada coluna, a soma dos nmeros dos cavalos deveria ser 450 o que implicaria uma alterao nula nesta soma, ou seja, 0. A alterao total ser a soma das alteraes individuais de cada cavalo. Porm, 0 par e a soma de 15 nmeros mpares no pode ser par, por isso, impossvel que continue um cavalo em cada linha e um em cada coluna. Logo, existem dois cavalos na mesma linha ou na mesma coluna.

RA
. . Sorteio .. . P

3, 1, 1 ou 3.

FT
B

Um dos motivos que 15 mpar, se o nmero de cavalos fosse par, poderamos colocar os cavalos de modo que

Exemplo 8

Em uma urna temos 2013 bolas brancas e 2013 bolas pretas e fora dela temos uma quantidade grande

cor, devolvemos uma bola preta, mas se as bolas tiradas so de cores distintas, devolvemos uma branca. No final fica uma nica bola na urna. Esta bola que fica na urna pode ser preta?

de bolas brancas e pretas. A cada jogada, tiramos duas bolas da urna sem olhar: Se as duas bolas tiradas so da mesma

1 +1 1

0 2 0

. . .

Soluo:

Temos trs situaes possveis quando retiramos duas bolas da urna:

(1) As duas bolas so pretas : Neste caso devolvemos urna uma bola preta. Assim, o nmero de bolas pretas diminui em um e o nmero de bolas brancas permanece igual; (2) As duas bolas so brancas: Neste caso devolvemos urna uma bola preta. Assim, o nmero de bolas pretas aumenta em um e o nmero de bolas brancas diminui em dois; (3) As duas bolas so de cores distintas: Neste caso devolvemos urna uma bola branca. Assim, o nmero de bolas pretas diminui em um e o nmero de bolas brancas permanece igual.

OBMEP PECI Preparao Especial para Competies Internacionais

27

Observemos que nos trs casos, o nmero de bolas brancas permanece igual ou diminui em dois, assim a paridade do nmero de bolas brancas na urna no muda em cada jogada. Portanto, o nmero de bolas brancas na urna nunca pode ser zero, pois comeamos com um nmero mpar de bolas brancas na urna. Logo, em qualquer caso a ltima bola na urna branca.

Exemplo 9
versa.

Em um tabuleiro 8 8, com casas pintadas de branco ou preto, dizemos que uma linha ou coluna

repintada quando trocamos as cores de todas as casas dessa linha ou coluna, isto , uma casa branca vira preta e vice Os quatro cantos de um tabuleiro totalmente branco so pintados de preto. Mostrar que impossvel, repintando linhas e colunas, deixar todas casas do tabuleiro da mesma cor.

Soluo: Suponhamos que queremos pintar todas as casas do tabuleiro de preto, j que ao obter um tabuleiro
totalmente preto fcil obter um tabuleiro totalmente branco, bastando repintar todas as linhas do tabuleiro e viceversa. Consideremos as quatro casas do canto inferior esquerda do tabuleiro 8 8 que forma um subtabuleiro 2 2. No comeo, este subtabuleiro tem uma casa preta e trs brancas, como ilustrado na figura:

d c
.

Denotamos por a e b o nmero de vezes que a 1a e a 2a colunas foram repintadas, e denotamos por c e d o nmero de vezes que a 7a e a 8a linhas foram repintadas. Como a cada duas vezes que uma casa repintada ela volta cor original, se quisermos que todo o tabuleiro fique preto, precisamos em particular que

a + c seja par; a + d seja mpar; b + c seja mpar; b + d seja mpar.

Como (a + c) + (a + d) + (b + c) + (b + d) = 2(a + b + c + d) par, as condies anteriores no podem ocorrer

RA

FT
a b

simultaneamente e, portanto, impossvel repintar o tabuleiro de modo que fique todo da mesma cor.

Exemplo 10

2013 cartas que tem uma face azul e a outra vermelha so colocadas em uma mesa de forma que

algumas ficam com a face vermelha para cima e as outras com a face azul. Dois jogadores podem realizar cada um em sua vez, uma das seguintes operaes:

retirar da mesa qualquer quantidade de cartas com a mesma cor na face de cima; virar qualquer quantidade de cartas com a mesma cor na face de cima.
Ganha quem pegar a ltima carta. Qual dos dois jogadores tem uma estratgia vencedora?

Soluo:
vencedora:

Como o nmero inicial de cartas mpar, temos que inicialmente existe uma quantidade par de cartas de

uma cor viradas para acima e um nmero mpar da outra cor. Vejamos como o primeiro jogador tem uma estratgia

Em seu turno, o primeiro jogador remove cartas de forma a ter a mesma quantidade de cartas azuis e vermelhas. Ele sempre pode fazer isto, pois inicialmente h mais cartas de uma cor do que de outra e quando o segundo jogador

28

Combinatria Paridade Professor Fbio Brochero

recebe uma quantidade igual de cartas azuis e vermelhas, qualquer operao que ele faa vai diminuir a quantidade de cartas de uma cor e manter ou aumentar a quantidade de cartas da outra cor, permitindo ao primeiro jogador repetir o procedimento. Como o primeiro jogador sempre est removendo cartas, vai chegar um momento em que o segundo jogador no ter outras opes a no ser acabar com as cartas de uma cor, permitindo ao primeiro jogador acabar com as cartas da outra cor.

Exemplo 11

Escrevemos +1 ou 1 em cada casa de um tabuleiro 5 5. Depois calculamos o produto dos nmeros

em cada coluna e em cada linha obtendo no total dez nmeros. possvel que a soma destes dez nmeros seja igual a

0?
Soluo:
O problema equivalente a comear por um tabuleiro preenchido com 25 uns e a pergunta se possvel ir

mudando os sinais de algumas destas entradas de forma que a soma dos produtos seja 0. Suponhamos que mudamos uma das casas de sinal. Logo, o sinal do produto da linha e a coluna qual pertence tambm muda. Assim temos trs possveis casos

se os produtos eram +1, depois da mudana os dois produtos ficaram 1; se os produtos eram 1, depois da mudana os dois produtos ficaram +1;

se um dos produtos era +1 e o outro 1, depois da mudana um dos produtos fica 1 e o outro +1.
Assim, no primeiro caso a soma diminui em 4, no segundo aumenta em 4 e no terceiro a soma no muda. Dado que a soma inicial era 10, os possveis valores que podem ser obtidos so 10, 6, 2, 2, 6 e 10. Deste modo, a soma nunca pode ser 0.

FT

Exemplo 12

Um aparelho formado por 64 lmpadas montadas em um tabuleiro 8 8 com 16 interruptores, onde

cada interruptor muda o estado das lmpadas de uma linha ou coluna do aparelho. Supondo que no comeo todas as lmpadas esto desligadas, possvel acionar os interruptores de tal forma que ao final fique exatamente uma lmpada acesa? E duas lmpadas acesas?

Soluo:

Podemos transformar o problema no seguinte: Colocamos em cada casa de um tabuleiro 8 8 o nmero

vemos que cada vez que mudamos uma linha ou coluna, mudamos os nmeros de oito casas, assim se em uma linha ou coluna arbitrria temos a uns e b, zeros, depois da mudana vamos ter a zeros e b uns nessa linha ou coluna. Como a + b = 8, temos que a e b tm a mesma paridade. Assim, a paridade do nmero de uns e zeros no tabuleiro no muda depois de tal troca. Como inicialmente temos 64 uns, ao final impossvel obter exatamente um nmero 1 e isto resolve a primeira parte do problema. Seguindo a mesma ideia, suponhamos que em algum momento temos A zeros e B uns no tabuleiro, ao mudar uma linha ou coluna obtemos A + b a zeros e B + a b uns. Assim inicialmente a diferena entre o nmero de zeros e de uns era B A e depois da mudana B A + 2(a b). De igual forma, como a + b = 8 temos que a b par, e portanto 2(a b) mltiplo de 4. Portanto, como inicialmente B A 64, depois de algumas mudanas esta diferena entre o nmeros de uns e de zeros tem que ser divisvel por 4, portanto no pode ser 2. Observe que este argumento tambm resolve a primeira parte do problema.

0 ou 1 e em cada passo, podemos mudar todos os nmeros de uma linha ou de uma coluna. Se comeamos com o tabuleiro preenchido com 64 uns, possvel obter, depois deste tipo de mudanas, exatamente um nmero 1. Obser-

RA

Exemplo 13

Num tabuleiro de damas 8 8 colocada uma pedra em cada casa das trs primeiras linhas. Uma

pedra se move pulando sobre outra pedra, ficando na casa simtrica a que estava em relao pedra pulada, desde que esta casa esteja vazia. possvel que aps uma sequncia de pulos, seja obtida uma configurao com as trs ltimas linhas do tabuleiro preenchidas pelas pedras?

OBMEP PECI Preparao Especial para Competies Internacionais

29

Soluo:

Se numerarmos as linhas do tabuleiro de 1 a 8, ento no comeo as linhas 1, 2 e 3 estaro totalmente

preenchidas. Observe que depois de cada movimento, os nmeros da linha inicial e final da pedra tm a mesma paridade (se uma pedra na linha a pula uma pedra na linha a + b, a pedra vai parar na linha a + 2b, com a mesma paridade da linha a). Como inicialmente tnhamos 16 pedras em linhas mpares e 8 em linhas pares, depois de qualquer nmero de movimentos vamos ter a mesma configurao. Deste modo, impossvel preencher as trs ltimas linhas do tabuleiro, j que neste caso teramos 8 pedras na linha 7 e 16 nas linhas 6 e 8 juntas.

Exemplo 14

Um baralho espanhol (40 cartas) distribudo em um arranjo de 5 linhas e 8 colunas com todas as

cartas viradas para baixo. Dois jogadores se alternam fazendo a seguinte jogada: o primeiro seleciona uma linha que no tenha todas as cartas viradas para cima, vira a primeira carta que esteja virada para baixo e depois vira e desvira as cartas seguintes a vontade. Perde o primeiro jogador que no consegue fazer uma jogada. Qual dos jogadores tem uma estratgia vencedora?

Soluo:

Observemos que o primeiro jogador sempre tem uma estratgia vencedora. Para isto, basta que em sua

primeira jogada, o primeiro jogador vire todas as cartas da primeira linha, de modo que as cartas que continuam em jogador. De fato, denote as linhas por a, A, b e B; se o segundo jogador joga nas linhas a ou b, o primeiro copia a jogada nas linhas A ou B e vice-versa. Assim, o primeiro jogador no ter jogadas apenas quando o segundo jogador no tiver.

FT

jogo estejam nas outras quatro linhas. Em seguida, basta para o primeiro jogador copiar as jogadas do segundo

Exemplo 15

Sobre uma reta r tome um segmento de extremos A e B. Fora do segmento AB, mas sobre a reta r,

existem 19 pontos distintos tais que alguns esto antes do ponto A e outros depois do ponto B, mas nenhum ponto est de todos os pontos que esto depois de B at o ponto B.

Prove que a soma das distncias de todos os pontos que esto antes de A at o ponto B no pode ser igual soma das distncias de todos os pontos que esto depois de B at o ponto A.

Soluo:

Chamemos de d a distncia entre A e B, x1 , . . . , xj as distncias dos pontos que esto antes de A ao ponto

A e y1 , . . . , yk as distncias dos pontos que esto depois de B ao ponto B. Pela hiptese do problema temos que j + k = 19;

x1 + + xj = y1 + + yk ;

A distncia dos pontos que esto depois de B ao ponto A y1 + d, . . . , yk + d; A distncia dos pontos que esto antes de A ao ponto B x1 + d, . . . , xj + d.
Segue que a soma das distncias dos pontos que esto depois de B ao ponto A

e a soma das distncias dos pontos que esto antes de A ao ponto B

Caso estas duas somas fossem iguais teramos que k = j =

RA

entre A e B. A soma das distncias de todos os pontos que esto antes de A at o ponto A igual soma das distncias

(y1 + d) + + (yk + d) = y1 + yk + kd = x1 + xj + kd

(x1 + d) + + (xj + d) = x1 + xj + jd, 19 , um absurdo. Portanto as somas no podem ser iguais. 2

30

Combinatria Paridade Professor Fbio Brochero (OBM) Pintamos de vermelho ou azul 100 pontos em uma reta.

Exemplo 16

Se dois pontos vizinhos so vermelhos, pintamos o segmento que os une de vermelho; Se pontos vizinhos so azuis, pintamos o segmento de azul; Finalmente, se dois pontos vizinhos tm cores distintas, pintamos o segmento de verde.
Feito isto, existem exatamente 20 segmentos verdes. O ponto mais esquerda vermelho. possvel determinar com estes dados a cor do ponto mais direita? Em caso afirmativo, qual a cor deste ponto?

Soluo:

Somente existem mudanas de cor entre pontos consecutivos quando o segmento que os une de cor

verde. Alm disso, se temos um nmero par de segmentos verdes entre dois pontos, vamos ter um nmero par de mudanas de cor entre eles, assim estes dois pontos tm a mesma cor e de igual forma, se o nmero de segmentos verdes entre dois pontos mpar, vamos ter um nmero mpar de mudanas de cor entre eles e assim estes dois pontos tm cores diferentes. No caso particular em que temos 20 segmentos verdes, conclumos que o ponto da ponta esquerda e o ponto da ponta direita tem a mesma cor, portanto o ponto da ponta direita vermelho.

Exemplo 17

Lema de Sperner. Um tringulo T dividido em tringulos menores de modo que quaisquer dois

tringulos pequenos ou no possuem pontos em comum, ou possuem um ponto em comum, ou possuem um lado completo em comum. Os trs vrtices de T so etiquetados com A, B, C. Todos os vrtices dos tringulos pequenos tambm so etiquetados com A, B, C, com a nica restrio de que vrtices sobre o lado AB no recebem a etiqueta C, vrtices sobre o lado AC no recebem a etiqueta B e vrtices sobre o lado BC no recebem a etiqueta A. Mostre que dentre os tringulos pequenos, existe no mnimo um cujos vrtices esto etiquetados com A, B e C.

RA
A A A C B A B A

D
B

Soluo:

Para resolver este problema, vamos atribuir valores a cada um dos lados dos tringulos menores da

seguinte forma: atribumos um 0 se os extremos do lado esto etiquetados com letras iguais, caso contrrio atribumos um 1. Desta forma, os lados de cada tringulo podem ter os seguintes valores:

0 0 0, se os trs vrtices tm a mesma etiqueta;

1 1 0, se dos vrtices tm uma mesma etiqueta e o terceiro tem uma etiqueta distinta;

1 1 1, se todos os vrtices tm etiquetas distintas.

FT
A

OBMEP PECI Preparao Especial para Competies Internacionais


A

31

0 0
A

1 0 1 1 1
C B A

0
A

0
A

1 1 0
A

1 . 0 1 1
B

1
B

1 1 1 0 0
B

1 1

1 1
B C

1
C

Agora podemos atribuir a cada tringulo a soma dos nmeros atribudos a seus lados. Assim, cada tringulo pode receber os nmeros 0, 2 ou 3. Observe que se um tringulo recebe o nmero 3 porque seus trs vrtices esto Para isto, seja S a soma dos nmeros atribudos aos tringulos. Se mostramos que S mpar, ento uma das parcelas mpar e, portanto, umas das parcelas 3. Para determinar a paridade de S, basta observar que cada lado interior ao tringulo maior somado duas vezes em S porque este lado pertence a dois tringulos distintos, enquanto todo lado que pertence ao bordo do tringulo maior contado apenas uma vez. Logo, a paridade de S igual paridade da soma dos lados que esto na borda do tringulo maior. Neste ponto, o problema parecido ao problema anterior: Num lado do tringulo do lado maior estamos atribuindo etiquetados por letras distintas e, neste caso, o problema estaria resolvido.

0 se no h troca de cor e 1 se h troca de cor. Como o ponto inicial e o ponto final so distintos, ento o nmero de
trocas de cor mpar. Assim, o nmero de uns em qualquer um dos lados do tringulo maior mpar e como a soma de trs nmeros mpares mpar, temos que S mpar, como queramos demonstrar.

RA
. .

FT
.

4.1

Problemas Propostos Problema 1

(OPM) A seguir, temos a parte de baixo de uma figura.

A figura delimita uma regio. O crculo preto est dentro ou fora dessa regio? Lembre-se: voc deve justificar sua resposta!

Problema 2 Das 28 peas de um domin, foram perdidas as 5 peas a seguir:

D
. . Bruno notou que com as peas restantes poderia fazer um arranjo de modo que todas as peas fossem enfileiradas, respeitando as regras do jogo de domin. Qual a soma dos nmeros das extremidades da fileira?

32

Combinatria Paridade Professor Fbio Brochero O produto de 50 inteiros 1. Mostrar que a soma dos inteiros no pode ser 0. Mostrar que a soma dos

Problema 3

inteiros no pode ser 4 nem 5.

Problema 4

Num tabuleiro de xadrez (88) um cavalo parte de uma casa do tabuleiro e faz sucessivos movimentos

at voltar casa inicial. Mostre que o cavalo realizou um nmero par de jogadas. Ateno! O cavalo faz movimentos em L conforme indicado na figura (duas casas em uma direo e uma em outra).

Problema 5

Um nmero de 17 algarismos escrito. Depois obtemos um nmero escrevendo os algarismos deste

de forma invertida. Mostre que a soma destes dois nmeros sempre contm um algarismo par.

Problema 6

10 meninos e 11 meninas ficam sentados ao redor de uma mesa circular. Mostre que existe uma pessoa

nesta mesa cujos vizinhos so meninos.

Problema 7 Prove que a igualdade

1 1 1 1 1 1 + + + + + =1 a b c d e f
no tem solues no conjunto dos naturais mpares.

Problema 8

Os nmeros 1, 0, 1, 0, 0, 0 esto dispostos ao redor de uma circunferncia e vamos mudando estes

nmeros seguindo a seguinte regra: pegamos dois nmeros adjacentes e somamos 1 a cada um deles. possvel depois de um nmero finito de passos obter 6 nmeros iguais ao redor da circunferncia?

Problema 9 (Cone Sul 1995) H dez pontos marcados sobre uma circunferncia, numerados de 1 a 10 em alguma ordem. Traamos em seguida todos os segmentos que esses pontos determinam e os pintamos, uns de vermelho e os demais de azul. possvel, sem trocar as cores dos segmentos, reenumerar os pontos de 1 a 10 de modo que se dois nmeros eram unidos por um segmento vermelho agora so unidos por um segmento azul e vice-versa?

Problema 10

possvel dividir um polgono de 13 lados em um nmero finito de paralelogramos?

RA

FT

Combintria

Princpio da Casa dos Pombos

O Princpio das Casas dos Pombos uma ferramenta elementar, mas muito poderosa para resolver problemas. A idia do princpio altamente intuitiva: Se temos mais pombos que casas, em alguma casa tem que ficar mais que um pombo.

. f

6 pombos em 5 casas: pelo menos 1 casa tem 2 pombos

De um modo mais geral,

Se temos nk + 1 ou mais pombos para colocar em n casas, alguma casa ficar k + 1 ou mais pombos

f
.

ff f

13 pombos em 6 casas: pelo menos 1 casa tem 3 pombos

Observemos que este princpio pode ser provado raciocionando por contradio: de fato, se em cada casa colocarmos

k ou menos pombos, ento colocaremos no mximo nk pombos. Como temos nk + 1 pombos, alguma casa ter pelo menos k + 1 pombos.
O Princpio da Casa dos Pombos tambm conhecido como Princpio de Dirichlet ou Princpio das Gavetas.

5.1

Exemplos Introdutrios

Exemplo 1

Demonstrar que entre os nmeros escolhidos sempre existem dois que so consecutivos.

Soluo:

D
.

Seja A = {1, 2, . . . , 99, 100} o conjunto dos nmeros de 1 a 100 e deste pegamos ao acaso 51 nmeros.

Para provar isto, primeiro escolhamos gavetas adequadas ao problema.

RA
g1 g2 g3
33

Suponhamos que construmos cinquenta gavetas g1 , g2 , . . . , g50 para colocar os 51 nmeros escolhidos. Na primeira iro o 1 e o 2, na segunda o 3 e o 4, e o geral, na i-sima iro o 2i 1 e o 2i. Agora, temos construdo 50 gavetas e cada nmero entre 1 e 100 temos associado alguma gaveta, onde, no importa como escolhamos os 51 nmeros sempre haver uma gaveta com dois nmeros e estes, graas a nossa construo, sero consecutivos.

FT
f f

... g50

Podemos generalizar este resultado considerando os nmeros {1, 2, . . . , 2n} e escolhendo dentre eles n+1 nmeros ao acaso.

Exemplo 2

Escolhemos n + 1 elementos distintos do conjunto {1, 2, 3, . . . , 3n}, onde n um inteiro positivo dado.

Prove que h dois deles x e y tais que xy + 1 ou 4xy + 1 seja um quadrado perfeito.

34

Combinatria Princpio da Casa dos Pombos Professor Fbio Brochero Consideremos os n conjuntos

Soluo:

{1, 2, 3}, {4, 5, 6}, . . . , {3n 2, 3n 1, 3n}.


Pelo Princpio das Casas dos Pombos, ao menos dois nmeros x e y dos n + 1 escolhidos esto em um mesmo destes

n conjuntos, donde |x y| = 1 ou 2.
Suponhamos, sem perda de generalidade, que x > y.

Se x y = 1, ento 4xy + 1 = 4(y + 1)y + 1 = (2y + 1)2 . Se x y = 2, ento xy + 1 = (y + 2)y + 1 = (y + 1)2 .


Em qualquer caso, xy + 1 ou 4xy + 1 ser um quadrado perfeito.

Exemplo 3

Seja A = {1, 2, . . . , 99, 100} o conjunto dos nmeros de 1 a 100 e deste escolhemos ao acaso 55 nmeros.

Demonstrar que entre os nmeros escolhidos sempre existem dois tais que sua diferena 9.

Soluo:

Do mesmo modo que no exemplo anterior o problema descobrir como formar as gavetas. Consideremos

as gavetas 0, 1, 2, . . . , 8, onde o nmero n colocado na gaveta i se e s se o resto ao dividir n por 9 i. Como escolhemos 55 = 96 + 1 nmeros, isto nos indica que existir uma gaveta j na qual tenham ficado 7 ou mais nmeros dos escolhidos. Mas cada gaveta pode conter no mximo 12 nmeros (por exemplo, a gaveta 1 pode conter quaisquer elementos do conjunto {1, 10, 19, 28, 37, 46, 55, 64, 73, 82, 91, 100}. Segue, pelo exemplo 1, que existiram dois nmeros que sero consecutivos em tal conjunto e portanto sua diferena 9.

FT
2/2.

Mostre que em qualquer poliedro existem duas faces com a mesma quantidade de lados. Considere a face S que tem a maior quantidade de lados, e suponha que esta face tenha N lados. Agora

Soluo:

considere N faces que so adjacentes face S. Cada uma desta faces tem entre 3 e N lados. Como temos N faces e

N 2 possveis valores para o nmero de seus lados, temos pelo Princpio das Casas dos Pombos que duas destas
faces tm a mesma quantidade de lados.

RA
2/2.
.

Exemplo 4

5.2

Exemplo 5

digamos A e B, tais que AB <

Soluo:

Pelo Princpio da Casa dos Pombos, existem dois dos cinco pontos em um mesmo quadrado de lado 1/2. A distncia entre estes pontos menor que a diagonal deste quadrado (prove!). Como a medida da diagonal de um quadrado de lado 2, existiro dois pontos determinado uma distncia menor que

Pombos Geomtricos

So dados 5 pontos no interior de um quadrado de lado 1. Prove que dentre estes pontos existem dois,

Divida o quadrado em quatro quadrados de lado 1/2.

Exemplo 6

Cinco pontos esto no interior de um tringulo equiltero de lado 1. Prove que dentre estes pontos

existem dois, digamos A e B, tais que AB < 1/2.

OBMEP PECI Preparao Especial para Competies Internacionais .

35

Soluo:

Dividimos o tringulo equiltero ligando os pontos mdios dos lados. Obtemos assim quatro tringulos

equilteros de lado 1/2. Dos cinco pontos considerados, pelo Princpio das Casas dos Pombos, existem dois que esto no mesmo tringulo pequeno, o qual vamos chamar de T1 . Os dois pontos que esto em T1 vamos chamar de A e B. Observe que como os pontos esto no interior do tringulo inicial, eles no podem coincidir com os vrtices de T1 . Como o lado de T1 1/2, ento AB < 1/2.

5.3 Problemas Propostos Problema 1


Cinquenta e um pontos esto no interior de um quadrado de lado 1. Prove que existem trs pontos

que podem ser cobertos por um crculo de raio 1/7.

Problema 2

(Balcnica Jnior 1997) Mostre que dados quaisquer 9 pontos no interior de um quadrado, mostre

que trs deles determinam um tringulo de rea menor que 1/8.

Problema 3 (OCM)

(a) Marca-se 151 pontos distintos no interior de um quadrado unitrio Q. Divide-se Q em 36 quadrados idnticos e justapostos e considera-se os crculos circunscritos a estes pequenos quadrados. Prove que existem pelo menos cinco pontos, dos 151 marcados, que esto no interior de um crculo de raio igual a 2/13;

(b) Marcamos 383 pontos distintos no interior de um cubo unitrio. Prove que, entre estes 383 pontos, existem pelo menos 4 que esto no interior de uma esfera de raio igual a 4/23.

5.4 Ramsey

trs pontos que determinam um tringulo monocromtico, isto , com os trs lados da cor azul ou os trs lados da cor vermelha.

Soluo:

aresta cheia e a cor vermelha por uma aresta tracejada.

D
A B
.

Exemplo 7

Pintamos cada um dos 15 segmentos determinados por 6 pontos de azul ou vermelho. Mostre que existem

Sejam A, B, C, D, E e F os pontos, como na figura esquerda. Vamos representar a cor azul por uma

RA
F E C D

FT
A B
.

Vamos olhar para uma ponto qualquer, por exemplo, A. Dos cinco segmentos que partem de A pelo menos trs so da cor azul ou pelo menos trs so da cor vermelha.

36

Combinatria Princpio da Casa dos Pombos Professor Fbio Brochero Suponha que partem trs segmentos azuis, sem perda de generalidade, para os pontos C, E e F. Se dois destes

pontos so ligados por um segmento azul, acabou. Caso contrrio os segmentos CE, EF e FC so vermelhos e o problema tambm acabou.

Este problema pode ser enunciado de outro modo: Prove que em qualquer grupo de seis pessoas existem trs pessoas que se conhecem mutuamente ou seis pessoas que no se conhecem mutuamente. Neste problema admitimos que a relao conhecer simtrica, ou seja, se A conhece B ento B conhece A. O desenho direita mostra que a afirmao no verdadeira para um grupo de 5 pessoas.

Exemplo 8

Cada ponto do plano colorido usando uma dentre k cores. Prove que podemos encontrar um retngulo

com todos os vrtices da mesma cor

Soluo:

Antes de resolver o caso geral, ilustraremos a soluo com o caso k = 2, isto , suponhamos que estamos

colorindo todos os pontos do plano de azul ou verde. Consideremos uma configurao formada por 3 retas paralelas horizontais e 9 retas paralelas verticais. Observe que cada reta horizontal corta cada reta vertical em trs pontos, e temos oito possveis configuraes para as cores desses pontos:

V V
.

A V V

V A V

V V

A A V

A V

V A A

A A A

Como temos nove retas verticais, pelo Princpio das Casas dos Pombos temos que uma das configuraes anteriores se repete pelo menos duas vezes. Como em cada uma das configuraes anteriores existe uma cor que se repete, A ideia do caso geral exatamente a mesma. Para isto, pegamos k + 1 retas horizontais e kk+1 + 1 retas verticais. Assim, cada reta vertical corta cada reta horizontal em k + 1 pontos. Como temos k cores, temos kk+1 possveis configuraes de cores para k + 1 pontos. Logo, temos duas retas horizontais que cortam as retas verticais em configuraes de cores iguais. Nesta configurao de cores, temos uma cor que ser repete, assim basta pegar os quatro pontos com a mesma cor na mesma configurao que eles formam um retngulo. pegando os pontos com a mesma cor, eles forma um retngulo queramos mostrar.

RA

FT
A A

5.4.1

Problemas Propostos Problema 4

que deve existir um caminho fechado formado por quatro segmentos, todos da mesma cor?

Problema 5 (IMO - 1964) Dezessete pessoas se comunicam por carta, cada uma com todas as demais. As cartas
tratam somente de trs assuntos. Cada par de pessoas trata em suas cartas de somente um deste temas. Demonstrar que existem pelo menos 3 pessoas que trocam cartas sobre o mesmo tema.

Problema 6

1. existem 4 pessoas que se conhecem mutuamente; 2. existem 4 pessoas tais que uma delas no conhece nenhuma das outras trs.

Problema 7
propriedade:

Dados kn pontos quaisquer no espao, 4 a 4 no coplanares, e associados nmeros inteiros entre 1 e n a cada aresta que liga dois desses pontos, h necessariamente um tringulo determinado por 3 deles a cujas arestas est associado o mesmo nmero.

(frica do Sul 1997) Seis pontos so ligados dois a dois por segmentos vermelhos ou azuis. verdade

Numa festa com 10 pessoas mostre que no mnimo uma das afirmaes abaixo verdadeira:

(Cone Sul 1993) Prove que dado um inteiro positivo n existe um inteiro positivo kn com a seguinte

OBMEP PECI Preparao Especial para Competies Internacionais

37

Problema 8

Se os pontos do plano forem coloridos com trs cores, sempre existiro dois pontos distanciados de

1 unidade com a mesma cor. Provar.


Problema 9
O conjunto dos inteiros positivos de 1 a 16 dividido em trs subconjuntos. Prove que existem trs

inteiros positivos a, b e c que pertencem a um mesmo subconjunto e satisfazem a condio a + b = c.

5.5 Pombos e Divisibilidade


Observemos que em alguns casos empregamos tal princpio de forma natural, por exemplo: Dados trs nmeros inteiros, existem dois que tem a mesma paridade. De fato como s temos duas possveis paridades (que fazem o papel de casas de pombo) e trs nmeros (que fazem o papel de pombos), pelo Princpio das Casas dos Pombos temos que dois o mais esto na mesma casa, isto , dois nmeros tem a mesma paridade. De fato este problema pode ser generalizado da seguinte forma: Dados n + 1 nmeros inteiros, ento dois destes nmeros deixam o mesmo resto quando divididos por n. A idia exatamente a mesma, mas neste caso temos que nossas casas so os possveis restos quando dividimos por n. Lembrando que o resto sempre um nmero inteiro na mesma casa, isto , temos, dois o mais nmeros que deixam o mesmo resto quando divididos por n. entre 0 e n 1, temos n possveis restos (casas de pombo) e n + 1 nmeros (pombos), segue que existem dois ou mais

Exemplo 9

Seja A um subconjunto de {1, 2, . . . , 200} com 101 elementos. Ento A possui dois elementos distintos

tais que um divisvel pelo outro. Provar.

Soluo:

Todo nmero inteiro positivo n pode ser escrito na forma n = 2a I, no qual a um inteiro no-negativo e

RA
15 = 20 15 18 = 22 9 1, 2, 22 , , 210

I um nmero inteiro positivo mpar. Dizemos que I o fator mpar de n. Se n for um nmero mpar temos a = 0. Se n for uma potncia de 2, temos I = 1. Exemplos: 21 = 20 21 32 = 25 1

Considerando os elementos de A, o fator mpar de cada um deles um elemento do conjunto B = {1, 3, . . . , 199}. Como

B possui 100 elementos e A possui 101 elementos, pelo Princpio da Casa dos Pombos, dois deles possuem o mesmo
fator mpar. Deste modo, A tem dois elementos distintos n1 = 2a1 I e n2 = 2a2 I, com partes mpares iguais. Se a1 < a2 , n1 divide n2 ; se a1 > a2 , n2 divide n1 .

FT

Observe que a afirmao do problema no verdade se A possui 100 elementos. De fato, o conjunto {101, 102, . . . , 200} possui 100 elementos e nenhum mltiplo de outro.

Exemplo 10

exista uma tripla (a, b, c) de nmeros distintos da mesma cor tais que a divide b e b divide c.

Soluo:

Suponha que 5 cores sejam suficientes. Considere as onze potncias de 2 menores que 2009:

Pelo Princpio das Casas dos Pombos, pelo menos uma cor utilizada 3 vezes para colorir estes onze nmeros. De fato, se cada cor fosse usada no mximo duas vezes, ento teramos no mximo 5 2 = 10 nmeros. Se 2m < 2n < 2p so essas potncias pintadas com a mesma cor, temos ento 2m | 2n e 2n | 2p , donde segue que

5 cores no so suficientes.
Vamos provar que possvel colorir os nmeros com 6 cores respeitando as condies do problema. Pintemos os nmeros assim:

Ache o nmero mnimo de cores necessrias para colorir os nmeros 1, 2, . . . , 2009 de modo que no

Vamos provar que o nmero mnimo de cores 6.

38

Combinatria Princpio da Casa dos Pombos Professor Fbio Brochero Nmeros Cor

{1, 2, 3} {4, . . . , 15} {16, . . . , 63} {64, . . . , 255} {256, . . . , 1023} {1024, . . . , 2009}

1 2 3 4 5 6

Para provar que esta colorao respeita as condies do problema, observe que dados dois nmeros naturais distintos m e n, tais que m | n, ento n 2m. Portanto se a, b e c so nmeros distintos tais que a divide b e b divide

c, ento c 4a.
Suponha que existam x, y e z da mesma cor tais que x | y e y | z. Pela observao segue que z 4x. Mas, dados dois nmeros x e z (x < z) pintados com a mesma cor acima, temos 1 <
z x

< 4, absurdo.

Exemplo 11
mltiplo de 1991.

(OBM-1991) Mostre que existe um nmero da forma 1 99 . . . 9 1 com mais de dois noves que um
n noves

Soluo:

Considere os nmeros 19991; 199991; 1999991;. . .. Pelo Princpio da Casa dos Pombos, dois desses

nmeros deixam o mesmo resto na diviso por 1991. Subtraindo esses nmeros, obtemos um nmero da forma

19 . . . 980 . . . (n noves) o qual divisvel por 1991.

RA
n vezes

Assim, como 1991 primo com 10, o nmero 19 . . . 98000 (n noves) divisvel por 1991 e, portanto,

1 9 . . . 9 8000 + 1991 = 1 9 . . . 9 1
n + 3 vezes

FT

divisvel por 1991.

Exemplo 12

Demonstrar que qualquer conjunto de n inteiros tem um subconjunto no vazio cuja soma dos ele-

Soluo:

mentos divisvel por n.

Sejam a1 , a2 , . . . , an os elementos do conjunto, e definamos

S1 = a1 S2 = a1 + a2
. . .

Sj = a1 + a2 + + aj
. . .

Sn = a1 + a2 + + an
Se algum dos Sj divisvel por n o problema fica resolvido. Se nenhum divisvel por n, ento existem dois ndices j e k com j < k tais que Sj e Sk deixam o mesmo resto quando os dividimos por n. Portanto, Sk Sj = aj+1 + + ak divisvel por n.

Soluo de Andr Reis Leal Olimpadas Brasileiras de Matemtica 9a a 16a SBM OBM

OBMEP PECI Preparao Especial para Competies Internacionais

39

Exemplo 13

Dado um conjunto S formado por inteiros positivos distintos, nenhum dos quais divisvel por nenhum

primo maior que 12. a. Mostrar que se S tem 33 elementos, ento existem dois elementos tais que seu produto um quadrado perfeito. b. Mostrar que se S tem 9 elementos, ento podemos escolher 4 ou menos elementos de S de forma que seu produto um quadrado perfeito.

Soluo:

Neste problema o mais importante a paridade dos expoentes na fatorao em primos de cada nmero.

De fato, cada elemento de S pode ser escrito na forma 2a 3b 5c 7d 11e , onde a, b, c, d e e so inteiros no negativos, e o produto de dois nmeros de S tambm da mesma forma. Alm disso um nmero um quadrado se cada um destes expoentes par. Por outra lado, se x = 2a1 3b1 5c1 7d1 11e1 e y = 2a2 3b2 5c2 7d2 11e2 so elementos de S, seu produto um quadrado perfeito se e somente se a1 tem a mesma paridade que a2 , b1 tem a mesma paridade que b2 , etc. Para solucionar a primeira parte, basta classificar os nmeros de S de acordo com paridade destes expoentes, isto , dois nmeros esto na mesma classe se seus expoentes tem mesma paridade. Como temos no total 25 = 32 possveis classes e o conjunto S tem 33 elementos, segue pelo Princpio das Casas dos Pombos que existem dois elementos na mesma classe, logo seu produto um quadrado perfeito. Para solucionar a segunda parte, considerem o conjunto de conjuntos R = {{x, y}|x, y S}. Observe que R tem

= 36 elementos formado por dois nmeros, e consideremos os 36 nmeros obtidos a partir do produto dos nmeros em cada elemento de R. Pela primeira parte do problema, o produto de dois destes nmeros um quadrado
2

perfeito. Daqui, temos duas possibilidades: Ou este produto formado por produto de 4 elementos distintos de S, ou os dois elementos escolhidos eram da forma {x, y}, {x, z}, com x2 yz igual a um quadrado, mas neste caso yz um tambm um quadrado. Fica de exerccio ao leitor mostrar que a parte (b) do problema continua valida quando o conjunto S possui 8 elementos.

FT

(9)

Exemplo 14 Soluo:

Prove que dentre 52 inteiros possvel escolher dois cuja soma ou diferena divisvel por 100.

Para aplicar o Princpio das Casas dos Pombos neste problema, construremos 51 casas da seguinte forma:

na casa j, colocamos o nmeros que deixam resto j ou 100 j quando dividimos por 100 onde j = 0, 1, . . . , 50. Como temos 52 nmeros, pelo Princpio das Casas dos Pombos, existem dois nmeros que esto na mesma casa. Caso estes dois nmeros deixem o mesmo resto quando so divididos por 100, sua diferena ser divisvel por 100. Por outro lado, se deixam restos distintos, sua soma ser divisvel por 100.

RA
a+b 2 ,

Exemplo 15

(a) Prove que em qualquer grupo de cinco nmeros inteiros existem trs cuja soma divisvel por 3. (b) Prove que em qualquer grupo de onze nmeros inteiros existem seis cuja soma divisvel por 6.

Soluo:

nmeros que deixam restos distintos quando dividimos por 3, e o segundo caso que isto no acontece. No primeiro caso, basta pegar estes trs nmeros que sua soma ser divisvel por 3. No segundo caso, podemos distribuir os nmeros em duas classes, dependendo do resto que deixam quando dividimos por 3, como temos cinco nmeros e dois possveis restos, segue que existem trs nmeros que deixam o mesmo resto e assim a soma destes nmeros divisvel por 3. Para solucionar o item b, vejamos como transformar este problema no problema anterior. De fato, como temos 11 nmeros, a pares e b mpares, podemos formar a pares de nmeros pares e b pares de nmeros mpares. Como 2 2 ou a ou b par, temos no total a + b = a+b = 5 pares de nmeros da mesma paridade. Com cada par a, b 2 2 2 construmos um nmero inteiro assim pelo problema anterior destes cinco nmeros inteiros possvel escolher

Para solucionar o item a. , consideremos dois possveis situaes: Entre os cinco nmeros temos trs

40

Combinatria Princpio da Casa dos Pombos Professor Fbio Brochero

trs tal que a soma divisvel por 3, e portanto a soma de seis dos nmeros originais divisvel por 6.

Exemplo 16

(Rioplatense) Prove que dentre 101 nmeros inteiros positivos quaisquer, possvel escolher 11 tal que

sua soma seja divisvel por 11.

Soluo:

Observemos que se entre os 101 nmeros temos onze nmeros que deixam restos distintos quando

dividimos por 11, temos que a soma destes onze nmeros divisvel por 11, assim podemos supor sem perda de generalidade que entre os 101 podemos obter no mximo 10 restos quando dividimos por 11, o que implica pelo Princpio das Casas dos Pombos que existem um resto que se repete onze vezes ou mais, assim somando onze nmeros que deixam o mesmo resto quando dividido por 11, esta soma divisvel por 11.

5.5.1 Problemas Propostos Problema 10 Problema 11


Prove que existe um inteiro positivo n tal que os quatro ltimos algarismos de 3n so iguais a 0001.

Para toda escolha de n inteiros existem no mnimo dois cuja soma ou diferena divisvel por 2k + 1.

5.6 Combinatria Aditiva

que existem em A quatro elementos distintos a, b, c, d tais que

Soluo:

claro que A possui infinitos elementos, j que possui elementos arbitrariamente grandes. Se ordenemos

os elementos de A de forma crescente: a1 < a2 < a3 < a4 < , temos que todas as diferenas a2 a1 , a4 a3 ,

a6 a5 , . . . so positivas e menores que 2013. Como temos infinitas diferenas, existem diferenas iguais, isto a2j a2j1 = a2i a2i1 com i = j e portanto a2j + a2i1 = a2i + a2j1 , como queramos mostrar.

Exemplo 18

cuja soma tambm esteja no conjunto. Seja A = {1, 2, 3, . . . , 200, 201}. (a) Encontre um subconjunto livre de somas do conjunto A com 101 elementos. (b) Prove que todo subconjunto de A com mais de 101 elementos no livre de somas.

Soluo:
(a) O subconjunto {1, 3, 5, . . . , 201} tem 101 elementos e livre de somas, pois a soma de quaisquer dois de seus elementos par. O subconjunto {101, 102, . . . , 201} tambm tem 101 elementos e livre de somas, porque a soma de quaisquer dois de seus elementos maior que o seu maior elemento. (b) Vamos provar que todo subconjunto de A com 102 elementos contm trs elementos x, y e z tais que x + y = z. Considere um subconjunto S = {x1 , . . . , x102 }, de A com x1 < x2 < < x102 . Observe que

Um conjunto de nmeros dito livre de somas se no contm dois elementos (possivelmente iguais)

RA
a + b = c + d.

Exemplo 17

O conjunto A N tal que existe um elemento de A entre quaisquer 2013 inteiros consecutivos. Mostre

1 x2 x1 < x3 x1 < < x102 x1 200.

FT

Seja k 1 um nmero natural. Determine o menor nmero natural n com a seguinte propriedade:

OBMEP PECI Preparao Especial para Competies Internacionais

41

Como A S tem 99 elementos, nem todas essas 101 diferenas podem estar em A S. Logo, ao menos uma delas est em S, donde segue que existem i e j, com 1 j < i 101, xj = xi x1 . Portanto, xi = xj + x1 e segue que S no livre de somas.

5.7 Problemas Avanados


O princpio das gavetas tambm aplicvel soluo de algumas desigualdades. Por exemplo, o seguinte problema proposto na IMO (Olimpada Internacional de Matemtica) de 1987.

Exemplo 19

(IMO 1987) Sejam x1 , x2 , . . . , xn nmeros reais tais que x2 + x2 + + x2 = 1. Demonstrar que n 2 1

para todo nmero natural k, k 2, existem n inteiros a1 , a2 , . . . , an , no todos nulos tais que |ai | k 1 e que

n |a1 x1 + a2 x2 + + an xn | (k 1) n . k 1
Soluo:
Primeiro vemos que a desigualdade de Cauchy-schwarz

|x1 | + |x2 | + + |xn |

Daqui segue se que para qualquer conjunto {b1 , b2 , . . . , bn } de nmeros inteiros no negativos menores ou iguais a

k 1 teremos que

Dado que cada bi pode assumir k valores, ento teremos kn n-uplas (b1 , b2 , . . . , bn ) tais que |b1 x1 +b2 x2 + +bn xn | est entre 0, (k 1) n . Assim, se dividimos este intervalo em kn 1 intervalos de tamanho mesmo subintervalo. Portanto,

RA
Aj = {x + tj | x A},

|b1 x1 + b2 x2 + + bn xn | (k 1)(|x1 | + |x2 | + + |xn |) (k 1) n.


k1 kn 1 n,

FT
2

nos permite afirmar que

x2 + x2 + + x2 n = n. n 1 2

pelo princpio

das gavetas existem duas n-uplas (b1 , . . . , bn ) e (c1 , . . . , cn ), tais que b1 x1 + + bn xn e c1 x1 + + cn xn esto no

|(b1 c1 )x1 + + (bn cn )xn |

k1 n, kn 1

Do mesmo modo podemos resolver o seguinte

Exemplo 20

existem t1 , . . . , t100 elementos de S tais que os conjuntos

como queramos demonstrar.

(IMO2003) Seja A um subconjunto de S = {1, 2, . . . , 1000000} com 101 elementos. Demonstrar que

j = 1, 2, . . . , 100

sejam disjuntos dois a dois.

Soluo:

Necessitamos escolher ti de tal forma x + ti = y + tj para todo x, y D e i = j, isto , que x y = tj ti .

Consideremos o conjunto D = {x y | x, y A}. Este conjunto tem no mximo 101100 + 1 = 10101 elementos. Logo precisamos escolher 100 nmeros ti , tal que a diferena de dois de eles no pertena a D. Vamos a escolher os ti por induo. Primeiro pegamos um elemento
2

Dados b1 , b2 , . . . , bn , c1 , . . . , cn nmeros reais, ento

(b2 + + b2 )(c2 + + c2 ) (|b1 c1 | + + |bn cn |)2 n n 1 1

42

Combinatria Princpio da Casa dos Pombos Professor Fbio Brochero

arbitrrio, e agora suponhamos que j temos escolhido t1 , tk com k 99 que cumprem as condies do problema.
Agora por cada ti escolhido existem 10101 que no podemos escolher, logo como escolhemos k existem 10101k <

999999 que no podemos escolher, assim simplesmente escolhemos uns dos que sobra.
Observao: O problema pode ser otimizado pegando D = {|x y| | x, y A} que ter no mximo elementos, e |ti tj | D. /
101100 2

+1

5.8 Problemas Complementares

Exemplo 21

(Torneio das Cidades) Existem 20 alunos em uma classe. Quaisquer dois deles possuem um av em

comum. Prove que existe um av que tem no mnimo 14 netos nesta classe.

Soluo:

Dizemos que uma criana do tipo [AB] se tem como avs a A e a B. Considere uma criana do tipo [AB].

Podem existir vrias crianas deste tipo. Sabemos que toda criana tem A ou B como av. Podemos supor que A no av de todos e que B tambm no av de todos, porque neste caso o problema estaria resolvido. Portanto, existe uma criana que no tem B como av, e portanto ela do tipo [AC]. Analogamente, existe uma criana que no tem

A como av, e ento, por um lado ela deve ter B como av, e como tem um av em comum com [AC], tambm deve ter C como av. Portanto, essa criana do tipo [BC]. Conclumos que existem somente trs tipos de crianas: [AB], [AC] ou [AC], e somente trs avs.
Desse modo, as 20 crianas tem 40 avs contando com multiplicidade e como 40 = 3 13 + 1, segue pelo Princpio da Casa dos Pombos que existe um av que est sendo contado com multiplicidade pelo menos 14, isto , existe um av com pelo menos 14 netos na turma.

FT
= 151 50 50 25 A = 2525 A. 2

Exemplo 22

Ana escolhe vinte e cinco nmeros diferentes dentre os nmeros 1, 2, . . ., 50. Bruno escolhe vinte e

cinco nmeros diferentes dentre os nmeros 51, 52, . . ., 100. Ana e Bruno fazem essa escolha de modo que dentre todos os nmeros escolhidos por eles no existam dois cuja diferena seja igual a 50. Calcule a soma de todos os nmeros escolhidos.

Soluo:

Observemos que se Ana escolhe o nmero j, ento Bruno no escolheu o nmero 50 + j. Assim como

Ana escolheu 25 nmeros, temos que 25 nmeros entre 51 e 100 que no podem ser escolhidos por Bruno. Em outras palavras, os nmeros escolhidos por Ana determinam univocamente os nmeros escolhidos por Bruno. Sejam por Bruno

j1 , j2 , . . . , j25 os nmeros escolhidos por Ana e A a soma destes nmeros. Segue que a soma dos nmeros escolhidos

RA

(51 + 52 + + 100) ((50 + j1 ) + (50 + j2 ) + + (50 + j25 ))

Portanto, a soma dos nmeros escolhidos pelos dois 2525.

Exemplo 23

So dados 40 pesos: 1, 2, . . . , 40 gramas. Dez pesos com massa par foram colocados sobre o prato

esquerdo da balana. Em seguida, dez pesos com massa mpar foram colocados no prato direito da balana. Os pratos esquerdo e direito da balana esto equilibrados agora. Prove que sobre um prato da balana existem dois pesos cuja diferena em massa exatamente 20 gramas.

Soluo:

Soluo de Gabriel Ilharco.

So pesos pares os pesos 2, 4, 6, . . . , 40. Dividamo-os em dois conjuntos A e B.

A = {2, 4, 6, 8, 10, 12, 14, 16, 18, 20}

OBMEP PECI Preparao Especial para Competies Internacionais e

43

B = {22, 24, 26, 28, 30, 32, 34, 36, 38, 40}.
Para no haver diferena 20 na balana de pesos pares, temos que, para qualquer peso escolhido em A, no podemos escolher seu correspondente em B. Isso s possvel se fizermos da seguinte forma: 2 ou 22, 4 ou 24, 6 ou 26, . . . ,

18 ou 38, 20 ou 40. Em que o ou exclusivo, isto , ou escolhemos um ou outro.


Desse modo, a soma dos pesos dessa balana dada por 2 + 4 + 6 + + 20 + 20k, em que k inteiro e 0 k 10, ou seja, o peso total 110 + 20k. Procedendo de modo anlogo com os pesos mpares, temos que a soma ser 100 + 20k , com k tambm inteiro. Deste modo, temos que 110 + 20k = 100 + 20k , o que resulta em 2(k k) = 1, o que um absurdo j que k e k so inteiros, e 2(k k) par enquanto que 1 mpar.

5.9 Problemas Propostos Problema 12


Dez amigos trocam postais, cada um deles enviando 5 postais para diferentes amigos. Prove que no

mnimo dois deles trocaram postais entre si.

Problema 13
na festa.

Mostre que em uma festa com 20 pessoas sempre existem duas com a mesma quantidade de amigos

Problema 14

Um professor e seus 30 alunos escreveram, cada um, os nmeros de 1 a 30 em uma ordem qualquer.

A seguir, o professor comparou as seqncias. Um aluno ganha um ponto cada vez que um mesmo nmero aparece na mesma posio na sua seqncia e na do professor. Ao final, observou-se que todos os alunos obtiveram quantidades diferentes de pontos. Mostre que a seqncia de pelo menos um aluno coincidiu com a seqncia do professor.

Problema 15 (Cone Sul 1992) Determine a quantidade de elementos que pode ter um conjunto B contido em

{1, 2, . . . , n} com a seguinte propriedade:

Para quaisquer a e b elementos de B, com a diferente de b, (a b) no divide a + b.

Problema 16

Uma senha de 7 dgitos chamada boa se todos os dgitos so diferentes. Um cofre tem uma senha

boa. O cofre aberto se uma senha boa for testada e se um de seus dgitos for igual ao dgito correspondente da senha do cofre. Existe um mtodo para abrir um cofre cuja senha no conhecemos usando menos que 7 tentativas?

cartas (no obrigatoriamente consecutivas) que esto em ordem crescente ou decrescente.

Problema 18

Demonstrar que entre os 51 existem dois nmeros tais que um divisvel pelo outro.

Problema 19

que a diferena entre k e seu inteiro mais prximo menor que 1/n.

Problema 20

Problema 17

Os nmeros de 1 at 10 so colocados de forma arbitrria formando uma fila. Mostre que existem 4

Seja A = {1, 2, . . . , 99, 100} o conjunto dos nmeros de 1 a 100 e deste escolhemos 51 nmeros.

Seja um nmero irracional. Demonstrar que para todo inteiro n existe um inteiro 0 < k n, tal

Se escolhem 7 pontos no interior de um retngulo de 23. Demonstrar que sempre possvel encon-

trar dois pontos tal que sua distancia menor o igual a

Problema 21

Se escolhem 9 pontos no interior de um quadrado de lado 1. Demonstrar que possvel escolher 3


1 8.

deles de tal forma que o rea do tringulo que formam menor que

Problema 22

(OCM) Separamos o conjunto N = {1, 2, . . . } como unio disjunta N = L (N L). O conjunto L

finito, tem g elementos e se os nmeros naturais a e b so tais que a L e b L, ento a + b L. Mostre que o maior elemento de L menor ou igual a 2g 1.

Problema 23

Dado um nmero irracional u, demonstrar que sempre possvel encontrar um nmero infinito de

RA
2.

FT

44 nmeros racionais
p q

Combinatria Princpio da Casa dos Pombos Professor Fbio Brochero de tal forma que

p 1 < 2. q q
p q

Um problema mais difcil que este demonstrar existem racionais

de tal forma que

p 1 < . q 5q2

Problema 24 (IMO 1983) Seja ABC um tringulo equiltero e E o conjunto de todos os pontos contidos nos
segmentos AB, BC e CA (incluindo A, B e C). verdade que, para toda partio de E em dois subconjuntos disjuntos, no mnimo um dos dois subconjuntos contm os vrtices de um tringulo retngulo? Justifique sua resposta.

Problema 25
quarta potncia.

(IMO 1985) Dado um conjunto M com 1985 inteiros positivos distintos, nenhum dos quais tem

divisores primos maior do que 23, mostre que M contm um subconjunto de 4 elementos tal que seu produto uma

permutao dos inteiros 1, 2, . . . , m, e f(x) = x1 n1 + + xm nm . Demonstre que existem duas permutaes a e b tais que f(a) f(b) divisvel por m!.

Problema 27

Demonstrar que dados 7 nmeros reais sempre possvel escolher 2 deles, digamos a e b, tais que

ab 1 < . 1 + ab 3

RA

FT

Problema 26

(IMO 2001) Sejam n1 , n2 , . . . , nm inteiros com m mpar. Denotemos por x = (x1 , . . . , xm ) uma

Combinatria

Grafos

6.1 Viajando em Pecilndia

Exemplo 1

Pecilndia um pequeno pas que possui 10 cidades e 37 estradas. Cada estrada une exatamente duas

cidades. Duas cidades so ligadas por no mximo uma estrada. Prove que possvel viajar entre duas cidade quaisquer de Pecilndia, percorrendo uma ou mais estradas.

Soluo:

Vamos representar as cidades C1 , C2 , . . . , C10 como pontos no plano e as estradas como segmentos

unindo dois dos pontos.

C3 C4 C5 C6

Por exemplo, na figura acima, o segmento significa que existe uma estrada entre as cidades C4 e C10 . Sabemos que ao todo Pecilndia possui 37 estradas, mas no sabemos quantas estradas partem de cada cidade. Sejam d1 , d2 , . . . , d10 as quantidades de estradas que partem de C1 , C2 , . . . , C10 . Podemos supor, sem perda de generalidade, que d1 d2 d10 . quantidade de estradas. Deste modo, Quanto vale a soma d1 + d2 + + d10 ? Como cada estrada tem duas extremidades, esta soma o dobro da

A soma destes dez nmeros inteiros no negativos igual a 74. O maior deles no mnimo 8. De fato, se todos fosse menores ou iguais a 7, a soma dos dez seria no mximo 70. Portanto, d10 8. Se d10 = 9, o problema acabou, pois todas as cidades estariam ligadas a C10 e para viajar entre quaisquer duas outras cidades, bastaria passar por C10 .

RA
C7 C8 C3 C4 C5 C6 C7
45 .

d1 + d2 + + d10 = 2 37 = 74.

FT
C2 C1
.

C10

C9

C2 C1 C10 C9 C8

46

Combinatria Grafos Professor Fbio Brochero

Se d10 = 8, ento existe uma cidade X que no est ligada diretamente a C10 . possvel viajar entre quaisquer duas cidades diferentes de X passando por C10 . Ser que X pode ser uma cidade isolada? Se este fosse o caso, existiriam no mximo 9 8/2 = 36, estradas em Pecilndia, o que contradiz a hiptese do problema.

C3 C4 C5 C6 C7
.

C2 C1 C10 C9 C8

Portanto, X est ligada a pelo menos uma outra diferente de C10 , digamos, Y . Ento possvel viajar para X de uma cidade qualquer diferente de C10 , de X e de Y , fazendo o percurso Ci C10 Y X.

Essa estrutura de vrtices e arestas que unem pares de vrtices conhecida como grafo. No nosso problema tnhamos um grafo com 10 vrtices e 37 arestas. Dizemos que dois vrtices so adjacentes ou vizinhos, se existe uma arestas que os une. em dois vrtices, podemos concluir que

A quantidade de arestas que incide em cada vrtice o grau de um vrtice. Como toda aresta incide em exatamente

sendo A o nmero de arestas.

Um caminho uma sequncia de vrtices consecutivos so unidos por uma aresta. Se o primeiro vrtice de um caminho igual ao ltimo dizemos que tal caminho um ciclo. Se quaisquer dois vrtices do grafos pertencem a um mesmo caminho, dizemos que o grafo conexo. O nosso problema poderia ser enunciado ento, como: Prove que todo grafo com 10 arestas e 37 vrtices conexo.

6.2 Entre Amigos

Exemplo 2
do grupo.

(a) possvel que haja exatamente 1997 pessoas no grupo? (b) possvel que haja exatamente 1998 pessoas no grupo? Observao: supe-se que se A conhece B, ento B conhece A.

Soluo:

representadas por estes vrtices se conhecem. (a) Em linguagem de grafos, o problema pergunta em (a) se existe um grafo com 1997 vrtices tal que todos possuem grau 101. Suponhamos que tal grafo exista. A soma dos graus de todos os vrtices ento igual a 1997 101. Mas a soma de todos os graus igual ao dobro do nmero de arestas. Como 1997 101 mpar, tal grafo no pode existir.

(Rioplatense 1997) Em um grupo de pessoas, sabe-se que cada uma conhece exatamente 101 pessoas

Vamos representar as pessoas como vrtices de um grafo e ligar dois vrtices por uma aresta se as pessoas

RA

d1 + d2 + + dn = 2A,

FT

OBMEP PECI Preparao Especial para Competies Internacionais

47

(b) Neste caso queremos um grafo com 1998 vrtices tal que cada um tem grau 101. O argumento usado em (a) no se aplica, pois a soma de todos os graus igual a 1998 101 e a quantidade de arestas ento 999 101. Tal grafo existe sim. Considere as pessoas P1 , P2 , . . . , P1998 . Suponha que elas esto ao redor de um crculo nesta ordem e olhando para o centro. Se cada pessoa conhece as 50 pessoas mais prximas esquerda, as 50 mais prximas direita e a pessoa que est diametralmente oposta, ento cada uma conhecer exatamente 101 pessoas. No difcil verificar que esse exemplo realmente funciona. O diagrama abaixo mostra uma situao anloga para 12 pessoas, cada uma conhecendo exatamente 5 outras. Duas direita, duas esquerda e a pessoa mais distante.

P3 P4 P5 P2 P1

P6

P12

P7 P8

FT
P11 P10 P9

Como a soma dos graus de todos os vrtices de um grafo um nmero par, a quantidade de vrtices de grau mpar tem que ser par. De fato, se a quantidade de vrtices de grau mpar fosse mpar, a soma de todos os graus seria mpar, um absurdo. Vamos mais uma vez aplicar esse resultado no problema a seguir.

Exemplo 3

que existem quatro estudantes que possuem o mesmo nmero de amigos.

Soluo:
aresta.

O grau de cada vrtice pertence ao conjunto A = {68, 69, . . . , 101} de 34 elementos. Como existem 102 vrtices e

34 possveis graus, se supusermos que no existem quatro com o mesmo grau, teremos que cada elemento de A est associado exatamente 3 vrtices (pois 102 = 34 3). Como existem 17 nmeros mpares no conjunto A, o nmero
de vrtices de grau mpar do nosso grafo ser 17 3 que um nmero mpar, absurdo. Portanto, devero existir quatro vrtices com o mesmo grau, ou seja, quatro estudantes com a mesma quantidade de amigos.

Cada um dentre os 102 estudantes em uma escola amigo de pelo menos outros 68 estudantes. Prove

Os estudantes so os vrtices do nosso grafo e amizade entre dois estudantes representada por uma

RA

O nosso prximo problema do Torneio Internacional das Cidades, uma competio que teve origem na Rssia e da qual participam cidades dos cinco continentes. A soluo abaixo foi apresentada nesta prova pelo aluno do PECI, Victor de Oliveira Bitares de Betim MG.

48

Combinatria Grafos Professor Fbio Brochero

Exemplo 4

2000 pessoas registraram-se em um novo website. Cada uma destas convidou 1000 pessoas (dentre

as registradas no website) para ser sua amiga. Duas pessoas so consideradas amigas se e somente se elas enviaram convite uma para a outra. Qual pode ser o nmero mnimo de pares de amigos neste website?

Soluo:

Podemos contar a quantidade de segmentos que podem ser formados por 2000 pontos num plano, sendo

que no existem 3 colineares. Esse nmero equivalente a

( ) 2000 . 2
A quantidade total de convites que sero enviados 2000 1000, pois cada pessoa envia 1000 convites. Ento temos que um par de amigos um dos

( ) 2000 2000 1999 = 2 2

segmentos que preenchido com 2 convites (um da primeira pessoa para a segunda e outra da segunda para a primeira). O mnimo de pares de amigos obtido quando preenchemos todos os segmentos pelo menos uma vez, restando assim

convites. ram 1000 pares de amigos.

Como todos os segmentos j foram preenchidos uma vez, podem ser preenchidos no mximo duas vezes e restaPodemos dar um exemplo no qual h exatos 1000 pares de amigos. Chamamos as pessoas de p1 , p2 , . . . , p2000 e usamos pi pj para indicar que pi enviou um convite para pj . Ento fazemos:

Pela representao acima, vemos que os pares de amigos so {p1 , p1001 }, {p2 , p1002 }, . . . , {p1000 , p2000 } e que h exatos

RA
. . . . . .

p1 p2 , p3 , . . . , p1001 p2 p3 , p4 , . . . , p1002

p1000 p1001 , p1002 , . . . , p2000

p1001 p1002 , p1003 , . . . , p1

p2000 p1 , p2 , . . . , p1000

FT

2000 1000

2000 1999 = 1000 2

1000 pares de amigos, que , portanto o nmero mnimo de pares de amigos.

Exemplo 5

A populao de Peclia, capital da Pecilndia, formada por n pessoas que respeitam as seguintes leis:

(i) Dentre quaisquer trs pessoas existem pelo menos duas pessoas que no se conhecem; (ii) Quaisquer duas pessoas que no se conhecem possuem exatamente duas pessoas conhecidas em comum. Mostre que existe um inteiro k tal que n = 1 + conhecidos.
k(k+1) . 2

Conclua que todas as pessoas tm a mesma quantidade de

Soluo:

Seja x uma pessoa qualquer e {x1 , x2 , . . . , xm } os conhecidos de x.

Como dentre trs pessoas quaisquer existem duas que no se conhecem, ento quaisquer duas pessoas xi e xj conhecidas de x no se conhecem.

OBMEP PECI Preparao Especial para Competies Internacionais

49

Assim, pela condio (ii), xi e xj possuem exatamente dois conhecidos em comum, um dos quais x. Seja yi,j o outro amigo comum. claro que x e yi,j no se conhecem porque caso contrrio teramos que quaisquer entre x, xi e yi,j se conheceriam, assim xi e xj so os nicos amigos comuns aos dois. Por outro lado, se y no conhece x, existem exatamente dois conhecidos comuns, que claramente esto no conjunto

{x1 , x2 , . . . , xm }. Conclumos que cada par xi , xj determina exatamente uma pessoa que x no conhece e vice-versa. ( ) m(m+1) Portanto o nmero total de pessoas 1 + m + m = 1 + . 2 2
Agora se a pessoa z conhece k pessoas, pelo mesmo processo temos que o nmero total de pessoas 1 + donde conclumos que
m(m+1) 2 k(k+1) , 2

k(k+1) , 2

assim k = m. Portanto, todas as pessoas conhecem exatamente a mesma

quantidade de outras em Pecilndia.


Para terminar os exemplos resolvidos, apresentamos um problema com a soluo do aluno Andr Macieira, de Belo Horizonte MG.

nmero de amigos no possuem amigos em comum. Se existem cientistas que se conhecem, prove que existe um cientista que possui apenas um amigo.

Soluo:

Construmos um grafo como quase sempre, ou seja vrtices representam cientistas e dois vrtices esto

ligados se cientistas se conhecem.

Considere dentro deste grafo o vrtice (ou um dos vrtices) de maior grau e seja k este grau. Veja que existem exatamente outros k vrtices ligados ao de maior grau e estes vrtices apresentam como grau um nmero entre 1 e k e todos estes vrtices tem grau diferente, uma vez que possuem um amigo em comum. Suponha que nenhum destes vrtices possui grau 1. Ento teremos k 1 opes de escolha para o grau destes vrtices, e pelo Princpio das Casas dos Pombos haver dois com mesmo grau, o que nos leva a uma contradio, pois ambos possuem um amigo comum. Portanto existe um vrtice de grau 1, como queramos demonstrar.

RA

.

FT
.

Exemplo 6

Cientistas esto reunidos para um congresso matemtico. Sabe-se que dois cientistas com o mesmo

Um grafo dito completo quando quaisquer dois vrtices so ligados por uma aresta. O grafo completo com n

D
.

vrtices denotado por Kn .

K3

K4

K5

K6

No prximo exemplo mostraremos que todo grafo com 2n vrtices e mais de n2 arestas possui um K3 .

6.3 Tringulos em Grafos

Exemplo 7
(a) Considere um grafo com 2n vrtices, n > 1 e pelo menos n2 + 1 arestas. Mostre que existem trs vrtices determinando um tringulo. (b) Mostre que para cada natural n existe um grafo com 2n vrtices e n2 arestas sem um tringulo.

50

Combinatria Grafos Professor Fbio Brochero

Soluo:
(a) Para resolver esta parte vamos utilizar o princpio de induo finita.

Para n = 2, fcil perceber que qualquer grafo com 4 vrtices e 5 arestas possui um tringulo. Vamos admitir todo grafo com 2n vrtices e pelo menos n2 + 1 arestas possui um tringulo. Considere um grafo com 2(n + 1) vrtices e pelo menos (n + 1)2 + 1 arestas. Vamos provar que pelo menos um
tringulo formado. Suponha que nenhum tringulo seja formado. Considere dois vrtices A e B conectados por uma aresta.

B A
.

Se o subgrafo determinado pelos outros 2n vrtices tiver pelo menos n2 + 1 arestas, pela hiptese de induo teremos um tringulo. Admita, ento, que este subgrafo de 2n vrtices possui no mximo n2 arestas. Cada um dos 2n pontos vizinho de no mximo um dentre A e B porque se for vizinhos de ambos encontramos um tringulo. Deste modo utilizamos no mximo n2 + 2n + 1 arestas: (n2 no subgrafo com 2n vrtices, 2n ligando A e B ao subgrafo e 1 para ligar a A a B). Mas n2 + 2n + 1 < (n + 1)2 + 1. Absurdo! Ento, pelo menos um tringulo formado e o resultado segue por induo.

(b) Para a parte (b) basta dividir os vrtices em dois grupos de n e ligar cada vrtice de um grupo a todos os vrtices do outro. Deste modo, obtemos n n = n2 arestas e nenhum tringulo formado. De fato, se considerarmos trs vrtices quaisquer, pelo menos dois deles estaro no mesmo grupo e portanto no so adjacentes.

RA
.

FT

Exemplo 8

(Rioplatense) Patrcia desenhou em uma folha de papel vrios pontos ao redor de uma circunferncia.

Em seguida traou alguns segmentos com extremidades nestes pontos. Ao final, observou que em sua figura partiam pelo menos trs segmentos de cada ponto e que no existiam tringulos nem quadrilteros com vrtices nos pontos desenhados. Determine o menor nmero possvel de pontos desenhados por Patrcia e uma possvel representao dos segmentos traados.

Soluo:

Vamos provar que o menor nmero possvel de pontos 10.

Considere um ponto qualquer A. Deste devem sair no mnimo 3 segmentos, para trs outros pontos. Estes trs pontos no podem estar ligados por um segmento, pois no h tringulos na figura de Patrcia. Portanto, de cada um desses trs pontos devem sair no mnimo 2 outros segmentos. As extremidades desses segmentos devem ser todas distintas, pois caso contrrio haveria a formao de um quadriltero. Segue que o nmero de pontos no mnimo

1 + 3 + 3 2 = 10.

OBMEP PECI Preparao Especial para Competies Internacionais

51

A.

Um exemplo com 10 pontos mostrado na figura.

FT
nmeros vizinhos seja um quadrado perfeito. soma de quaisquer dois nmeros vizinhos seja um quadrado perfeito. correta.

6.4 Problemas Propostos Problema 1

(a) Mostre que os nmeros de 1 a 16 podem ser escritos numa reta, de tal modo que a soma de quaisquer dois

(b) Mostre que os nmeros de 1 a 16 no podem ser escritos ao redor de uma circunferncia, de tal modo que a

Problema 2

Marquem 6 pontos sobre uma circunferncia. Eu quero que vocs pintem o maior nmero de cordas determinadas por estes pontos, de modo que no existam quatro dos pontos sobre a circunferncia determinando um quadriltero com todos os lados e diagonais coloridos. (a) Edmilson encontrou uma soluo correta colorindo 12 cordas. Exiba uma maneira de como fazer isto. (b) Gustavo afirmou ter encontrado uma soluo na qual pintara 13 cordas. Mostre que a soluo de Gustavo no est

Problema 3

uma pessoa que conhece as outras trs. Qual o nmero mnimo de pessoas na festa que conhecem todas as demais? (Assuma que se A conhece B, ento B conhece A)

Problema 4

Estas estradas no se intersectam. Se necessrio, alguma delas passa sobre ou sob outra por meio de pontes. Um mgico mau estabeleceu regras de mo - nica nas estradas de tal modo que se algum sai de uma certa cidade ele no conseguir voltar. Prove que

(OBM 1997) A professora de Matemtica props o seguinte problema para seus alunos:

(USAMO 1982) Ema festa com 1982 pessoas, em qualquer grupo de quatro deles existe no mnimo

(Torneio das Cidades) Em Shvambrania existem n cidades, duas quaisquer ligadas por uma estrada.

RA

52 (a) possvel estabelecer tais regras

Combinatria Grafos Professor Fbio Brochero

(b) Existe uma cidade a partir da qual podemos chegar a qualquer outra, e existe uma cidade que no podemos sair. (c) O mgico pode realizar seu objetivo de n! maneiras.

Problema 5

Em um pas h 1995 cidades. Duas quaisquer delas se comunicam diretamente, por via area ou

terrestre (mas nunca por ambos os meios). H tambm duas cidades X e Y tais que no possvel viajar de uma outra, mesmo com paradas em outras cidades, s por via terrestre. Prove que, dadas duas cidades quaisquer das 1995, possvel ir de uma outra por via area, fazendo no mximo uma escala (as rodovias so de mo dupla e os vos em ambos os sentidos).

Problema 6

(Hungria 1977) Trs escolas possuem cada uma n estudantes. Cada estudante conhece no total n + 1

estudantes das outras duas escolas. Prove que existem trs estudantes das diferentes escolas que se conhecem.

Problema 7

A cintura de um grafo o tamanho do menor ciclo no grafo. Seja G um grafo com cintura 5 para o

qual todos os vrtices tem grau maior ou igual a d. Mostre que G tem no mnimo d2 + 1 vrtices.

menos 1 linha de nibus, no qual: (i) Cada linha passe por exatamente 3 paradas.

(ii) Cada duas linhas distintas tenham exatamente uma parada em comum.

(iii) Para cada duas paradas distintas exista exatamente uma linha que passe por ambas. Determine o nmero de paradas de nibus da cidade.

RA

FT

Problema 8 (Cone Sul 1998) O prefeito de uma cidade deseja estabelecer um sistema de transportes com pelo

D
Euclides.

Teoria dos Nmeros


Professor Svio Ribas

RA
PARTE III .

Assuntos do Mdulo: Divisibilidade; Induo; Algoritmo de Pr-requisitos: Captulos 1, 2 e 3 da Apostila 1 do PIC.

FT

Nmeros

Ferramentas Bsicas

7.1 Aquecendo os motores 7.1.1 Critrios de divisibilidade

2, se for par. 4, se o nmero formado pelos dois ltimos algarismos da direita for divisvel por 4. 8, se o nmero formado pelos trs ltimos algarismos da direita for divisvel por 8. 3, se a soma dos seus algarismos for divisvel por 3. 9, se a soma dos seus algarismos for divisvel por 9. 5, se terminar em 0 ou 5. 10, se terminar em 0.

25, se terminar em 00, 25, 50 ou 75.

11, se a soma dos algarismos de ordem mpar menos a soma dos algarismos de ordem par for um nmero divisvel por 11. 6, 15, 22, . . . , se for divisvel simultaneamente pelos seus dois fatores primos. 12, se for divisvel simultaneamente por 3 e 4.
7.1.2

Exerccios para esquentar Problema 1


so primos.

Problema 2 Problema 3 Problema 4

(OMM 2005) Mostre que todos os nmeros inteiros de 1 a 100 que no so divisveis por 2, 3, 5 nem 7

Quantos divisores tem o nmero 72? Qual a soma desses divisores? Demonstre que a equao
1 a

Os alunos da turma de Pedro praticam a soma e a multiplicao de nmeros inteiros. A professora

escreve os nmeros de 1 a 9 em nove fichas, uma para cada nmero, e as coloca em uma urna. Pedro retira trs fichas e deve calcular a soma e o produto dos trs nmeros correspondentes. Ana e Julin fazem o mesmo, esvaziando a urna. Pedro informa professora que retirou trs nmeros consecutivos cujo produto 5 vezes a soma. Ana informa que no tem nenhum primo, mas sim dois consecutivos e que o produto desses trs nmeros 4 vezes a soma dos mesmos. Quais nmeros retirou Julin? 54

RA
+
1 b

1 c

FT
1 d

1 e

1 f

= 1 no tem soluo nos inteiros mpares.

OBMEP PECI Preparao Especial para Competies Internacionais

55

Problema 5

(Olimpada de Maio) Na minha calculadora, uma das teclas de 1 a 9 est com defeito. Ao pression-

la, aparece na tela um dgito entre 1 e 9 que no o correspondente. Quando tentei escrever o nmero 987654321 apareceu na tela um nmero divisvel por 11 e que deixa resto 3 ao ser dividido por 9. Qual a tecla defeituosa? Qual o nmero que apareceu na tela?

7.2 Ferramentas preliminares 7.2.1 Princpio da Induo Finita - 1 forma


Seja P(n) uma propriedade do nmero natural n. Uma maneira de provar que P(n) verdadeira para todo natural

n n0 utilizar o Princpio da Induo Finita (PIF), que um dos axiomas que caracterizam o conjunto dos nmeros
naturais. O PIF consiste em verificar duas coisas: 1. (Base da Induo) P(n0 ) verdadeira;

Na base da induo, verificamos que a propriedade vlida para um valor inicial n = n0 . O passo indutivo consiste em mostrar como utilizar a validade da propriedade para um dado n (a chamada hiptese da induo) para provar a validade da mesma propriedade para o seu consecutivo n + 1. Exemplo 7.2.1. Para todo inteiro positivo n temos:

De fato, para n = 1 temos 1 =

algum inteiro positivo n = k, ou seja, suponha que temos:

Somando k + 1 a ambos os lados da igualdade acima (esse o passo!) obtemos:

que justamente a equao 7.3 para n = k + 1. Logo, pelo PIF, a igualdade vale para todo n 1.

7.2.2

Princpio da Induo Finita - 2 forma


Uma variao do PIF consiste em diferenciar um pouco a hiptese e mostrar que: 1. P(n0 ) verdadeira; 2. se P(k) verdadeira para todo k satisfazendo n0 k n ento P(n + 1) verdadeira; ento podemos concluir que a propriedade P vlida para todo natural n n0 . Exemplo 7.2.2. A sequncia de Fibonacci Fn definida recursivamente por F0 = 0, F1 = 1 e se n 2 ento Fn =

Fn1 + Fn2 . Podemos escrever Fn em funo apenas de n como: Fn = n n ,


(7.4)

RA
1(1+1) , 2

1 + 2 + + n =

FT
n(n + 1) . 2 k(k + 1) (k + 1)(k + 2) + (k + 1) = , 2 2

2. (Passo Indutivo) Se P(n) verdadeira para algum nmero natural n n0 ento P(n + 1) tambm verdadeira.

(7.3)

que verdade (essa a base!). Suponha que a iguadade acima seja vlida para

1 + 2 + + k =

k(k + 1) (essa a hiptese!). 2

(1 + 2 + + k) + (k + 1) =

56 onde =
1+ 5 2

Teoria dos Nmeros Ferramentas Bsicas Professor Svio Ribas e=


1 5 2

so as razes de x2 = x + 1.
0 0

De fato, para n = 0 e n = 1 em 7.4 temos F0 = igualdade 7.4 vlida para todo 0 k n. Temos:

= 0 e F1 =

= 1 (base). Seja n 1 e suponha que a

Fn+1 = Fn + Fn1 =

n n n1 n1 (n + n1 ) (n + n1 ) n+1 n+1 + = = ,

que justamente a equao 7.4 para n + 1, uma vez que x2 = x + 1 implica xn+1 = xn + xn1 . No exemplo acima, observe que o passo indutivo utiliza os valores de dois termos anteriores da sequncia de Fibonacci, logo precisamos verificar a base de induo para os dois primeiros termos F0 e F1 e no apenas para o primeiro.

7.2.3 Princpio da Boa Ordenao


O Princpio da Boa Ordenao (PBO) equivalente ao PIF. Ele diz que todo subconjunto no vazio dos nmeros naturais possui um elemento mnimo. Exemplo 7.2.3. Toda funo f : N N no-crescente constante a partir de um certo nmero natural. De fato, seja

A N a imagem de f. Pelo PBO, A possui um elemento mnimo a0 . Seja n0 um natural tal que f(n0 ) = a0 . Como f no-crescente, f(n) f(n0 ) = a0 para todo n n0 . Por outro lado, f(n) a0 . Logo, f(n) = a0 para todo n n0 .
7.2.4 Princpio da Casa dos Pombos

O Princpio da Casa dos Pombos (PCP) diz que se n + 1 pombos forem alocados em n casas ento ao menos uma casa conter 2 pombos. A demonstrao desse fato por absurdo: suponha que cada casa contenha no mximo um pombo, ento o nmero de pombos no mximo n, contradio. Podemos generalizar o PCP da seguinte forma: se mais do que kn pombos forem alocados em n casas ento ao menos uma casa conter mais que k pombos, e a demonstrao a mesma. Exemplo 7.2.4. Ao escolher 51 nmeros do conjunto A = {1, 2, . . . , 99, 100} sempre existem 2 que so primos entre si.

An , onde An = {2n 1, 2n}. Como so 50 conjuntos menores e 51 nmeros escolhidos de A, algum Aj foi todo escolhido e portanto 2j 1 e 2j satisfazem o que queramos pois mdc(2j 1, 2j) = 1.
De fato, escreva A =
n=1

50

7.2.5

Exerccios de xao
1. Demonstrar que:

(a) 12 + 22 + + n2 =

(b) 13 + 23 + + n3 = (1 + 2 + + n)2 . (c) 6 divide n3 n para todo n inteiro positivo. (d) 3 divide 2n + 1 para todo n mpar. (e) 1 + q + q2 + + qn1 =
qn 1 q1

D
n(n+1)(2n+1) . 6

(f) F1 + F2 + F3 + + Fn = Fn+2 1 para todo n inteiro positivo, onde (Fn ) a sequncia de Fibonacci dada por F1 = F2 = 1 e Fn+2 = Fn+1 + Fn (n 1). 2. Prove que toda sequncia com n2 + 1 nmeros reais contm ou uma subsequncia crescente de tamanho n + 1 ou uma subsequncia decrescente de tamanho n + 1. Alm disso, mostre que isso pode ser evitado com n2 nmeros reais.

RA
para todo n natural.

FT

OBMEP PECI Preparao Especial para Competies Internacionais

57

3. Mostre que os princpios da induo finita e da boa ordenao so equivalentes. Dica: mostre que: PIF (1 forma)

= PIF (2 forma) = PBO = PIF (1 forma).


7.3 Divisibilidade 7.3.1 Mltiplos e divisores
Dados dois inteiros d e a, dizemos que d divide a ou que d divisor de a ou que a mltiplo de d se existir q Z tal que a = qd. No caso, denotamos d | a. Caso contrrio, d | a. Exemplo 7.3.1. 5 | 10 (tomando q = 2), mas 10 | 5. Lema 7.3.2. Sejam a, b, c, d inteiros. Temos: 1. Se d | a e d | b ento d | ax + by para todos x, y Z.

3. Se a | b e b | c ento a | c.

Demonstrao. Demonstrao: Exerccio. Dica: use a definio de ser divisvel.

Definio 7.3.3. Dados a, b Z, no ambos nulos, definimos o mximo divisor comum como ... Adivinha!? O maior nmero inteiro que divide a e b. Denotamos esse nmero por mdc(a, b) ou (a, b). Quando (a, b) = 1 dizemos que a e

b so primos entre si.

Definio 7.3.4. Analogamente, definimos mnimo mltiplo comum como o menor nmero inteiro positivo que mltiplo tanto de a quanto de b. Denotamos esse nmero por mmc(a, b).

7.3.2

O algoritmo de Euclides

Existe um mtodo para calcular o mdc de dois nmeros, que chamado Algoritmo de Euclides. Vamos apresent-lo a seguir.

Definio 7.3.5. Seja x R. Definimos o piso de x, x, como sendo o nico k Z tal que k x < k + 1 (ou seja, o maior inteiro menor ou igual a x). Definimos tambm o teto de x, x, como sendo o nico k Z tal que k 1 < x k (ou seja, o menor inteiro maior ou igual a x).

Exemplo 1

5 = 5 = 5; 3 = 3 = 3; 2, 5 = 3 e 2, 5 = 2; = 3 e = 4. Proposio 7.3.6 (diviso de Euclides). Dados a, b Z com b = 0, existem q, r Z com a = bq + r e 0 r < |b|. Alm disso, q, r so nicos. Dizemos que o q nas condies acima o quociente da diviso de a por b e r o resto. Demonstrao. Basta tomar:

RA

FT

2. Se d | a ento a = 0 ou | d || a |.

58

Teoria dos Nmeros Ferramentas Bsicas Professor Svio Ribas

a/b, se b>0 q= a/b, se b<0

r = a bq.

fcil verificar que 0 r < |b|. Alm diso, se a = bq1 + r1 = bq2 + r2 ento r2 r1 = b(q1 q2 ) mltiplo de b com |r2 r1 | < |b|, logo r2 r1 = 0 = r2 = r1 = q2 = q1 , que prova a unicidade. Lema 7.3.7 (Euclides). Se a = bq + r ento mdc(a, b) = mdc(b, r). Demonstrao. Demonstrao: Exerccio. Dica: mostre que todos os divisores comuns de a e b tambm so divisores comuns de r e todos divisores comuns de b e r tambm so divisores de a. Note que esse lema vale mesmo que no tenhamos 0 r < |b|. O algoritmo de Euclides consiste na aplicao sucessiva do lema acima, onde q e r so o quociente e o resto na diviso de a por b. Como os restos formam uma sequncia estritamente decrescente de inteiros, o algoritmo termina quando atingirmos o 0. Exemplo 7.3.8. Vamos calcular mdc(36, 28). Temos:

36 = 28 1 + 8 28 = 8 3 + 4 8=42+0
Logo, mdc(36, 28) = mdc(28, 8) = mdc(8, 4) = mdc(4, 0) = 4.

Teorema 7.3.9 (Bzout). Sejam a, b Z. Existem x, y Z com ax + by = mdc(a, b).

(pense a respeito!). Afirmamos que d divide todos os elementos de I(a, b). De fato, dado m = ax + by I(a, b), sejam

q, r o quociente e o resto na diviso euclidiana de m por d. Temos:

Como r < d e d o menor elemento positivo de I(a, b), segue que r = 0 e portanto d | m. Em particular, como a, b I(a, b) temos d | a e d | b, logo d mdc(a, b). Note ainda que se c | a e c | b ento

c | ax0 + by0 c | d. Tomando c = mdc(a, b) temos que mdc(a, b) | d. Logo, d = mdc(a, b).

Demonstrao. Exerccio.

Proposio 7.3.11. Se mdc(a, b) = 1 e a | bc ento a | c. Demonstrao. Pelo Teorema de Bzout, existem x, y Z tais que ax + by = 1 = a.cx + (bc)y = c. Notemos que a divide o lado esquerdo, logo deve dividir o direito. Corolrio 7.3.12. Sejam p um nmero primo e a1 , a2 , . . . , am Z. Se p | a1 a2 . . . am ento p | aj para algum

j, 1 j m.
Demonstrao. Exerccio.

7.3.3 O Teorema Fundamental da Aritmtica


Teorema 7.3.13. Seja n 2 um nmero natural. Podemos escrever n de forma nica como um produto n = p1 p2 . . . pm onde m N e p1 p2 . . . pm so primos.

Corolrio 7.3.10. Se c Z tal que c | a e c | b ento c | mdc(a, b).

RA

Demonstrao. Seja I(a, b) := {ax + by; x, y Z}. Seja d = ax0 + by0 o menor elemento positivo de I(a, b), que existe

r = m dq = a(x qx0 ) + b(y qy0 ) I(a, b).

FT

OBMEP PECI Preparao Especial para Competies Internacionais

59

Demonstrao. A existncia segue pelo PBO. De fato, seja A N {0, 1} o conjunto dos nmeros que no podem ser escritos como produto de primos. Note que 2 A. Suponha que A = = A possui um menor elemento n. claro que n no pode ser primo. Assim, n = ab, com 1 < a, b < n e portanto a, b A. Logo, n o produto dos primos de a pelos primos de b, absurdo pois n no possui fatorao em primos. Sendo assim, A = e todos os nmeros naturais maiores que 1 possuem fatorao em primos. Alternativamente, poderamos provar a existncia da fatorao usando a 2 forma do PIF: se n primo no h o que fazer (basta tomar m = 1 e p1 = n). Se n composto podemos escrever n = ab, a, b N, 1 < a < n, 1 < b < n. Por hiptese, a e b se decompe como produto de primos. Juntando as fatoraes de a e b e reordenando os fatores obtemos uma fatorao de n. Quanto unicidade, tambm seguiremos pelo PIF (1 forma). Suponha n = p1 . . . pm = q1 . . . qk , com p1

. . . pm e q1 . . . qk . Como p1 | q1 . . . qk temos p1 | qi para algum i e como qi primo, p1 = qi e p1 q1 .


Analogamente, q1 p1 , logo p1 = q1 . Por hiptese:

n = p2 . . . pm = q2 . . . qk p1
admite fatorao nica, logo m = k e pi = qi para todo i. Outras formas de escrever a fatorao so n = p11 p22 . . . pem com p1 p2 . . . pm e n = 2e2 3e3 5e5 . . . pep . . . m
e e

onde apenas um nmero finito de expoentes so positivos.

A partir do Teorema Fundamental da Aritmtica podemos mostrar que todo nmero inteiro positivo pode ser escrito como ab2 , onde a N livre de quadrados (isto , no existe p primo tal que p2 | a) e b N. Teorema 7.3.14. Existem infinitos primos.

Observemos que isso no prova que p1 . . . pm + 1 primo para todo conjunto finito de primos. Contra-exemplo:

2 3 5 7 11 13 + 1 = 30031 = 59 509.
plo, a sequncia:

interessante notar que existem cadeias arbitrariamente grandes de nmeros compostos consecutivos. Por exem-

possui k termos, nenhum dos quais primo.

Novamente, mdc e mmc

Podemos calcular o mdc e o mmc entre dois nmeros de outra forma:

mdc: fatoramos os dois nmeros e tomamos os fatores comuns com os menores expoentes; mmc: fatoramos os dois nmeros e tomamos os fatores que aparecem em alguma das fatoraes com os maiores
expoentes.

Exemplo 2

mmc(36, 14) = 22 32 7 = 252.


Proposio 7.3.15. 1. Se a | c ento mdc(a, b) = mdc(c, b).

2. Se mdc(a, b) = 1 ento mdc(ac, b) = mdc(c, b). 3. Se mdc(a, n) = mdc(b, n) = 1 ento mdc(ab, n) = 1. Demonstrao. Demonstrao: Exerccio. Dica: use o Teorema Fundamental da Aritmtica.

Vamos calcular mdc(36, 14) e mmc(36, 14). Temos 36 = 22 32 e 14 = 2 7. Logo, mdc(36, 14) = 2 e

RA

Demonstrao. Exerccio. Dica: suponha que existem finitos e considere o nmero (produto de todos os primos)+1.

(k + 1)! + 2, (k + 1)! + 3, (k + 1)! + 4, . . . , (k + 1)! + (k + 1)

FT

60

Teoria dos Nmeros Ferramentas Bsicas Professor Svio Ribas

7.4 Problemas Propostos Problema 6

Uma urna contm 101 bolinhas. Paulo e Robrio disputam um jogo, jogando alternadamente um de

cada vez (Paulo comea), eles retiram no mnimo 1 e no mximo 10 bolinhas da urna. Quando a urna esvaziar, eles iro contar a quantidade de bolinhas que cada um retirou da urna. Se os nmeros forem primos entre si, Paulo ganha. Caso contrrio, Robrio ganha. Quem ir vencer esse jogo e qual a estratgia vencedora?

Problema 7

Seja n = p11 . . . pem a fatorao de n em potncias de primos distintos pi . Mostre que n possui m

(e1 + 1) . . . (em + 1) divisores.


Problema 8
Sejam n = p11 . . . pem a fatorao de n em potncias de primos distintos pi e (n) a soma dos seus m
e

divisores. Mostre que:

(n) =

pe1 +1 1 pem +1 1 1 ... m . p1 1 pm 1

Problema 9

Seja p um primo e a maior potncia de p que divide n!. Mostre que:

n n n = + + + ... 2 p p p3

Problema 10 Problema 11

Determine com quantos zeros termina 2013!.

Dica: suponha m > n e fatore a2

Problema 12 Problema 13 Problema 14

Demonstre que mdc(2a 1, 2b 1) = 2mdc(a,b) 1 para todos a, b N. Mostre que existe um inteiro positivo a tal que
a29 1 a1

RA
mdc(a2 + 1, a2
m n m

Sejam m = n nmeros naturais. Demonstre que:

1.

Sejam a, b, c Z. Mostre que a equao ax + by = c tem soluo inteira em x e y se, e somente se, mdc(a, b) | c. No caso de admitir soluo, determine todas.

Problema 15

equao ax + by = c admite solues inteiras com x, y 0.

Problema 16 Problema 17 Problema 18

nmeros divisvel pelo outro.

Problema 19

perfeito. Mostre que a e b so quadrados perfeitos.

Problema 20

p1 = 2;

Sejam a, b inteiros positivos com mdc(a, b) = 1. Mostre que para todo c Z com c > ab a b a

Sejam an = 100 + n2 e dn = mdc(an , an+1 ). Calcule dn . Sejam a, b N . Mostre que mdc(a, b).mmc(a, b) = ab. Sejam m, n inteiros positivos tais que mdc(m, n) + mmc(m, n) = m + n. Prove que um dos dois

Sejam a e b nmeros inteiros positivos primos entre si. Suponha que o produto ab um quadrado

(Ibero 1987) A sequncia pn definida da seguinte forma:

FT
1, se a par, + 1) = 2, se a mpar.
tem pelo menos 2007 fatores primos distintos.

OBMEP PECI Preparao Especial para Competies Internacionais

61

Para todo n 2, pn o maior divisor primo da expresso p1 p2 p3 . . . pn1 + 1.


Demonstrar que pn diferente de 5.

Problema 21 Problema 22 Problema 23 Problema 24

(IMO 1959) Mostre que a frao

21n + 4 irredutvel para todo n natural. 14n + 3

Determine todas as solues de a2 + b2 = c2 com a, b, c Z. Determine todas as solues de a2 + b2 = 2c2 com a, b, c Z. Mostre que:

1. 215 1 e 210 + 1 so primos entre si; 2. 232 + 1 e 24 + 1 so primos entre si; 3. se m | a b ento m | ak bk para todo natural k; 4. se k natural mpar ento a + b | ak + bk .

Problema 25

Encontre todos os inteiros positivos tais que:

1. n + 1 | n3 1. 2. 2n2 + 1 | n3 + 9n 17. 3.

RA
1 99 . . . 9 1
n noves

1 1 1 + + =1 a b c

FT
( ) 1 1 1 d(k) n 1 + + + + 2 3 n

Problema 26 Problema 27 Problema 28 Problema 29

Mostre que se n um nmero natural composto ento n divisvel por um primo p com p n. (OBM 1988) Determine todos os primos que so a soma e a diferena de dois primos. (OBM 1990) Mostre que a equao x3 + 1990y3 = z4 tem infinitas solues inteiras com x, y, z > 0. (OBM 1991) Mostre que existe um nmero da forma:

com mais de dois noves que mltiplo de 1991.

Problema 30

algarismos iguais a 1.

Problema 31

D
( n

(OBM 1992) Prove que existe um natural n tal que a expanso decimal de n1992 comea com 1992

(OBM 1992) Seja d(n) o nmero de divisores positivos de n. Prove que:

1 1 1 + + + 2 3 n

n k=1

D
centro

RA
PARTE IV

Geometria
Professor Paulo Rodrigues

Assuntos do Mdulo: ngulos, Base Mdia, Medianas, BariPr-requisitos: ngulos, Congruncia de Tringulos, Paralelismo. Ser til conhecimento de contedos da apostila 8 do PIC Uma Introduo s Construes Geomtricas.

FT
.

Geometria

Problemas de Aquecimento

Problema 1 Prove que a medida da mediana AM do


tringulo ABC maior que (AB + AC BC)/2.

A N

Problema 2 DEFG um quadrado no exterior do pen tgono regular ABCDE. Quanto mede o ngulo EAF?

P
.

B F G E A
.

Problema 3 Construmos dois tringulos equilateros:

ABE interno e BFC externo ao quadrado ABCD. Prove que os pontos D, E e F se localizam na mesma reta. D E C

RA
F B
.

D
.

FT
A M B H O1 C
.

Se BN perpendicular a AP , calcular AP BN.

Problema 8 No tringulo ABC, AB = 13, BC = 14 e CA = 15. Alm disso, M o ponto mdio do lado AB e H
o p da altura relativa ao lado BC. Determine as medidas

dos segmentos HM, AH, BH e CH.

Problema 4 Demonstre que, num quadriltero qualquer, os pontos mdios dos lados so vrtices de um paralelogramo.

Problema 9 As circunferncias C1 e C2 so tangentes a


reta nos pontos A e B e tangentes entre si no ponto C. Prove que o tringulo ABC retngulo.

Problema 5 O ponto O a interseco das diagonais


do trapzio ABCD. So conhecidas as seguintes reas:

O2

[AOB] = 2 e [COD] = 8. Determine a rea do trapzio.


Problema 6 Os catetos de um tringulo retngulo me dem 7 2 e 42 2. Calcule o comprimento da bissetriz do
ngulo reto.

Problema 10 Um quadrado ABCD possui lado 40cm. Problema 7 ABCD um retngulo, AD = 5 e CD = 3.


63 Uma circunferncia contm os vrtices A e B e tangente

64

Geometria Problemas de Aquecimento Professor Paulo Rodrigues

ao lado CD. Determine a medida do raio desta circunferncia. .

E B C

A A B

Qual a medida de CD? .

Problema 14 Os vrtices de um tringulo de lados 3,

4 e 5 so centros de trs crculos mutuamente tangentes,


como mostra a figura.Calcule a soma das reas dos trs crculos.

Problema 11 A figura abaixo formada por arcos de


circunferncias. Os centros destas so os vrtices de um quadrado e todos os arcos passam pelo centro do quadrado. Calcule a rea sombreada, sabendo que o lado do quadrado mede 1 cm.

A 3
.

4 5 C

RA
B 2a A

Problema 12 Uma circunferncia de raio r est inscrita


do setor 2a.

em um setor circular de raio R. O comprimento da corda

FT
B PB = 6 e APB = BPC = CPA. A P
.

Problema 15 O tringulo retngulo ABC tem hipotenusa AC e contm um ponto P para o qual PA = 10,

Determine a medida do segmento PC.

D
.

Problema 16 Como mostra a figura abaixo o ABC


est dividido em seis tringulos por retas que passam pelo vrtice e por um ponto comum no interior do tringulo. As reas de quatro destes tringulos esto indicadas. Determine a rea do ABC.

Prove que

1 1 1 = + . r R a
84

Problema 13 As circunferncias de centros A e B tem


raios 3 e 8, respectivamente. Uma tangente comum interna interscta as circunferncias em C e D, respectivamente. As retas AB e CD intersectam-se em E e AE = 5.

N
35 40

M
.
30

Geometria

Bases Mdias, Medianas e Tesouros

9.1 A Base Mdia


Nesta seo vamos resolver problemas utilizando uma elemento muito simples da geometria elementar: a base mdia. Em um tringulo ABC, considere os pontos mdios M, N e P dos lados BC, CA e AB, respectivamente. O tringulo tem trs bases mdias, obtidas ligando os pontos mdios de dois lados. Por exemplo, uma das bases mdias o segmento NP. Diremos que esta base mdia relativa ao lado BC. Analogamente, MN e P so as bases mdias relativas aos lados AB e CA, respectivamente.

RA
B M C N M B
65

Cada base mdia paralela ao lado que corresponde e tem comprimento igual a metade do comprimento do lado. Uma consequncia deste resultado o Teorema de Varignon.

Teorema 9.1.1. Os pontos mdios dos lados de um quadriltero so vrtices de um paralelogramo. Este resultado vlido mesmo em um quadriltero no-convexo ou at mesmo entrecruzado.

D
P D Q
.

FT
N C C P D N Q
.

Demonstrao. Seja ABCD um quadriltero convexo e M, N, P e Q os pontos mdios dos lados AB, BC, CD e DA, respectivamente. Vamos provar que MNPQ um paralelogramo. Considerando o tringulo ABC, o segmento MN base mdia relativa ao lado AC, sendo paralelo ao mesmo e medindo a metade de AC. Analogamente, olhando para o tringulo CDA, o segmento PQ base mdia relativa ao lado AC, e portanto paralelo a AC e mede a metade de AC. Segue que os segmentos MN e PQ so iguais e paralelos, mostrando que o quadriltero MNPQ um paralelogramo.

66

Geometria Bases Mdias, Medianas e Tesouros Professor Paulo Rodrigues Os demais casos so anlogos e deixados para o leitor.

Problema 1 Prove que a rea de MNPQ a metade da rea de ABCD.


Definio 9.1.2. Dado um trapzio ABCD de bases AB e CD, a base mdia do trapzio o segmento de reta determinado pelos pontos mdios dos lados BC e AD. Teorema 9.1.3. A medida da base mdia de um trapzio paralela s bases e tem por medida a mdia aritmtica das medidas das bases. Demonstrao. Considere um trapzio ABCD de bases AB e CD, com CD AB. Construa X sobre CD de tal modo que AB = DX. Ento o quadriltero ABXD possui dois lados opostos iguais e paralelos (AB = DX e AB DX), sendo portanto um paralelogramo. Desse modo, AD = BX e AD BX.

A .

Sejam M e N os pontos mdios de BC e AD. Marque tambm o ponto P , mdio de BX. O quadriltero NDXP um paralelogramo porque ND = PX e ND PX. Por outro lado, PM base mdia do tringulo BXC, donde PM mede a metade de XC e paralelo CD. Desse modo podemos concluir que N, P e M so colineares e

MN = MP + PN =

9.2 O Primeiro Tesouro

Exemplo 1

R1 , e depois a mesma distncia e na mesma direo at o ponto X. Ele fez o mesmo em relao a entrada da caverna C e em relao rocha R2 , alcanando os pontos Y e Z, respectivamente. Ele enterrou o tesouro em T , ponto mdio de AZ. Y C X R2 R1 .

Um pirata resolveu enterrar um tesouro em uma ilha. Para tal, ele caminhou da rvore A para a rocha

RA
A T

XC CD DX CD + DX AB + CD + DX = + DX = = . 2 2 2 2

Ao voltar ilha para desenterrar o tesouro, o pirata encontrou as rochas e a caverna, mas no encontrou a rvore. Como o pirata pode descobrir o tesouro?

Soluo: A chave para o pirata encontrar o tesouro est no fato de que em todo quadriltero, os pontos mdios dos
lados so vrtices de um paralelogramo.

FT
X C Z

OBMEP PECI Preparao Especial para Competies Internacionais

67

Isto significa que a posio T do tesouro independe da posio da rvore. No quadriltero AXYZ, R1 , C, R2 e T so os pontos mdios dos lados. Portanto, R1 CR2 T um paralelogramo. O pirata pode comear de um ponto qualquer e repetir os procedimentos, ou pode determinar T traando uma reta paralela a R1 C por R2 e uma paralela a CR2 por R1 . O ponto de interseo das paralelas o ponto T , localizao do tesouro.

9.3 Outro Tesouro Perdido

Exemplo 2

Um pirata encontrou uma mapa acompanhado de instrues para localizar um tesouro em uma ilha

deserta. As instrues eram as seguintes: (1) Andar da palmeira P at a rocha R, girar 90 direita e caminhar a mesma distncia palmeira-rocha, chegando a um ponto no qual deve-se fazer uma marca M1 . (2) Voltar a palmeira, caminhar desta caverna C e girar 90 esquerda. Caminhar a mesma distncia palmeira-

(3) O tesouro est enterrado no ponto mdio das duas marcas.

RA
R P
encontrar o tesouro?

M1

Chegando, ilha, o pirata conseguiu localizar a rocha e a caverna, mas no viu palmeira alguma. Como ele pode

Soluo: Na resoluo de problemas de geometria plana o comeo ideal a construo de uma boa figura. Fixadas
as posies da rocha e da caverna, variamos a posio da palmeira, nos pontos P e P1.

M3 M1 T

R P

FT
M2 T C M2 M4 P1 C

caverna, chegando a um ponto no qual faz-se uma segunda marca M2 .

68

Geometria Bases Mdias, Medianas e Tesouros Professor Paulo Rodrigues Esta construo leva a seguinte conjectura: a posio do tesouro independe da localizao de P . Se isto for verdade,

o pirata facilmente poder refazer o procedimento escolhendo arbitrariamente a posio da palmeira. Vamos provar algo mais forte: o tringulo determinado pelas posies da rocha, da caverna e do tesouro retngulo e issceles, o que fixa a posio do tesouro.

M2 T M1 Y X R C P
Sejam X o ponto mdio de M1 P e Y o ponto mdio de PM2 . O segmento TY base mdia do tringulo M2 PM1 e do tringulo retngulo M1 RP . Portanto, TY = RX.

Analogamente, considerando o lado M2 P do tringulo M1 PM2 , conclumos que XT = YC. .

Alm disso, os ngulos T XR e CYT so congruentes, pois T XM1 = Y PX = M2 YT e M1 XR = 90 = CYM2 .


Segue que os tringulos TXR e CYT so congruentes pelo caso de congruncia lado-ngulo-lado e ento TR = CT . Portanto, provamos que o tringulo TRC issceles de base RC. Provemos que tambm retngulo:

RT C = RT X + XT Y + Y T C = = RT X + XPY + XRT =

pois TYPX um paralelogramo e Y T C = XRT pela congruncia provada anteriormente.


9.4

RA

= RT X + M1 XT + XRT = 180 M1 XR = 90 ,

Problemas Propostos

Problema 2 (Torneio das Cidades - 1983) Construa um quadriltero, dadas as medidas de seus lados e a medida do
segmento que une os pontos mdios das diagonais.

Problema 3

congruentes. As perpendiculares de P a BC e CA intersectam estes lados em L e N respectivamente, e D o ponto mdio de AB. Prove que DL = DM.

Problema 4 (OBM 2002 Nvel 3 2 Fase) Em um quadriltero convexo ABCD, os lados opostos AD e BC so
congruentes e os pontos mdios das diagonais AC e BD so distintos. Prove que a reta determinada pelos pontos mdios das diagonais forma ngulos iguais com AD e BC.

Problema 5

45 . Demonstrar que os pontos mdios de seus lados so vrtices de um quadrado.


Problema 6 ADB e AEC so tringulos retngulos issceles (AB e AC como hipotenusas) construdos externamente
sobre os lados AB e AC do tringulo ABC; F o ponto mdio de BC. Prove que DEF um tringulo retngulo issceles.

Problema 7

retas que unem os pontos mdios dos lados opostos do hexgono so concorrentes.

(Austrlia 1983) P um ponto no interior do tringulo ABC tal que os ngulos PAC e P BC so

(Torneio das Cidades 1994) Trs ngulos de um quadriltero no convexo e no entrecruzado medem

Desao! Seja ABCDEF um hexgono convexo no qual AC = DF, CE = FB e EA = BD. Prove que as

FT

ento TY mede a metade de M1 P . O segmento RX tambm mede a metade de M1 P pois mediana relativa a hipotenusa

OBMEP PECI Preparao Especial para Competies Internacionais

69

9.5 Medianas
Definio 9.5.1. Chamamos de mediana de um tringulo um segmento que une um vrtice ao ponto mdio do lado oposto.

Um tringulo possui ento trs medianas, cada uma partindo de um vrtice.

FT
N P
.

Teorema 9.5.2. As trs medianas de um tringulo contm um ponto comum, chamado de baricentro do tringulo.

D
.

RA
A P G B M A X G Y
.

Demonstrao. Trace as medianas AM e BN no tringulo ABC. Seja G o ponto de interseo destas medianas. Trace a base mdia MN do tringulo ABC e a base mdia XY do tringulo ABG, com X em AG e Y em BG.

70

Geometria Bases Mdias, Medianas e Tesouros Professor Paulo Rodrigues Sabemos que MN base mdia do tringulo ABC e ento a reta MN paralela reta AB e o segmento MN tem

comprimento igual metade da medida do segmento AB. Por outro lado, o segmento XY base mdia do tringulo ABG e portanto, a reta XY paralela reta AB e o segmento XY tem por medida a metade da medida do segmento AB. Portanto, as retas XY e MN so paralelas e os segmentos XY e MN possuem o mesmo comprimento.

Pelo paralelismo, os ngulos Y XG e GMN so iguais (alternos internos). Pelo mesmo motivo tambm so iguais os ngulos GYX e GNM. A

X N G

Segue que os tringulos GXY e GMN so congruentes pelo caso de congruncia ALA. Portanto, XG = MG e

YG = NG. Portanto, como sabemos que AX = XG, podemos concluir que AX = XG = GM e ento o ponto G divide a mediana AM na razo 2 : 1. Analogamente, este ponto divide a mediana BN na razo de 2 : 1.
De modo anlogo, podemos provar que o ponto de interseo das medianas AM e CP divide cada uma destas na razo 2 : 1. Assim, o ponto G comum as trs medianas e este ponto chamado de baricentro do tringulo ABC.

9.6

Mediana em um tringulo retngulo


Um resultado muito til o seguinte:

Teorema 9.6.1. Em todo tringulo retngulo, a medida da mediana relativa a hipotenusa tem comprimento igual a

Demonstrao. 1 Formando tringulos issceles.

D
C X
.

metade da medida da hipotenusa.

RA
B

FT
M C C
90

X
90

Considere o tringulo ABC, retngulo em A. Construa o ponto X sobre CB de tal modo que AX = XB. Seja a

medida do ngulo XBA. Ento, BAX = , XAC = 90 (porque o ngulo A reto) e ACX = 90 (lembre-se que ABC retngulo).
Portanto, AX = CX e como por construo, AX = XB, temos que AX = BX = CX e portanto, X o ponto mdio de CB e a mediana AX mede a metade da hipotenusa BC.

OBMEP PECI Preparao Especial para Competies Internacionais Demonstrao. 2 Completando o retngulo.

71

Dado o tringulo ABC retngulo em A construa o ponto D de tal modo a obter o retngulo ABDC. Construa a diagonal AD.

Como um retngulo um paralelogramo, o ponto de interseo das diagonais o ponto mdio de ambas. Deste modo, AM = MD e BM = MC. Como as diagonais de um retngulo so iguais, temos que AM = MC = MB = MD e portanto, M o ponto mdio de BC e a medida da mediana AM igual a metade da medida da hipotenusa BC.

Problema 8

(OCM) Sejam ABC um tringulo qualquer e G o ponto de encontro das suas medianas. Veja que uma

reta r qualquer que passe pelo ponto G, excetuando-se as medianas, separa um dos vrtices do tringulo, por exemplo

A, dos outros dois B e C. Prove que a soma das distncias de B e C reta igual a distncia de A reta r.
Soluo:
A situao do problema est representada na figura abaixo:

RA
Y G X
.

Sejam X, Y e Z os ps das perpendiculares traados dos pontos A, B e C com relao a reta r. Devemos provar Observe que YBCZ um trapzio de bases BY e CZ (pois estes so perpendiculares a reta r). Sendo M o ponto mdio de BC, traamos a base mdia do trapzio MT (veja figura abaixo).

que AX = BY + CZ.

Y X G
.

MT perpendicular a r, pois paralelo a cada uma das bases BY e CZ. Como MT a base mdia do trapzio, sabemos que 2MT = BY + CZ. Basta ento provar que 2MT = AX. Observe os tringulos AXG e MTG. So semelhantes, pois as retas AX e MT so paralelas. Lembrando que o
baricentro G divide a mediana em segmentos cuja razo de 2 : 1, obtemos:

AG AX = = 2 = AX = 2TM. TM GM

FT
Z C T Z M C

72 Segue que AX = 2TM = BY + CZ.

Geometria Bases Mdias, Medianas e Tesouros Professor Paulo Rodrigues

9.7 Problemas Propostos Problema 9


Sobre o lado BC do tringulo ABC o ponto A1 marcado de tal modo que BA1 : A1 C = 2 : 1. Qual

a razo em que a mediana CP divide o segmento AA1 ?

Problema 10

Trace as medianas AM, BN e CP do tringulo ABC de baricentro G. Prove que os seis tringulos

AGP, PGB, BGM, MGC, CGN e NGA possuem reas iguais.


Problema 11
No tringulo ABC traamos as medianas AM, BN e CP . Por M traamos MD paralelo e igual a BN

de modo que A e D estejam no mesmo semiplano determinado pela reta BC. Prove que AD = CP .

Problema 12

As medianas AM e BN do tringulo ABC so perpendiculares e medem 12 cm e 9 cm, respectiva-

mente. Determine a medida do lado AB.

Problema 13 Problema 14
que

Construa um tringulo dadas as medidas das trs medianas. Considere um tringulo equiltero ABC; escolha sobre os lados AB, BC e AC pontos A1 , B1 , C1 tais

AA1 = BB1 = CC1 =


(a) Prove que o tringulo A1 B1 C1 equiltero.

(b) Prove que cada uma das retas A1 B1 , B1 C1 e C1 A1 perpendicular a um dos lados do tringulo.

RA

FT
1 AB. 3

Geometria

10

Problemas Extras

Problema 1

Considere dois pontos A e B do mesmo

lado de uma reta r. Determine um ponto P sobre a reta de modo que a soma das distncias AP + PB seja a menor possvel.

Problema 2

Mostre como (em que posio) devemos

B A

construir uma ponte sobre um rio de modo que uma pessoa que saia de A e caminhe at o ponto B percorra o menor caminho possvel. (A ponte deve ser perpendicular s margens do rio - veja a figura abaixo).

r
Soluo:

Observe que se a reta r separasse os pontos

A e B no haveria o que fazer. O menor caminho seria o

segmento de reta AB e a o ponto P seria a interseco de neste outro do seguinte modo:

AB com a reta r. Podemos transformar o problema dado B

RA
P1 B1
73

Seja B1 o simtrico de B em relao a reta r e P1 a interseco de AB1 e r. Afirmamos que o caminho AP1 B o menor possvel. De fato, sendo P um ponto qualquer de r diferente de

P1 , temos AP + PB = AP + PB1 > AB1 = AP1 + P1 B1 = AP1 + P1 B.

FT
A
.

Problema 3

(Canad - 91) Seja C um crculo e P um

ponto do plano. Cada reta atravs de P que intersecta C determina uma corda em C . Mostre que os pontos mdios destas cordas esto sobre uma circunferncia.

Problema 4

(Olimpada do Cone Sul 92) Seja P um

ponto fora de uma circunferncia C. Determinar dois pontos Q e R na circunferncia tais que P , Q, R esto alinmero de solues)

nhados e Q o ponto mdio do segmento PR. (discutir o

Problema 5

Um mosca est pousada em um dos vr-

tices de um cubo de madeira. Qual o menor caminho que ela pode seguir para chegar ao vrtice oposto?

Problema 6

Seja P um ponto no interior de um trin-

gulo equiltero ABC. Prove que a soma das distncias de

P aos lados de ABC constante, isto , no depende da


localizao de P .

D
PARTE V

Sesses de Problemas
Professor Paulo Rodrigues

RA

FT

Problemas

11

Sesso 1

1. 2.

A soma de treze inteiros positivos distintos igual a

Determine a medida do maior lado da folha, sabendo que o lado mais curto mede 8 cm.

92. Determine estes nmeros.


Mostre como formar um quadrado utilizando quatro

4.

Pedro nasceu no sculo 19, enquanto seu irmo Paulo

figuras idnticas mostrada na figura 2.1.

nasceu no sculo 20. Certa vez os irmos se encontraram em uma festa comemorando o aniversrio de ambos. Pedro disse, Minha idade igual soma dos dgitos do meu ano de nascimento. A minha tambm, respondeu Paulo. Quantos anos Paulo mais jovem que Pedro?

.
Figura 2.1

3.

Cristiane dobrou uma folha retangular de papel de tal

modo que um vrtice coincidiu com o ponto mdio de um lado, como indicado na figura 3.1. Ela descobriu que os tringulos I e II so iguais.

II

6. Seguem alguns exemplos nos quais a soma dos quadrados de k nmeros positivos consecutivos igual soma dos
quadrados dos k 1 inteiros seguintes:

D
Figura 3.1

Encontre uma frmula geral para todos os casos.

RA
32 + 42 = 52 ,
75

362 + 372 + 382 + 392 + 402 = 412 + 422 + 432 + 442 , 552 + 562 + 572 + 582 + 592 + 602 = 612 + 622 + 632 + 642 + 652 .

FT
satisfazem o sistema

5. Encontre todas as ternas (x, y, z) de nmeros reais que


x(x + y + z) = 26 y(x + y + z) = 27 z(x + y + z) = 28.

Problemas

12

Sesso 2

7. Pinte de preto seis casas de um tabuleiro branco 6 6,


de tal modo que no seja possvel cortar um retngulo branco 1 6 ou um quadrado branco 3 3.

11.

O tringulo retngulo ABC tem ngulo reto em C e

o ngulo A mede 30 . O centro da circunferncia inscrita no tringulo ponto I e D o ponto de interseo desta circunferncia com o segmento BI. Prove que os segmentos AI e CD so perpendiculares.

8.

Temos 9 moedas, uma das quais falsa (ela mais

leve do que as outras). Encontre a moeda falsa utilizando duas pesagens em uma balana de pratos.

12. No quadrado ABCD, a linha poligonal KLAMN tal

a seguinte regra: o primeiro nmero 7 e cada nmero,

a partir do segundo, igual a soma dos dgitos do quadrado do nmero anterior, aumentada em uma unidade. Por exemplo, o segundo nmero 14, porque 72 = 49 e Qual o milsimo nmero da sequncia?

FT
B L K
45
.

9.

Uma sequncia numrica formada de acordo com

que os ngulos KLA, LAM e AMN medem 45 .

4 + 9 + 1 = 14. O terceiro nmero 17 e assim por diante.

Escreva um dos nmeros de 1 a 12 em cada um dos

doze tringulos equilteros pequenos da figura de modo

RA
Figura 10.1

10.

A
Figura 12.1

N D

que, em cada tringulo equiltero formado por quatro tringulos pequenos, a soma dos nmeros escritos seja igual a 20.

Demonstre que KL2 + AM2 = AL2 + MN2 .

D
. 76

Problemas

13

Sesso 3

13.

O rei pretende construir seis castelos em seu reino

16.

O treinador da equipe de natao decidiu organi-

e ligar dois quaisquer deles por uma estrada. Faa um diagrama dos castelos e das estradas de modo que elas se cruzem ao todo trs vezes e exatamente duas estradas passem em cada cruzamento.

zar uma srie de competies entre os 7 integrantes da equipe. Em cada dia ser realizado uma nica prova com a participao de trs nadadores. Cada nadador competir exatamente uma vez com cada um dos outros. (a) Quantos dias durar esta srie de competies? Explique ou justifique por que no pode durar nem mais dias, nem menos dias que o nmero afirmado.

14. A soma dos algarismos de um nmero igual a 2010.


Este nmero pode ser um quadrado perfeito?

15.

Sete estudantes contam de 1 a 1000 como segue:

Andr diz todos os nmeros, com exceo do nmero


do meio em cada grupo de trs nmeros consecutivos. Isto , Andr diz 1, 3, 4, 6, 7, 9, . . . , 997, 999, 1000.

Bruno diz todos os nmeros que Andr no disse, exgrupo de trs nmeros consecutivos.

ceto que ele tambm salta o nmero do meio em cada

RA
77

FT
B M
45
.

(b) Mostre uma possvel distribuio indicando os trs nadadores que competem em cada dia.

17. Considere os pontos M e N sobre os lados BC e CD


do quadrado ABCD, tais que o ngulo MAN mede 45 . C

Clara diz todos os nmeros que Andr e Bruno no disseram, exceto que ele tambm salta o nmero do meio em cada grupo de trs nmeros consecutivos.

Daniel, Estevo e Fbio dizem todos os nmeros que

nenhum dos estudantes com o primeiro nome come-

A
Figura 17.1

eles tambm saltam o nmero do meio em cada grupos de trs nmeros consecutivos. Prove que a diagonal BD divide o tringulo AMN em duas partes de mesma rea.

Finalmente, Gabriel diz o nico nmero que ningum


disse.

ando antes do seu no alfabeto disseram, exceto que

18. Existe um hexgono que pode ser dividido em quatro


tringulos congruentes por um nico corte reto?

Que nmero Gabriel disse?

Problemas

14

Sesso 4

19.

O campo do jogo Batalha Naval um tabuleiro

quantas moedas ele ganhar. Observao: cada pirata escolhido deve receber pelo menos uma moeda.

10 10, o qual contm um navio oculto no formato de um retngulo 13. sempre possvel acertar o navio com
at 33 tentativas?

22. Voc possui 6 moedas de pesos 1, 2, 3, 4, 5 e 6 gramas


que parecem iguais, exceto por seus rtulos que indicam o respectivo peso de cada uma. Como determinar se todas as indicaes dos rtulos esto corretas, usando uma balana de pratos somente duas vezes?

20.

O primeiro termo de uma sequncia 439 e cada

termo, a partir do segundo, igual soma dos algarismos do termo anterior, multiplicada por 13. Qual o 100o termo desta sequncia?

21.

A lei pirata estabelece que para repartir as moedas

de um tesouro o capito deve escolher um grupo de piratas e repartir igualmente as moedas entre estes at que no possua moedas suficientes para dar uma a mais a pito.

O capito Morgan deve repartir um tesouro que contm menos de 1000 moedas de ouro. Ele sabe que se escolhe 99 piratas ficar com 51 moedas e se escolhe 77 piratas cabero a ele apenas 29 moedas. Determinar quan-

tos piratas deve escolher Morgan para ficar com a maior

quantidade de moedas, e para essa quantidade de piratas,

D
78

RA
svel.

cada pirata. As moedas que sobram so a parte do ca-

FT
24.
ponto I. Prove que ID = IE.

23. Duas pessoas realizam um truque. A primeira retira


5 cartas de um baralho de 52 cartas (previamente embaralhado por um membro da plateia), olha-as, e coloca-as em uma linha da esquerda para a direita: uma com a face para baixo (no necessariamente a primeira), e as outras com a face para cima. A segunda pessoa deve adivinhar a carta que est com a face para baixo. Prove que elas podem combinar um sistema que sempre torna isto pos-

No tringulo ABC, o ngulo B mede 60 . Traamos

as bissetrizes AD e CE, sendo D um ponto do lado BC e

E um ponto do lado AB. As bissetrizes intersectam-se no

Problemas

15

Sesso 5

25. Um polgono legal se seus vrtices esto sobre uma


grade retangular de pontos e cada um de seus lados horizontal ou vertical. A distncia entre dois pontos vizinhos da grade 1 cm. Por exemplo, o polgono da figura seguinte legal.

A B C D

Figura 25.1

(a) Existe um polgono legal com permetro igual a 22 cm e rea igual a 14 cm2 ? Em caso afirmativo, mostre um exemplo e caso contrrio justifique.

(b) Existe um polgono legal com permetro igual a 21 cm e rea igual a 14 cm2 ? Em caso afirmativo, mostre um exemplo e caso contrrio justifique.

26.

(a) Existem dois nmeros naturais consecutivos tais que as somas de seus algarismos so ambas divisveis por

7?

(b) Existem dois nmeros naturais consecutivos tais que as somas de seus algarismos so ambas divisveis por

9?
Em ambos os casos, se a resposta for afirmativa, d um exemplo. Se a resposta for negativa, justifique.

27.

O ponto K marcado sobre a hipotenusa AB do

tringulo retngulo ABC de modo que CK = BC. O segmento CK divide a bissetriz interna AL em dois segmentos de mesma medida (L um ponto do lado BC). Determine as medidas dos ngulos do tringulo ABC.

28.

Em uma folha quadriculada marcamos os pontos A,

B, C, D, M e N, como mostra a figura 28.1.


79

RA
29. 30.

FT
.

Figura 28.1

Prove que a soma dos ngulos MAN, MBN, MCN e MDN igual a 45 .

Um pas tem 12 ministros. Cada ministro amigo

de 5 ministros e inimigo dos outros 6. Cada comit formado por 3 ministros. Um comit considerado legtimo se todos os seus membros so amigos ou se todos so inimigos. Quantos comits legtimos podem ser formados?

Uma professora de matemtica pensou em um in-

teiro positivo de dois algarismos. Ela deseja que seus dois inteligentes alunos Daniela e Adriano determinem o valor exato do nmero pensado. Para tal, informa reservadamente a Daniela a quantidade de divisores positivos do nmero e confidencia a Adriano a soma dos algarismos do nmero. Uma breve conversa entre Daniela e Adriano transcrita abaixo:

Adriano: Eu no posso determinar o nmero. Daniela: Nem eu, mas posso dizer se ele par ou mpar. Adriano: Agora eu sei qual o nmero. Daniela: Voc sabe? Ento eu tambm sei.
Suponha que os estudantes so honestos e existe lgica perfeita em tudo o que falaram. Determine o nmero pensado pela professora justificando sua resposta.

D
PARTE VI

Avaliaes e Simulados
Professor Paulo Rodrigues

RA

FT

Problemas

16

Prova de Seleo 2013

Problema 1 Andr possui n pesos cujas massas em gramas so dadas pelos nmeros 1, 2, . . . , n.
(a) possvel que Andr consiga dividir estes pesos em trs grupos de mesma massa se n = 2011? (b) possvel que Andr consiga dividir estes pesos em trs grupos de mesma massa se n = 2010?

Problema 2 Fbio escreveu um nmero inteiro em cada uma das quarenta e nove casas de um tabuleiro 7 7. Ele
Prove que a soma dos nmeros escritos por Fbio nas 24 casas que formam o bordo do tabuleiro tambm igual a 0.

Problema 3 Prove que a equao a2 + b2 = c2 + 3 possui infinitas solues com a, b e c inteiros positivos. Problema 4 Os pontos A, B, C, D e E so vrtices consecutivos de um polgono regular de 18 lados inscrito em uma
circunferncia de centro O. O ponto P est sobre a semirreta OA de tal modo que o tringulo OPB issceles de base

D
(b) Prove que PB = BE.

(a) Determine a medida do ngulo CEB.

(c) Determine as medidas dos ngulos do tringulo PBC.

Problema 5 Em uma fila existem 30 crianas marcadas com os nmeros 1, 2, . . . , 30 da esquerda para a direita. Para
cada criana marcada com o nmero i tal que 2 i 15, a quantidade de amigos com nmero maior que i igual a 1 81

RA
P A B
.O

OP.

D E

FT

fez isso de modo que, em cada quadrado 2 2 e em cada quadrado 3 3, a soma dos nmeros escritos foi igual a 0.

82

Avaliaes e Simulados Prova de Seleo 2013 Professor Paulo Rodrigues

mais a quantidade de amigos com nmero menor que i. Para cada criana marcada com nmero i tal que 16 i 29, a quantidade de amigos com nmero menor que i igual a 2 mais a quantidade de amigos com nmero maior que i. A criana 1 tem 19 amigos. Quantos amigos possui a criana 30?

Problema 6 Colocamos 15 cavalos em um tabuleiro 15 15 de modo que no existam dois cavalos em uma mesma
linha ou em uma mesma coluna. Cada cavalo faz ento um movimento. Prove que agora existem dois cavalos na mesma linha ou na mesma coluna.
Observao: O cavalo movimenta-se em L. Assim, um cavalo na casa marcada com pode mover-se para as casas destacadas.

RA

FT

Problemas

17

Avaliao 1

Avaliao aplicada no primeiro encontro presencial no dia 13/01 com durao de 4 horas e 30 minutos.
17.1 lgebra Problema 1 Problema 2
(a) x2 + (b) x3 + Determine m para que as razes da equao x2 3x + 2m = 0 sejam reais e iguais.

1 x2 1 x3

Problema 3

Resolver, em R, o sistema de equaes abaixo.

Combinatria Problema 4

17.2

Dados 50 pontos arbitrrios dentro de um quadrado de lado 1, mostre que existem dois desses pontos

cuja distncia entre eles menor que

Problema 5

Em um tabuleiro de xadrez 88 so retirados os cantos superior esquerdo e o inferior direito, obtendo-

se um tabuleiro truncado. Explique se possvel um cavalo, partindo de uma casa qualquer, percorrer todas as casas do tabuleiro truncado passando por cada casa uma nica vez.

Problema 6 17.3 Geometria Problema 7

Prove que dentre 52 inteiros possvel escolher dois cuja soma ou diferena divisvel por 100.

A figura mostra parte de um polgono regular de 20 lados (icosgono) ABCDEF..., um quadrado BCYZ

e um pentgono regular DEVWX. 83

RA
2 . 9

y2 + u2 + v2 + w2 = 4x 1 2 x + u2 + v2 + w2 = 4y 1 x2 + y2 + v2 + w2 = 4u 1 2 x + y2 + u2 + w2 = 4v 1 2 x + y2 + u2 + v2 = 4w 1

FT

Se x um nmero real tal que x +

1 = k, determine, em funo de k x

84

Avaliaes e Simulados Avaliao 1 Professor Paulo Rodrigues

C D X

F W (a) Determine a medida do ngulo Y DC.


(b) Mostre que o vrtice X est sobre a reta DY .

Problema 8

Na figura, ABCD e CEFG so quadrados e o lado do quadrado CEFG mede 12 cm.

Quais so os possveis valores da rea do tringulo AEG?

Problema 9

Mostre que possvel construir um pentgono com todos os lados de mesma medida e cujos ngulos

internos meam 60 , 80 , 100 , 140 e 160 , em alguma ordem.

17.4 Nmeros Problema 10

Existe um nmero natural tal que o produto de seus algarismos igual a 1980?

Problema 11 Determine todos os inteiros positivos n tais que 3n 4, 4n 5 e 5n 3 so todos primos. Problema 12
perfeito?

Um nmero escrito com cem algarismos 0, cem algarismo 1 e cem algarismos 2 pode ser um quadrado

RA

FT
C E

Problemas

18

Simulado 1

Simulado aplicado no primeiro encontro presencial no dia 16/01 com durao de 4 horas e 30 minutos.
Problema 1
(a) Simplifique a expresso

x2 (x + 1)2 (x + 2)2 + (x + 3)2 .


(b) Prove que, para todo inteiro positivo n, existe um inteiro positivo k e uma escolha adequada de sinais + e para

n = 12 22 k2 .
(por exemplo, 12 = 12 + 22 + 32 ).

Problema 2

No tringulo ABC com AB = BC, traamos a bissetriz BD, com D sobre AC. Marcamos o ponto E

sobre BD, diferente de D, de tal modo que CE = CD. Prove que o ponto mdio de DE est sobre a base mdia de

ABC, paralela ao lado AB. B

RA
E
.

Problema 3

com cada um dos outros n 1 jogadores, e no mais que uma partida por dia. Determine o menor nmero de dias necessrios para terminar o torneio. Mostre como poderia ser realizado o torneio com a quantidade de dias encontrada.

Existem n jogadores participando de um torneio de xadrez. Cada jogador deve disputar uma partida

FT
D A
85

a qual

Problemas

19

Simulado 2

Simulado aplicado no primeiro encontro presencial no dia 16/01 com durao de 4 horas e 30 minutos.
Problema 1
Calcule a rea do tringulo ABC abaixo, dados BD = 4, DE = 2, EC = 6, BF = FC = 3.

F D B

.E

Problema 2

Esto alinhados N gafanhotos (N 3). A cada segundo, um dos gafanhotos (exceto os dois primeiros)

pula e passa frente dos dois imediatamente sua frente. Aps K segundos, os N gafanhotos esto novamente na

Problema 3

Em Terra Brasilis existem n casas, onde vivem n duendes, cada um em uma casa. Nesse pas temos

estradas de mo nica satisfazendo as seguintes condies: (i) Cada estrada une duas casas.

(ii) Em cada casa comea exatamente uma estrada.

(iii) Em cada casa termina exatamente uma estrada.

diz que, quando todos os duendes regressarem s suas casas originais, o mundo acabar. (a) Mostre que o mundo acabar.

(b) Se n = 98, mostre que possvel que os duendes construam e orientem as estradas de modo que o mundo no acabe antes de 300 000 anos.

Todos os dias, a partir do dia 1, cada duende sai da casa onde est e chega casa vizinha. Uma lenda de Terra Brasilis

RA
86

ordem original. Quais os possveis valores de K?

FT
A

Solues

20

Gabarito da Prova de Seleo

Problema 1 Andr possui n pesos cujas massas em gramas so dadas pelos nmeros 1, 2, . . . , n.
(a) possvel que Andr consiga dividir estes pesos em trs grupos de mesma massa se n = 2011? (b) possvel que Andr consiga dividir estes pesos em trs grupos de mesma massa se n = 2010?

Soluo:

de Thiago Lucas Faustino da Silva de Itumbiara Gois.

(a) No, para que Andr consiga dividir os pesos em trs grupos de mesma massa, a soma da massa de todos os pesos deveria ser um mltiplo de 3, porm, se n = 2011, a soma das massas no ser um mltiplo de 3. Veja:

S=

(1 + 2011) 2011 2012 = 2011 = 1006 2011 = 2023066. 2 2

Vamos ver se este nmero mltiplo de 3 somando os algarismos:

19 no um mltiplo de 3, logo, S no mltiplo de 3, por isso impossvel dividir S em trs parcelas iguais, que
seriam os trs grupos de pesos.

(b) Sim, pois a soma das massas ser um mltiplo de 3. Veja:

Se somarmos os algarismos, teremos certeza que a soma mltiplo de 3.

que um mltiplo de 3. Assim, podemos dividir em trs grupos cuja soma das massas 673685.3 Uma maneira de dividir os pesos neste caso seria formar 335 grupos de 6 nmeros consecutivos, que inicia com um nmero da forma 6x + 1 e termina num nmero da forma 6x. Exemplo: 1 6, 7 12, 13 18. Vamos chamar o menor nmero de cada grupo (massa do peso de menor massa) de x. O grupo teria:

A partir disso, dividimos os grupos A, B e C da seguinte forma: (a) Grupo A ter nmeros da forma x e x + 5. (b) Grupo B ter nmeros da forma x + 1 e x + 4. (c) Grupo C ter nmeros da forma x + 2 e x + 3.
3

Observe que esta uma condio necessria, mas no garante que a diviso seja de fato possvel.

RA
S=
87

Sa = 2 + 2 + 3 + 6 + 6 = 19

(1 + 2010) 2010 = 1005 2011 = 2021055. 2

Sa = 2 + 2 + 1 + 5 + 5 = 15,

x, x + 1, x + 2, x + 3, x + 4, x + 5

FT

88

Avaliaes e Simulados Gabarito da Prova de Seleo Professor Paulo Rodrigues

Comentrios da Equipe do PECI Muitos alunos fizeram confuso em relao ao que uma condio necessria e
o que uma condio suficiente.

Para que seja possvel dividir em trs grupos de mesma massa necessrio que a soma 1 + 2 + + n seja divisvel
por 3. Assim, no item (a), como a soma no divisvel por 3, no possvel.

Ja no item (b), o simples fato da soma ser divisvel por 3 no implica que seja possvel fazer a diviso em trs grupos de mesma massa. Por exemplo, se n = 3, a soma 1 + 2 + 3 = 6, mltiplo de 3, mas no possvel separar em trs
grupos de mesma massa.

Existem muitas maneiras de construir o exemplo do item b. Uma maneira diferente mas que utiliza o mesmo
princpio da soluo apresentada considerando os conjuntos:

A = {1, . . . , 335} {1676, . . . 2010} B = {336, . . . , 670} {1341, . . . , 1675} C = {671, . . . , 1005} {1006, . . . 1340}

Dividimos os nmeros de 1 a 2010 em 6 blocos com a mesma quantidade de inteiros consecutivos e colocamos num mesmo grupo o primeiro e o ltimo blocos, o segundo e o penltimo blocos, o terceiro e o quarto blocos. fcil verificar que os trs grupos possuem a mesma soma.

Problema 2 Fbio escreveu um nmero inteiro em cada uma das quarenta e nove casas de um tabuleiro 7 7. Ele
fez isso de modo que, em cada quadrado 2 2 e em cada quadrado 3 3, a soma dos nmeros escritos foi igual a 0. a 0.

Soluo:

de Guilherme Goulart Kowalczuk de Porto Alegre RS.

RA
a a
.
Figura 2.2

Prove que a soma dos nmeros escritos por Fbio nas 24 casas que formam o bordo do tabuleiro tambm igual

FT
d c
.

D
.
Figura 2.1

b
Figura 2.3

Chamamos de a a soma dos seis nmeros indicados da borda do tabuleiro na figura 2.1. Tambm analisaremos com um todo, as duas partes 3 3 assinaladas no desenho, cuja soma 0. Vamos analisar a parte destacada no desenho anterior, como na figura 2.2. A soma de todos os quadradinhos da figura ao lado tem que ser 0. A soma dos quadrados 2 2 assinalados tambm tem que ser 0. Portanto, a soma dos quadrados sombreados a = 0 0, ou seja, a = 0. Da mesma forma podemos encontrar os valores de b, c e d como na figura 2.3.

Comentrios da Equipe do PECI: Existem vrias maneiras de resolver o problema. Uma observando as formas
L de quadrados 3 3 e 4 4, mostradas abaixo. Na figura 2.4, a soma dos nmeros do L3 igual a diferena entre a soma dos nmeros do tabuleiro 3 3 e a soma dos nmeros do tabuleiro 2 2 (veja a figura 2.5), ou seja, igual a zero.

OBMEP PECI Preparao Especial para Competies Internacionais

89

Na figura 2.6, a soma dos nmeros do L4 igual a diferena entre a soma dos nmeros do tabuleiro 4 4 e a soma dos nmeros do tabuleiro 3 3. Como um tabuleiro 4 4 pode ser formado por quatro tabuleiros 2 2 (figura 2.7), a soma dos nmeros igual a zero e ento a soma dos nmeros do L4 tambm igual a zero. Para concluir o problema, basta observar que o bordo do tabuleiro a unio de dois L3 e dois L4 , como mostrado na figura 2.8.

L4 L3

.
Figura 2.4

.
Figura 2.5

.
Figura 2.6

.
Figura 2.7

.
Figura 2.8

Problema 3 Prove que a equao a2 + b2 = c2 + 3 possui infinitas solues com a, b e c inteiros positivos. Soluo:
de Renan Felipe Bergamaschi de Morais de Bariri SP (Adaptada pela Banca).

O problema pode ser escrito da seguinte forma: a soma de dois quadrados perfeitos igual a um quadrado perfeito mais trs. Ento

a2 + b2 = c2 + 3 a2 + b2 c2 = 3.

Fixamos um valor positivo para a, com a > 3. Por exemplo, se a = 4, temos que a2 = 16, e 16 3 = 13. Assim a equao ficaria b2 + 13 = c2 . Para encontrar os valores de b e c, fazemos um quadro com os quadrados perfeitos

RA
12 22 32 42 52
+3 +5 +7 +9 +11

82

92

102

112

+17

+19

+21

+23

Perceba que a diferena entre dois quadrados perfeitos consecutivos sempre mpar, e aumenta de 2 em 2 a cada quadrado. Na equao b2 + 13 = c2 , b2 = c2 13 temos que a diferena entre os quadrados 13, ento esses nmeros so 62 e 72 , formando a igualdade 42 + 62 = 72 + 3. Perceba que no caso anterior a = 4, e se a = 5, a2 = 25, ento 25 3 = 22, sendo assim, no h dois quadrados perfeitos consecutivos com essa diferena. Generalizando, um (nmero mpar)2 = nmero mpar, menos 3 o resultado par, que no soluciona o problema para quadrados consecutivos. Assim a prxima escolha a = 6, e a2 3 = 33, ou seja a diferena entre os quadrados de b e c 33, ento b = 16 e c = 17, formando a igualdade 62 + 162 = 172 + 3. Como h infinitos nmeros pares, h infinitos quadrados perfeitos pares, e assim infinitas solues para esta equao.

FT
62
+13

72
+11

122

132
+27

142

+25

Comentrios da Equipe do PECI: Existem vrias formas de resolver o problema. Algebricamente, a soluo de
Renan pode ser escrita como a = 2k, para algum inteiro k. Substituindo na equao dada obtemos c2 b2 = 4k2 3. Escolhendo c = b + 1, obtemos (b + 1)2 b2 = 4k2 3, donde b = 2k2 2 e c = 2k2 1. De fato,

a2 + b2 = (2k)2 + (2k2 2)2 = 4k2 + 4k4 8k2 + 4 = 4k4 4k2 + 1 + 3 = (2k2 1)2 + 3 = c2 + 3,
e ento obtemos as infinitas solues. A tabela abaixo mostra solues com alguns valores de k:

90

Avaliaes e Simulados Gabarito da Prova de Seleo Professor Paulo Rodrigues

k 2 3 4 5 6

a 4 6 8 10 12

b 6 16 30 48 70

c 7 17 31 49 71

Nem todas as solues da equao so geradas por est frmula, como por exemplo, a soluo a = 12, b = 22 e c = 25.

Problema 4 Os pontos A, B, C, D e E so vrtices consecutivos de um polgono regular de 18 lados inscrito em uma


circunferncia de centro O. O ponto P est sobre a semirreta OA de tal modo que o tringulo OPB issceles de base

OP. (a) Determine a medida do ngulo CEB.


(b) Prove que PB = BE. (c) Determine as medidas dos ngulos do tringulo PBC.

Soluo:

de Luan Lima Freitas Rio de Janeiro RJ.

B C

Podemos dividir a circunferncia em 18 setores, ou tringulos issceles cujos lados so raios, demarcados por dois pontos consecutivos do polgono e o centro O, cada um. Como so 18 tringulos congruentes, o ngulo relativo ao vrtice O de cada um igual a 360 /18 = 20 .

b) No tringulo EOB, o ngulo O corresponde a 3 setores, ou seja, vale 3O = 60 . Como o tringulo EOB issceles (pois seu lados so raios) de base EB, EBO e BEO valem, cada um,
18060 2

RA
D E

equiltero. Como PBO issceles, PB = OB, e sendo EBO equiltero, EB = BO, logo, PB = BE.

a) Sabemos que BEO mede 60 . No DEO, O = 20 (um setor) e D = E = . I igual a DEO (DEC + BEO)

. Sabemos que DEO = 80 e BEO = 60 . Analisemos um tringulo um pouco


180160 2

diferente, DEC: D = 160 , um ngulo do polgono de 18 lados. DEC = DCE = . os valores em I ,

FT
.O

= 60 . Vemos assim que ele = 80 . O ngulo CEB

18020 2

= 10 . Substituindo

CEB = DEO (DEC + BEO) = 80 (60 + 10 ) = 10 . c) Sendo OPB issceles, P = O = 20 . Assim, B = 180 (20 + 20 ) = 140 . O ngulo CBO mede 80 , pois CBO issceles e COB = 20 . O ngulo CBP, ento medir 360 (140 + 80 ) = 140 . O tringulo EPB issceles, pois BE = PB, pelo item b). Sabemos que CEB = 10 , porm no est claro se C
est sobre o segmento PE ou no. Vamos fazer uma figura caricata:

OBMEP PECI Preparao Especial para Competies Internacionais

91

B
.

B = CBP + CBE (da figura original): logo B = 140 + CBE. CBE, por sua vez, igual a CBO EBO = 80 60 = = 10 . Bem, se CEB = 10 e PEB = 10 tambm, isso significa que CEP = 0 , ou 20 . Tambm temos que CEB seja C est sobre PE. Assim provado, fica bvio que CPB = 10 e PCB = 180 (10 + 140 ) = 30 .

FT
1 2 EBO

Comentrios da Equipe do PECI: A dificuldade do problema consistia em no usar sem provar que os pontos P, B
e E esto alinhados como sugeria a figura. Muitos alunos erraram no item (c) ao usar esta condio com base apenas

A maneira mais direta para calcular o ngulo CEB no item (a) observar que este um ngulo inscrito na circunferncia e portanto, mede a metade do arco CB.

Uma soluo alternativa para o item (c), tambm usa ngulos inscritos na circunferncia, porm no a construda no enunciado:

Como a distncia de P , O e E a B a mesma, ento podemos traar uma circunferncia com centro B e raio BO que

se deduz que P, C e E esto alinhados. Conclumos que

= 30 . Por outro lado, como o tringulo OPB issceles, = BOP = 20 , segue que PEB = EPB = 30 20 = 10 . Pelo item a) sabemos que CEB = 10 , donde temos que BP0 vai passar pelos pontos P e E. Assim o ngulo EPO =

RA

na figura e alguns usaram tambm para resolver o item (a).

CPB = EPB = 10 PCB = CBE + CEB = 20 + 10 = 30 PBC = 180 CPB PCB = 140

Problema 5 Em uma fila existem 30 crianas marcadas com os nmeros 1, 2, . . . , 30 da esquerda para a direita. Para
cada criana marcada com o nmero i tal que 2 i 15, a quantidade de amigos com nmero maior que i igual a 1 mais a quantidade de amigos com nmero menor que i. Para cada criana marcada com nmero i tal que 16 i 29, a quantidade de amigos com nmero menor que i igual a 2 mais a quantidade de amigos com nmero maior que i. A criana 1 tem 19 amigos. Quantos amigos possui a criana 30?

Soluo:

92

Avaliaes e Simulados Gabarito da Prova de Seleo Professor Paulo Rodrigues

19 1 1 1

2 2 2
.

com 0, e a diagonal no ser preenchida. Observemos que, como a relao de amizade recproca, o tabuleiro ser simtrico em relao diagonal que vai do canto superior esquerdo ao canto inferior direito, por exemplo, na casa que esta na fila 3 coluna 26 temos um 1, se e somente se, na casa que esta na fila 26 coluna 3 temos tambm um 1. Agora, vamos construir um novo tabuleiro, mudando o sinal dos nmeros que esto abaixo da diagonal, obtendo uma tabela que tem soma de todas suas entrada igual a 0. Vejamos que a soma das entradas da linha i corresponde ao nmero de amigos que tem nmero maior que i menos o nmero de amigos com nmero menor que i. Pelas hipteses do problema, esta soma na primeira linha 19, nas linhas da 2a 15a esta soma 1, nas linha da 16a 29a esta soma

2 e na linha 30a esta soma corresponde x, onde x o nmero de amigos da criana 30. Como a soma deste nmeros
zero temos que

RA
14 vezes

19 + 1 + 1 + + 1 (2 + 2 + + 2) x = 0.
14 vezes

FT
para 2 i 15 para 16 i 29
14 vezes

Construmos um tabuleiro 3030, onde preenchemos segundo as relaes amizade da seguinte forma: se a criana i amiga da criana j ento a casa na linha i e coluna j ser preenchida com 1, e caso no sejam amigas ser preenchida

Portanto, x = 19 14 = 5.

Comentrios da Equipe do PECI: Outra maneira de escrever basicamente a mesma soluo:


Denotemos por xi o nmero de amigos da criana numerada com i, os quais tm nmero menor que i, e yi o nmero de amigos da criana numerada com i, os quais tm nmero maior que i. Pelas hipteses do problema sabemos que

x = 0 1 y = 19 1 y =0 30 x = y 1 i i x = y + 2
i i

Alm disso, para cada par de amigos a b, esta relao est adicionando uma unidade para xa e uma unidade para

yb . Portanto, x1 + x2 + + x30 = y1 + y2 + + y30 ,


donde

(y1 x1 ) + (y2 x2 ) + + (y15 x15 ) + (y16 x16 ) + + (y29 x29 ) + (y30 x30 ) = 0.
Assim

19 + 1 + 1 + + 1 (2 + 2 + + 2) x30 = 0.
14 vezes

OBMEP PECI Preparao Especial para Competies Internacionais Logo, x30 = 19 + 14 2 14 = 5.

93

A figura abaixo mostra que a situao descrita no problema possvel. Ligamos os vrtices i e j se as pessoas

i e j so amigas.

30

29

28

27

26

25

24

23

22

21

20

19

18

17

16

10

11

12

13

14

15

Problema 6 Colocamos 15 cavalos em um tabuleiro 15 15 de modo que no existam dois cavalos em uma mesma
linha ou em uma mesma coluna. Cada cavalo faz ento um movimento. Prove que agora existem dois cavalos na mesma linha ou na mesma coluna.

Observao: O cavalo movimenta-se em L. Assim, um cavalo na casa marcada com

Soluo:

Um dos motivos que 15 mpar, se o nmero de cavalos fosse par, poderamos colocar os cavalos de modo que dois cavalos troquem de posio no movimento, se fizermos isto, sobra um cavalo sem movimentar-se. Uma maneira de provar a seguinte, vamos numerar cada cavalo pela posio, se est na linha 1 coluna 2, como

1 + 2 = 3, o nmero dele 3. A soma de todos os nmeros ser 450 se no houver cavalos diferentes na mesma linha ou coluna, a cada movimento, o nmero de cavalos se altera em: 3, 1, 1 ou 3.
So nmeros mpares, para que continue com apenas 1 cavalo em cada linha e em cada coluna, a soma dos nmeros dos cavalos deveria ser 450, o que implicaria uma alterao nula nesta soma, ou seja, 0, a alterao total ser a soma das alteraes individuais de cada cavalo, porm, 0 par e a soma de 15 nmeros mpares no pode ser par, por isso, impossvel que continue um cavalo em cada linha e um em cada coluna, logo, existem dois cavalos na mesma linha ou na mesma coluna.

de Thiago Lucas Faustino da Silva de Itumbiara Gois.

RA
.

FT

pode mover-se para as casas destacadas.

Comentrios da Equipe do PECI: Muitos alunos partiram da hiptese que os cavalos deviam estar em uma das
diagonais do tabuleiro, o que falso. Na verdade existem 15! = 1 307 674 368 000 maneiras de colocar os cavalos no tabuleiro, de modo que no fiquem dois na mesma linha ou na mesma coluna. A figura abaixo ilustra uma dessas maneiras.

94

Avaliaes e Simulados Gabarito da Prova de Seleo Professor Paulo Rodrigues

RA

FT

Solues

21

Gabarito da Avaliao

21.1 lgebra Problema 1


Determine m para que as razes da equao x2 3x + 2m = 0 sejam reais e iguais.

Soluo: Basta fazer = 0. Deste modo, b2 4ac = 0 9 8m = 0 m =

9 . 8

Problema 2
(a) x2 + (b) x3 +

Se x um nmero real tal que x +

1 = k, determine, em funo de k x

1 x2 1 x3

RA
95

Soluo: ( )2 1 1 1 1 1 (a) x + = x2 + 2 x + 2 k2 = x2 + 2 + 2 x2 + 2 = k2 2. x x x x x ( )3 ( ) 1 1 1 1 1 1 1 (b) x + = x3 + 3 x2 + 3 x 2 + 3 k3 = x3 + 3 x + + 3 x3 + 3 = k3 3k. x x x x x x x

FT

Problema 3

Resolver, em R, o sistema de equaes abaixo.

D
Soluo:

y2 + u2 + v2 + w2 = 4x 1 2 x + u2 + v2 + w2 = 4y 1 x2 + y2 + v2 + w2 = 4u 1 2 x + y2 + u2 + w2 = 4v 1 2 x + y2 + u2 + v2 = 4w 1

Somando todas as equaes, obtemos

4x2 + 4y2 + 4u2 + 4v2 + 4w2 = 4x 1 + 4y 1 + 4u 1 + 4v 1 + 4w 1.


Reorganizando essa equao, vem (2x 1)2 + (2y 1)2 + (2u 1)2 + (2v 1)2 + (2w 1)2 = 0, o que resulta

x=y=u=v=w=
ocorre.

1 . Voltando ao sistema, observe que a igualdade 1/4 + 1/4 + 1/4 + 1/4 = 4 1/2 1 de fato 2

96

Avaliaes e Simulados Gabarito da Avaliao Professor Paulo Rodrigues

21.2 Combinatria Problema 4


Dados 50 pontos arbitrrios dentro de um quadrado de lado 1, mostre que existem dois desses pontos

cuja distncia entre eles menor que

2 . 9

Soluo: Dividindo o quadrado em 49 quadrados de lado 1/7, temos pelo Princpio das Casas dos Pombos, que dois
pontos esto no mesmo quadrado. Observemos que a distncia mxima dentro de um quadrado sua respectiva diagonal, que mede

2/7.

Mas fcil verificar que

2/7 < 2/9, pois elevando ao quadrado, isto equivalente a

2 49

<

4 81 .

Problema 5

Em um tabuleiro de xadrez 88 so retirados os cantos superior esquerdo e o inferior direito, obtendo-

se um tabuleiro truncado. Explique se possvel um cavalo, partindo de uma casa qualquer, percorrer todas as casas do tabuleiro truncado passando por cada casa uma nica vez.

Soluo:

Podemos supor, sem perda de generalidade que o canto superior esquerdo preto, e portanto o canto

inferior direito tambm preto, assim o tabuleiro truncado tem 30 casas pretas e 32 casas brancas. Para percorrer todo o tabuleiro o cavalo tem que dar 61 pulos, e cada vez que o cavalo pula, as casas de partida e de chegada so de cores distintas, segue que depois de um nmero mpar de pulos ele j percorreu a mesma quantidade de casas brancas que pretas e depois de um nmero par de pulos ele percorreu uma casa a mais de uma cor que da outra. Conclumos que impossvel percorrer todo o tabuleiro, j que o nmero de casas brancas supera em dois o nmero de casas pretas.

FT
C D X Y E F W V

Problema 6 Soluo:

Prove que dentre 52 inteiros possvel escolher dois cuja soma ou diferena divisvel por 100.

Para aplicar o Princpio das Casas dos Pombos neste problema, construremos 51 casas da seguinte forma:

na casa j, colocamos o nmeros que deixam resto j ou 100 j quando dividimos por 100 onde j = 0, 1, . . . , 50. Como temos 52 nmeros, pelo Princpio das Casas dos Pombos, existem dois nmeros que esto na mesma casa. Caso estes dois nmeros deixem o mesmo resto quando so divididos por 100, sua diferena ser divisvel por 100. Por outro lado, se deixam restos distintos, sua soma ser divisvel por 100

RA
A B Z
.

Geometria

Problema 7

e um pentgono regular DEVWX.

(a) Determine a medida do ngulo Y DC.


(b) Mostre que o vrtice X est sobre a reta DY .

21.3

A figura mostra parte de um polgono regular de 20 lados (icosgono) ABCDEF..., um quadrado BCYZ

OBMEP PECI Preparao Especial para Competies Internacionais

97

Soluo:

C D

A E

C D X E F W V

Y F

(a) O ngulo interno do icosgono regular mede

180 18 = 162 . Segue que Y CD = 162 90 = 72 . Como 20 180 72 YC = CD, o tringulo YCD issceles de base YD. Assim, Y DC = DYC = = 54 . 2

(b) Cada ngulo interno de um pentgono regular mede reta.

(c) Este problema no tem item (c), mas poderamos ter perguntado: Qual a nica letra do alfabeto que ainda poderamos usar nesta figura?

Resposta:

Problema 8

Quais so os possveis valores da rea do tringulo AEG?

Fatos que Ajudam: Tringulos com mesma base e mesma altura possuem reas iguais.

Soluo: Traamos a diagonal AC do quadrado ABCD. Como as retas AC e GE formam ngulo de 45o em relao
reta BE, conclumos que AC e GE so paralelas.

Como T a vigsima letra do alfabeto, o icosgono ABCDEF . . . T . Como usamos tambm V , W , X, Y e Z, s

RA
A D G
.

Na figura, ABCD e CEFG so quadrados e o lado do quadrado CEFG mede 12 cm.

faltou a letra U! Voc j tinha visto um problema de geometria com tantas letras?

FT
F C E D G X F C E

180 3 = 108 . Assim, CDX = 162 108 = 54 . Como as 5 retas XD e YD formam o mesmo ngulo com a reta CD, segue que os pontos X, Y e D pertencem a uma mesma

98

Avaliaes e Simulados Gabarito da Avaliao Professor Paulo Rodrigues Seja X um ponto arbitrrio sobre AC. Os tringulos AGE e XGE possuem a mesma rea, pois ambos tm a mesma

base GE e a mesma altura que corresponde distncia entre as retas paralelas AC e GE. Tomando X = C, conclumos que a rea do tringulo AGE igual rea de CGE, isto , 12 12/2 = 72 cm2 .

Problema 9

Mostre que possvel construir um pentgono com todos os lados de mesma medida e cujos ngulos

internos meam 60 , 80 , 100 , 140 e 160 , em alguma ordem.

Fatos que Ajudam: Se um quadriltero possui os quatro lados de mesma medida, ento ele um losango. Em um
losango, os ngulos opostos possuem a mesma medida.

Soluo:

C B B
140

C
100 80

60

D E
.
60

160

Suponhamos que j construmos o pentgono ABCDE e que o ngulo em A mede 60 . Traando a reta BE,

conclumos que o tringulo ABE equiltero, pois AB = AE e EAB = 60 . Logo, BE = AB e, portanto, BCDE tem
todos os seus lados com a mesma medida, isto , BCDE um losango. Em particular, os ngulos opostos do losango so iguais. Isto implica que, no pentgono, o ngulo em B igual Como 160 = 100 + 80 e 140 = 80 + 60 , conclumos que os ngulos em C e D devem assumir os valores 80 e 100 , no necessariamente nessa ordem, enquanto B e E assumem os respectivos valores de D e C, adicionados de

60 .

Portanto, para construir tal pentgono basta construir um tringulo equiltero ABE e um losango BCDE com ngulos de medidas 100 e 80 .

RA

ao ngulo em D mais 60 e o ngulo em E igual ao ngulo em C mais 60 .

FT
A E

21.4 Nmeros

Problema 10

Soluo: Como 1980 = 22 32 5 11 segue que 11 fator primo de 1980 e possui mais que um algarismo. Logo,
no pode existir um nmero natural cujo produto de seus algarismos seja 1980.

Existe um nmero natural tal que o produto de seus algarismos igual a 1980?

Problema 11 Determine todos os inteiros positivos n tais que 3n 4, 4n 5 e 5n 3 so todos primos. Soluo:
Note que (3n 4) + (4n 5) + (5n 3) = 12n 12 = 2(6n 6), ou seja, a soma dos trs nmeros par.

Se os trs fossem mpares ento a soma seria mpar, logo algum deles par. Como 2 o nico primo par, algum deles igual a 2. Temos 3 casos: 1. Se 3n 4 = 2 ento n = 2. Assim, uma soluo 2, 3 e 7. 2. Se 4n 5 = 2 ento n no inteiro, absurdo. 3. Se 5n 3 = 2 ento n = 1. Porm, para n = 1 temos 3n 4 = 1, que no positivo, absurdo.

OBMEP PECI Preparao Especial para Competies Internacionais Logo, a nica soluo n = 2.

99

Problema 12
perfeito?

Um nmero escrito com cem algarismos 0, cem algarismo 1 e cem algarismos 2 pode ser um quadrado

Soluo: A soma dos algarismos de qualquer nmero dessa forma 0 100 + 1 100 + 2 100 = 300. Portanto,
qualquer nmero dessa forma mltiplo de 3, pois 3 divide 300, mas no mltiplo de 9 pois 9 no divide 300. Assim, se um nmero dessa forma um quadrado perfeito, como 3 divide esse nmero teramos que ter 32 = 9 dividindo esse nmero. Logo, nenhum nmero dessa forma pode ser um quadrado perfeito.

RA

FT

Solues

22

Gabarito do Simulado 1

Problema 1
(a) Simplifique a expresso

x2 (x + 1)2 (x + 2)2 + (x + 3)2 .


(b) Prove que, para todo inteiro positivo n, existe um inteiro positivo k e uma escolha adequada de sinais + e para a qual

(por exemplo, 12 = 12 + 22 + 32 ).

Soluo:

(a)

x2 (x + 1)2 (x + 2)2 + (x + 3)2 =

x2 (x2 + 2x + 1) (x2 + 4x + 4) + (x2 + 6x + 9) = 4


Logo, x2 (x + 1)2 (x + 2)2 + (x + 3)2 = 4 para todo x. (b) Vejamos para n = 1, n = 2, n = 3 e n = 4.

1 = +12 ;

2 = 12 22 32 + 42 ; 3 = 12 + 22 ; 4 = 12 22 + 32 .

Considere agora n > 4. Indutivamente, suponha o resultado vlido para n4. Existem k inteiro positivo e uma escolha de sinais + e tais que:

Pelo item anterior e pela hiptese de induo temos:

Logo, o resultado tambm vlido para n. Pelo Princpio da Induo Finita segue que o resultado vale para todos os nmeros inteiros positivos.

RA
n 4 = 12 22 k2 .
n = 12 22 k2 + (k + 1)2 (k + 2)2 (k + 3)2 + (k + 4)2 .

FT

100

n = 12 22 k2 .

OBMEP PECI Preparao Especial para Competies Internacionais

101

Problema 2

No tringulo ABC com AB = BC, traamos a bissetriz BD, com D sobre AC. Marcamos o ponto E

sobre BD, diferente de D, de tal modo que CE = CD. Prove que o ponto mdio de DE est sobre a base mdia de

ABC, paralela ao lado AB.


Soluo 1
Seja H o ponto mdio de DE e trae, por H, o segmento

FG com F AC e G BC tal que FG AB. No tringulo B


issceles DCE, temos que CH mediana e altura com relao base DE. Se CBD = DBA = , ento temos BHG =

DBA = (alternos internos), logo o tringulo BGH issceles e BG = GH. Como o tringulo BCH retngulo, temos HCB = 90 CBH = 90 e

CHG = 90 BHG = 90 , ou seja, o tringulo CGH issceles e CG = GH = BG.


Portanto, G ponto mdio de BC e FG base mdia

E
90

do tringulo ABC.

H D F

Soluo 2

D
E H
.

RA
entes.

FT
A
Como BD bissetriz, ao refletir o tringulo BCD com relao BD, obtemos um tringulo BC D, com C sobre

AB. Temos tambm que o tringulo C DE uma reflexo do tringulo CDE em relao BD, logo eles so congruSe H ponto mdio de DE, ento em ambos os trin-

gulos, H p da altura com relao base. Assim, C, H e

C esto alinhados e, em particular, H ponto mdio da ceviana CC , logo H est sobre a base mdia em relao

AB (ou de outro modo, traando FG paralelo a AB, conforme a soluo anterior, temos que CFG CAB, com CH 1 AB razo de semelhana igual a = , logo FG = CC 2 2
base mdia).

102

Avaliaes e Simulados Gabarito do Simulado 1 Professor Paulo Rodrigues Existem n jogadores participando de um torneio de xadrez. Cada jogador deve disputar uma partida

Problema 3

com cada um dos outros n 1 jogadores, e no mais que uma partida por dia. Determine o menor nmero de dias necessrios para terminar o torneio. Mostre como poderia ser realizado o torneio com a quantidade de dias encontrada.

Soluo:
claro que se o nmero n for mpar, ento pelo menos um jogador dever descansar em cada rodada e teremos no mnimo n rodadas para completar o torneio, porque o jogador que descansar na primeira rodada precisar de no mnimo mais n 1 rodadas para enfrentar todos os jogadores. Se n for par, como so realizados no mximo n/2 partidas por dia e o torneio tem n(n 1)/2 partidas, so necessrios no mnimo

1 4. n = 5. 1 1 2 3 4 5 0 1 2 3 4 2 1 2 3 4 5 3 2 3 4 5 1 4 3 4 5 1 2 5 4 5 1 2 3

dias para concluir o torneio.

Vamos mostrar que esses valores so de fato, os mnimos, construindo uma tabela com essas quantidades de dias em cada caso. Para tal, vamos escrever na casa que est linha i e na

neio. claro que no escreveremos nada nas casas do tipo

(i, i) e que no podem aparecer dois nmeros iguais em


uma mesma linha ou coluna. casos particulares:

Antes de passar para o caso geral, observemos dois

n = 3.

1 1 2 3 0 1 2

D
1 2 3
.

A tabela para n = 4 pode ser obtida a partir de n = 3. O jogador 4 enfrentar os jogadores na ordem 3 4, 2 4 e

RA
2 3 2 3 1

coluna j o dia em que se jogadores se enfrentam no tor-

FT
. como 1 j, k n, teremos j = k. rodada para resolver o caso par.

n(n 1)/2 =n1 n/2

Observe que cada jogador descansa um dia. Para construir uma tabela para n = 6, basta que o

jogador 6 dispute as partidas com os jogadores que descansavam na tabela de n = 5. Assim, a sequncia de jogos deste jogador ser: 4 6, 2 6, 5 6, 3 6 e 1 6. No caso geral para n mpar, colocamos a partida entre os jogadores i e j no dia i + j 2 (mod n), lembrando de escrever n no lugar de zero. Todas as linhas da tabela te-

ro nmeros distintos. De fato, suponha que os nmeros que aparecem em (i, j) e (i, k) sejam iguais. Ento tere-

mos i + j 2 i + k 2 (mod n), donde j k mod n e O jogador i descansa no dia 2i 2 (mod n). No possvel que dois jogadores descansem no mesmo dia porque se 2i 2 2j 2 (mod n), ento 2(i j) 0

(mod n), e como n mpar, devemos ter i j (mod n), donde i = j.


Assim, a cada dia um jogador diferente descansa e esse jogador o que enfrentar o jogador n + 1 nessa

Solues

23

Gabarito do Simulado 2

Simulado aplicado no primeiro encontro presencial no dia 13/01 com durao de 4 horas e 30 minutos.
Problema 1
Calcule a rea do tringulo ABC abaixo, dados BD = 4, DE = 2, EC = 6, BF = FC = 3.

F D B

.E

Soluo:

Como BD/DE = 2 e D um ponto da mediana AF, ento D baricentro do tringulo ABC e consequen-

temente E ponto mdio do lado CA. Alm disso, os tringulos BCE e ABE tm mesma altura e mesma base, ou seja,

RA
103

mesma rea.

FT
A

Como o tringulo BCE equiltero de lado 6, ento BCE tem rea 9 3 e o tringulo ABC tem rea 18 3.

Problema 2

Esto alinhados N gafanhotos (N 3). A cada segundo, um dos gafanhotos (exceto os dois primeiros)

pula e passa frente dos dois imediatamente sua frente. Aps K segundos, os N gafanhotos esto novamente na ordem original. Quais os possveis valores de K?

Soluo:

Se N = 3 em cada segundo apenas um gafanhoto pode pular e assim K tem que ser mltiplo de 3.

Se N > 3 temos as seguintes possibilidades:

(i) 1234 . . . N 1342 . . . N 3412 . . . N 3124 . . . N 1234 . . . N (ii) 1234 . . . N 2314 . . . N 3124 . . . N 1234 . . . N Assim possvel voltar posio original em 3 ou 4 jogadas. Deste modo, tambm em 3a+4b jogadas, combinandoas (a e b naturais). Em 1 ou 2 jogadas impossvel pois dois gafanhotos pulados na primeira jogada tm que pular o primeiro e com esse argumento tambm descartamos K = 5. Como 3 2 = 6, 3 1 + 4 1 = 7, 4 2 = 8 e para obter os demais valores basta ir acrescentando 3, qualquer natural K 6 pode ser escrito como 3a + 4b. Assim, K pode ser 3,

4 ou qualquer natural maior ou igual a 6.

Problema 3

Em Terra Brasilis existem n casas, onde vivem n duendes, cada um em uma casa. Nesse pas temos

estradas de mo nica satisfazendo as seguintes condies: (i) Cada estrada une duas casas. (ii) Em cada casa comea exatamente uma estrada. (iii) Em cada casa termina exatamente uma estrada.

104

Avaliaes e Simulados Gabarito do Simulado 2 Professor Paulo Rodrigues

Todos os dias, a partir do dia 1, cada duende sai da casa onde est e chega casa vizinha. Uma lenda de Terra Brasilis diz que, quando todos os duendes regressarem s suas casas originais, o mundo acabar. (a) Mostre que o mundo acabar. (b) Se n = 98, mostre que possvel que os duendes construam e orientem as estradas de modo que o mundo no acabe antes de 300 000 anos.

Soluo:
(a) Numere os duendes de 1 a n e seja f(i) o vizinho do duende nmero i. Fixe 1 i n. Como a sequncia

f(i), f(f(i)), f(f(f(i))), . . .


assume somente um nmero finito de valores, existiro inteiros positivos r < s tais que

f(s) (i) = f(r) (i).

f(sr) (i) = i.

Seja agora g(i) o menor inteiro positivo tal que o duende i retorna sua casa aps g(i) dias. Ento aps mmc(g(1), g(2), . . . , g(n))

dias todos os duendes retornaro posio original e o mundo acabar. (b) Divida os 98 duendes em 8 ciclos de tamanhos 3, 5, 7, 11, 13, 17, 19, 23 (veja que 98 = 3+5+7+11+13+17+19+23). Da discusso do item (i) imediato que os duendes s retornaro simultaneamente s suas posies originais aps um nmero de dias igual a

RA
3 5 7 11 13 17 19 23

ou seja, aps mais de 300 000 anos. Alternativamente, podemos dividir os duendes em ciclos de tamanhos 3, 8, 9, 5, 7, 11, 13, 19 e 23, e eles retornaro posio original aps mmc(3, 8, 9, 5, 7, 11, 13, 19, 23)

dias, ou aproximadamente 431 444 anos.

FT
= 111546435 > 366 300 000,

Desde que a funo f claramente uma bijeo, temos

Apndice A

Utilizando TEX no Frum


A.1 O que TEX? A.1.1 Knuth inventou o TEX...

O problema de escrever matemtica no computador surgiu na dcada de 70. Um dos primeiros grandes matemticos a pesquisar em Cincia da Computao, Donald Knuth da Universidade de Stanford, encontrou uma soluvolta de 1976, Knuth tinha escrito os dois primeiros volumes da coleo The Art of Computer Programming e

o que continua atual mais de trinta anos depois. Por

estava totalmente insatisfeito com o resultado impresso. Ele no queria apenas que o livro fosse impresso, mas queria algo belo.

Knuth partiu para a busca de uma soluo. Um dos primeiros passos foi a interrupo de sua pesquisa por um ano para, acompanhado por sua esposa, assistir aulas de design com o professor de arte de Stanford, Matthew Kahn. A ideia era tentar capturar a essncia do design, no apenas seu visual. Por exemplo, como um processador deveria quebrar as linhas em um pargrafo? Esteticamente, o ideal que no existam espaos excessivos entre as palavras e que no existam muitos hifens. Knuth transformou esse problema em combinatria e fez um algoritmo que calcula a maneira tima de quebrar as linhas em um pargrafo. Como resultado do seu trabalho, surgiram o processador de textos TEX e o sistema de descrio de fontes META-

FONT, ambos colocados em domnio pblico. O TEX foi projetado com dois objetivos principais em mente: permitir
que qualquer pessoa possa produzir livros de alta qualidade com um esforo razovel e dar exatamente o mesmo resultado em todos os computadores, agora e no futuro.

A.1.2
A ... e Lamport criou o LTEX

No incio da dcada de 80, o matemtico Leslie Lamport planejava escrever o livro Great American Concurrency Book e digitar utilizando TEX. Ele escreveu um conjunto de macros que facilitaram bastante o trabalho. Essas macros foram 105

RA

FT

106

Avaliaes e Simulados Gabarito do Simulado 2 Professor Paulo Rodrigues

A posteriormente colocadas em domnio pblico. Era o incio do LTEX. Lamport at hoje no escreveu o livro pretendido, A A mas em 1986 lanou o livro LTEX: A Document Preparation System, que ajudou a popularizar o LTEX. De l para c, A vrios conjuntos de macros para TEX surgiram, como ConTeXt e JadeTeX, mas sem dvida o LTEX o mais utilizado. Hoje o TEX popular em todo o mundo, principalmente na rea acadmica, notadamente em matemtica, fsica,

cincia da computao e engenharias.


A Uma das vantagens do LTEX a sua modularizao. Qualquer um pode escrever um conjunto de macros que A automatizam determinados procedimentos e facilitam a vida de todos. No caso do LTEX, estas macros so chamadas

de pacotes e existem milhares de pacotes escritos por centenas de usurios ao redor do mundo. A principal desvantagem inicial do TEX que no um editor WYSIWYG (acrnimo da expresso em ingls What You See Is What You Get O que voc v o que voc recebe). Isto significa que digitamos o texto usando uma linguagem especfica, compilamos e depois vemos o resultado. Isto pode parecer muito estranho para quem est acostumado a editores WYSIWYG, como o Word, mas uma barreira que pode ser facilmente superada.

Curiosidades
trs volumes completos foram lanados, alm de cinco fascculos do volume 4.

Ele recebeu inmeros prmios como pesquisador em Cincia da Computao e em agosto de 1999 seu nome foi dado a um
pequeno planeta descoberto por P. Pravec and P. Kusnirk.

Desde 2001, Lamport pesquisador da Microsoft.

Existem verses de TEX para praticamente todos os sistemas operacionais, incluindo Windows, Mac OS X e Linux.
mas, para garantir a mesma sada em todas as verses do TEX, ele deseja que qualquer novo programa obtido tenha outro nome. Para garantir isso, a American Mathematical Society registrou a marca TEX e qualquer implementao do sistema deve passar por um teste antes de ser chamada de TEX.

O nome TEX deve ser pronunciado como tekh. O X representa a letra grega (chi). TEX uma abreviao de (techn), que
tambm a origem da palavra tcnico.

Knuth escreveu cinco livros sobre TEX: The TEX book, TEX: The Program, The METAFONT book, METAFONT: The Program e
Computer Modern Typefaces, todos lanados pela Addison-Wesley.

numeradas como aproximaes do nmero e, base dos logaritmos naturais.

Atualmente, os grupos de usurios TEX (www.tug.org) de diversos pases so responsveis pela distribuio, manuteno e
atualizaes nas macros para TEX.

A.2 Escrevendo e desenhando no frum


A O frum do Programa Oficinas de Formao est associado a uma instalao do programa LTEX, utilizado para digitar

matemtica. Para tal, voc deve digitar [tex] comandos [/tex]. Por exemplo, digitando [ tex ] \ f r a c { 3 } { 8 } [ / tex ] o sistema converter seu cdigo para uma imagem contendo
3 8.

As verses de TEX so numeradas como aproximaes do nmero . A verso atual a 3.141592. Knuth deseja que, aps a sua morte o TEX no seja alterado, com exceo da verso, que dever ser a . Analogamente, as verses de METAFONT so

clicando no boto Prever ou no boto Enviar. Sugerimos que voc aprenda inicialmente a escrever os exemplos
A bsicos abaixo, os quais representam mais de 90% da utilizao do LTEX no frum.

RA

O cdigo fonte do TEX foi colocado em domnio pblico, e Knuth recomenda modificaes ou experincias com esse cdigo fonte,

FT

Knuth at hoje no terminou a coleo The Art of Computer Programming. Dos sete volumes previstos inicialmente, somente

A imagem s exibida na mensagem a ser visualizada

OBMEP PECI Preparao Especial para Competies Internacionais

107

A.2.1 Exemplos Bsicos 3+5 ........................ 3 + 5 7-2 ........................ 7 2 \times ....................... 3\times2 ................. 3 2 3\cdot2 ..................... 3 2 \frac{3}{8} ................
3 8

\pm ............................ 10\% ........................ 10% a_1 ........................... a1 b_{23} ...................... b23 x^{11} ...................... x11 \sqrt{2} ................... 2 3 \sqrt[3]{2} ............... 2 < .............................. < 2<3 ........................ 2 < 3 \le ............................

a\le b ................... a b \ge ............................ a\ge b ................... a b (1,2) ...................... (1, 2) [1,2] ...................... [1, 2] \{ .............................. { \} .............................. } \{1,2\} ................... {1, 2} \mid-3\mid ............... | 3 |

3/8 .......................... 3/8 3 \dfrac{3}{8} .............. 8 20\div3 .................. 20 3 \ne ............................ =

A Ateno! A melhor maneira de aprender a digitar em L TEX praticando. Voc no precisa

prtica, voc j conhecer os principais comandos e com certeza ter prazer em escrever
A usando o L TEX.

A.2.2 Letras Gregas \pi ............................

\Pi ............................

RA

\Delta ........................ \delta ........................ \epsilon ..................... \phi ..........................

\alpha ........................ \beta ......................... \Gamma ........................

\gamma ........................

\lambda ....................... \mu ............................

A.2.3 Aritmtica

\equiv .......................................................................................................

(mod n) a\equiv b\pmod{n} ........................................................................... a b (mod n)


\phi(n) ................................................................................................... (n) \lfloor x\rfloor \lceil x\rceil A.2.4 Geometria \angle ABC ................................. ABC \measuredangle ABC ...................... ABC A\hat{B}C .................................... ABC \widehat{ABC} ............................... ABC r\parallel s ................................. r s r\perp s ..................................... r s \circ ............................................. 90^{\circ} .................................... 90 \overline{AB} ................................ AB ....................................................................................... x ......................................................................................... x

\pmod{n} .............................................................................................

FT

decorar todos os comandos. Consulte-os neste manual, quando precisar. Aps alguma

\rho ........................... \sigma ........................ \Sigma ........................ \theta ........................ \Omega ....................... \omega .......................

108

Avaliaes e Simulados Gabarito do Simulado 2 Professor Paulo Rodrigues

\vec{v} ........................................... v \arco{AB}....................................... AB1 \triangle ABC ............................. ABC \cong ............................................. = \triangle ABC \cong \triangle XYZ A.2.5 Setas \iff ......................................... ................................... .................................... ............................ = ............................

ABC = XYZ \sim ............................................... \triangle ABC \sim \triangle XYZ ABC XYZ

\searrow \swarrow \nwarrow \uparrow \nearrow

....................................... ..................................... .................................... ........................................ ....................................... ....................................... .......................................

\Rightarrow \Leftarrow

\downarrow \leftarrow

\Longrightarrow \longrightarrow \mapsto \rightarrow

........................................ ...................................

A.2.6 Smbolos Diversos

\$ .............................. $
\dots............................ . . \ldots ......................... . . . . \vdots............................ . \cdots ......................... . \ddots....................... . .

FT
................. \clubsuit \square \TeX \S \P ................. ................. ..............

\therefore \bullet

..................

RA
\diamond \Diamond \Box \heartsuit \spadesuit \diamondsuit

\approx ......................

\star........................... \bigstar...................... ..................... .............. \blacksquare

..................... .................... ....................

.........................

\ell..............................

....................... TEX A \LaTeX ................... LTEX

\infty..........................

............................

............................

A.2.7

Conjuntos \in

......................... .....................

\cup .......................... \emptyset .................... \mathbb{N} .................. N \mathbb{Z} .................. Z \mathbb{Q} .................. Q

\mathbb{R} .................. R \mathbb{C} .................. C \mathcal{P}(X)........... P(X)

\not\in

\subset ....................... \not\subset ................. \cap .......................... A.2.8 Matrizes e Determinantes

\begin{matrix}1 & 2\\ 3 & 4\end{matrix}


1

......................................................

1 2 3 4

Comando personalizado para o frum

OBMEP PECI Preparao Especial para Competies Internacionais

109

(
\begin{pmatrix}1 & 2\\ 3 & 4\end{pmatrix} \begin{bmatrix}1 & 2\\ 3 & 4\end{bmatrix} \begin{Bmatrix}1 & 2\\ 3 & 4\end{Bmatrix} \begin{vmatrix}1 & 2\\ 3 & 4\end{vmatrix} \begin{Vmatrix}1 & 2\\ 3 & 4\end{Vmatrix} ................................................

2 4

3 [ 1 ................................................. 3 { 1 ................................................ 3
.................................................. .................................................

] 2 4 } 2 4

1 2 3 4 1 2

\begin{pmatrix}1 \\ 2 \\ 3\end{pmatrix}

3 4 1 ...................................................... 2 3

A.2.9 Somatrios e Produtrios

FT

\det{A} .................................................................................................. det A

\sum_{i=1}^{n} i^2..................................................................................

n
i=1 n i=1

i2 i2

\displaystyle\sum_{i=1}^{n} i^2...................................................................

RA

\sum_{i=1}^{100} i(i+1)......................................................................

100
i=1 100 i=1

i(i + 1) i(i + 1)
i i=1 i+1

\displaystyle\sum_{i=1}^{100} i(i+1)....................................................... \prod_{i=1}^{10} \frac{i}{i+1}................................................................

10
10 i=1

\displaystyle\prod_{i=1}^{10} \frac{i}{i+1}............................................... \displaystyle\sum_{i\ge 1} \frac{1}{i^2} = \frac{\pi^2}{6}....................... A.2.10

i i+1

1 2 = i2 6
i1

Diversos

x + y = 10 \begin{cases}x+y=10\\ x-y=4\end{cases}................................................ x y = 4
40

1\overbrace{22\dots2}^{40}5 ................................................................... 1 22 . . . 2 5 \underbrace{11\dots1}_{100} ...................................................................... 11 . . . 1


100

ax + b (\dfrac{ax+b}{cx+d}).............................................................................. ( ) cx + d ( ) ax + b \left(\dfrac{ax+b}{cx+d}\right).............................................................. cx + d

110

Avaliaes e Simulados Gabarito do Simulado 2 Professor Paulo Rodrigues

A.2.11
A Construindo guras com LTEX
A possvel fazer desenhos dos mais variados tipos usando LTEX. Existem centenas de pacotes para fazer figuras. No frum foram instalados os pacotes pstricks e pst-eucl. Os comandos para desenhar figuras devem estar entre as tags

[teximg] e [/teximg].
Apresentamos a seguir alguns exemplos bsicos de figuras feitas com pstricks. Em todos os casos, a imagem mostrada direita contm, para facilitar a compreenso, uma grade de pontos que no consta no cdigo exibido. Traando segmentos de reta Os comandos abaixo definem uma caixa com extremidades (0,0) e (4,4) e trs segmentos de reta que tm por extremidades os pontos designados. O parmetro opcional no segundo segmento ([linestyle=dashed]) muda o estilo da linha para tracejado.
4

\begin{pspicture}(0,0)(4,4) \psline(0,1)(1,4) \psline[linestyle=dashed](2,0)(2,4) \psline{->}(1,0)(4,3) \end{pspicture}


3

Circunferncias

Para construir uma circunferncia, devemos informar o centro e o raio com o comando \pscircle(x,y){r}, sendo

(x,y) as coordenadas do centro da circunferncia e r o raio.

Na segunda circunferncia, aumentamos a espessura da linha com o parmetro [linewidth=2pt].


4

\begin{pspicture}(0,0)(4,4) \pscircle(2,2){2} \end{pspicture}

\pscircle[linewidth=2pt](3,3){1}

D
Rotulando Pontos

RA
3 2 1 0 0

O pacote pst-euclides nos permite dar nome aos pontos e depois utilizar os nomes dados para fazer outras construes. O comando \pstGeonode[PosAngle=](x,y){Nome} marca um ponto de coordenadas (x,y) com o rtulo Nome. O ngulo do rtulo em relao ao ponto dado pelo parmetro opcional PosAngle.

\begin{pspicture}(0,0)(4,4) \pstGeonode(4,3){A} \pstGeonode[PosAngle=90](1,4){B} \pstGeonode[PosAngle=180](0,1){C} \pstGeonode[PosAngle=-90](3,0){D}

FT
2 1 0 0 1 2 3 4 1 2 3 4

\psline(A)(B)(C)(D)(A) \end{pspicture}

OBMEP PECI Preparao Especial para Competies Internacionais


4

111

0 0 1 2

Utilizando coordenadas polares possvel utilizar coordenadas polares para definir os pontos. Neste caso, devemos indicar a distncia r do ponto origem e o ngulo formado entre o segmento de reta que une esse ponto origem e o eixo x. As coordenadas polares devem ser separadas por ponto e vrgula: (r;).
2

\begin{pspicture}(-2,-2)(2,2) \psline(2;0)(2;72) \psline(2;72)(2;144) \psline(2;144)(2;216) \psline(2;216)(2;288) \psline(2;288)(2;0) \psline[linestyle=dashed](2;0)(2;144) \pscircle(0,0){2} \end{pspicture}


1

RA

FT
0 -1 -2 -2 -1 0 1 2

112

Avaliaes e Simulados Gabarito do Simulado 2 Professor Paulo Rodrigues

RA

FT

Apndice B

Competies e Prmios
B.1 Torneio Internacional das Cidades
O Torneio das Cidades uma competio organizada em Moscou desde 1980 e que permite a participao de estudantes de todo o mundo, sem haver deslocamento. As provas elaboradas pelo comit organizador so enviadas para as cidades participantes. Aps a aplicao, h uma primeira correo das provas e as melhores so traduzidas e enviadas para Moscou. L as provas so corrigidas novamente e so anunciados os ganhadores de diplomas. Existem duas modalidades: Jnior e Snior.

2009

1. Tadeu Pires de Matos Belfort Neto Fortaleza (CE) Diploma Modalidade Jnior (14,67) 2. Vincius Canto Costa Salvador (BA) Diploma Modalidade Jnior (14,67) 3. Vitor Ramos de Paula Belo Horizonte (MG) Diploma Modalidade Jnior (13,33) 4. Marina Pessoa Mota Fortaleza (CE) Diploma Modalidade Jnior (14) 5. Mateus Henrigue Ramos de Souza Pirapora (MG) Diploma Modalidade Jnior (12) 6. Victor de Oliveira Bitares Betim (MG) Diploma Modalidade Jnior (12) 7. Maria Clara Mendes Silva Pirajuba (MG) Diploma Modalidade Snior (16,25)

B.2

Asian Pacic Mathematical Olympiad


uma competio de carter internacional realizada em diversos pases asiticos e da America dedicada a estudantes do Ensino Mdio. No Brasil a olimpada APMO aplicada apenas aos alunos que tenham sido premiados na Olimpada Brasileira de Matemtica (medalhas de ouro, prata, bronze e menes honrosas). As provas dos alunos selecionados so enviadas para a comisso organizadora no Japo onde dada a classificao final.

2010 8. Maria Clara Mendes Silva Pirajuba (MG) Medalha de Bronze


113

RA

FT

114

Avaliaes e Simulados Gabarito do Simulado 2 Professor Paulo Rodrigues

2011 9. Andr Macieira Braga Costa Belo Horizonte (MG) Medalha de Prata 10. Henrique G. Fiuza do Nascimento Braslia (DF) Medalha de Bronze 11. Maria Clara Mendes Silva Pirajuba (MG) Medalha de Bronze 12. Victor de Oliveira Bitares Betim (MG) M. Honrosa

2012 13. Maria Clara Mendes Silva Pirajuba (MG) Medalha de Prata 14. Andr Macieira Braga Costa Belo Horizonte (MG) Medalha de Bronze 15. Henrique G. Fiza do Nascimento Braslia (DF) Medalha de Bronze 16. Alessandro de Oliveira Pacanowsky Rio de Janeiro (RJ) M. Honrosa B.3 Olimpada de Matemtica do Cone Sul

Competio realizada anualmente entre 8 pases sul-americanos. Os 4 estudantes de cada pas no podem ter completado 16 anos at 31 de Dezembro do ano anterior ao da realizao da prova. Comeou em 1988 e as ltimas edies aconteceram no Brasil (2010), Bolvia (2011) e Peru (2012). Em 2013 acontecer no Paraguai.

2010 Brasil

17. Maria Clara Mendes Silva Pirajuba (MG) Prata

2011 Bolvia

18. Henrique Fiuza do Nascimento Braslia (DF) Prata 19. Vincius Canto Costa Rio de Janeiro (RJ) Medalha de Prata

2012 Peru 20. Henrique Gasparini Fiuza do Nascimento Braslia (DF) Medalha de Prata B.4 Olimpada Iberoamericana de Matemtica
Competio entre mais de 20 pases Iberoamericanos. Os 4 estudantes de cada pas no podem ter completado 18 anos at 31 de Dezembro do ano anterior ao da realizao da prova. Comeou em 1985 e as ltimas edies aconteceram no Paraguai (2010), na Costa Rica (2011) e na Bolvia (2012). Em 2013 acontecer no Panam.

RA

FT

OBMEP PECI Preparao Especial para Competies Internacionais

115

2011 Costa Rica 21. Henrique Gasparini Fiza do Nascimento Braslia (DF) Medalha de Prata 22. Maria Clara Mendes Silva Pirajuba (MG) Medalha de Prata 23. Andr Macieira Braga Costa Belo Horizonte (MG) Medalha de Bronze

2012 Bolvia 24. Andr Macieira Braga Costa Belo Horizonte (MG) Medalha de Prata B.5 Romanian Masters in Mathematics
Desde 2010 o Brasil participa da Romanian Master in Mathematics, olimpada que convoca apenas os melhores pases do mundo em competies internacionais do gnero. A Romanian Master in Mathematics (RMM) organizada desde 2007 pela Escola Nacional de Informtica Tudor Vianu em colaborao com a Sociedade Cientfica Romena de Matemtica e o Ministrio de Educao Investigao e Juventude.

25. Maria Clara Mendes Silva Pirajuba (MG) Meno Honrosa

2012 Romnia

26. Maria Clara Mendes Silva Pirajuba (MG) Medalha de Bronze

28. Andr Macieira Braga Costa Belo Horizonte (MG) Meno Honrosa B.6

Olimpada de Matemtica dos Pases de Lngua Portuguesa

2012 Brasil 29. Daniel Santana Rocha Rio de Janeiro (RJ) Medalha de Ouro B.7 Olimpada Rioplatense de Matemtica
Competio realizada anualmente na Argentina em 4 nveis com a participao de cidades ou pases convidados. So Paulo e Fortaleza participam h mais de 15 anos desta competio.

27. Henrique Gasparini Fiza do Nascimento Braslia (DF) Meno Honrosa

RA

2011 Romnia

FT

116

Avaliaes e Simulados Gabarito do Simulado 2 Professor Paulo Rodrigues

2011 Argentina 30. Gabriel Fazoli Domingos (SP) Medalha de Prata Nvel 1

2012 Argentina 31. Pedro Henrique Alencar Costa Fortaleza (CE) Medalha de Prata Nvel 1 B.8 Olimpada Internacional de Matemtica
A primeira edio foi realizada em 1959 com a participao de pases do Leste Europeu. O Brasil comeou a participar em 1979 e sediar a competio pela primeira vez em 2017. Cada pas compete com 6 estudantes que ainda no tenham ingressado no nvel superior. Em 2011 a IMO foi realizada na Holanda, em 2012 na Argentina, em 2013 ser

2011 Holanda

32. Andr Macieira Braga Costa Belo Horizonte (MG) Medalha de Prata

33. Henrique Gasparini Fiza do Nascimento Braslia (DF) Medalha de Prata 34. Maria Clara Mendes Silva Pirajuba (MG) Medalha de Bronze

2012 Argentina

35. Henrique Gasparini Fiza do Nascimento Braslia (DF) Medalha de Bronze 36. Maria Clara Mendes Silva Pirajuba (MG) Meno Honrosa

RA

FT

na Colmbia, em 2014 na frica do Sul e em 2015 na Tailndia.

OBMEP PECI Preparao Especial para Competies Internacionais

117

Principais Competies de Matemtica para estudantes do Ensino Bsico


.
Olimpada Brasileira de Matemtica (OBM)
realizada em 3 fases (Junho, Setembro e Outubro) com a participao de alunos do Fundamental II e do Ensino Mdio, divididos em 3 nveis. As duas primeiras fases so aplicadas pela prpria escola do aluno. A terceira fase aplicada em local determinado pelo coordenador regional. Ser premiado nesta competio pr-requisito para participar do processo seletivo para as principais competies internacionais de matemtica.

Matemtica Olimpada de

l do Cone Su

raguai. tecer no Pa

Outras Competies Internacionais: Olimpada de Maio (para alunos de at 15 anos) e APMO (para alunos no Ensino Mdio)
nas quais os estudantes participam sem precisar se deslocar. Alm destas, h tambm a Olimpada Rioplatense de Matemtica, da qual participam anualmente na Argentina estudantes paulistas e cearenses premiados em competies regionais e o Torneio Internacional das Cidades, tradicional competio organizada em Moscou e aplicada em cidades dos cinco continentes.

RA

8 pases lmente entre alizada anua mpetio re Co pas no ntes de cada Os 4 estuda o l-americanos. su de Dezembr anos at 31 mpletado 16 meou podem ter co a. Co zao da prov ior ao da reali do ano anter no Brasil aconteceram es ltimas edi em 1988 e as 2013 aconru (2012). Em ia (2011) e Pe (2010), Bolv

(IMO) onal de Matemtica Olimpada Internaci

einternacional de Mat respeitada competio A mais tradicional e ticiem 1959 com a par edio foi realizada mtica, cuja primeira a participar u. O Brasil comeou Leste Europe pao de pases do a eira vez em 2017. Cad competio pela prim em 1979 e sediar a essado a no tenham ingr estudantes que aind pas compete com 6 Holanda, em IMO foi rea1lizada na 2011 a no nvel superior. Em bia, em 2014 na frica 2013 ser na Colm 2012 na Argentina, em Tailndia. do Sul e em 2015 na

FT

Olimpada Iberoame ricana de Matemtica

Competio entre mai s de 20 pases Iberoam ericanos. Os 4 estudantes de cada pas no pod em ter completado 18 ano s at 31 de Dezemb ro do ano anterior ao da realiza o da prova. Comeo u em 1985 e as ltimas edi es aconteceram no Paraguai (2010), na Costa Rica (2011) e na Bolvia (20 12). Em 2013 acontecer no Panam.

S-ar putea să vă placă și